THE ART AND PLAN OF CROSS EXAMINATION

THE ART AND PLAN OF CROSS EXAMINATION

BY: GAZAL BHATNAGAR,VIDHYARTHI DARPAN

Q.) WHAT DO YOU MEAN BY CROSS EXAMINATION? WHY IS IT IMPORTANT?

Ans) Cross examination is a technique that is implemented and practiced by prosecutors by the means of direct examination. It is used to enquire averse deponent in depositions and in civil and criminal trials.

Cross examination is a sharp art in which the probability of explanation of witness answers are low. Basically, used to impeach a witness in the following ways:

  • It reveals the witness character whether biased, prejudiced, lying or is an egotism.
  • Shows witness past behavior usually dismally or has been charged with a crime like deceit and lying.
  • Uncovers witness reputation when it comes to truth telling.

Cross examination is a type of interrogation by the opposing party of the witness that is called for testing. If a party calls a witness to provide testimony and statements in its favor, the party opposing the claim has the right to examine such a witness. Examination- in- chief occurs after cross examination which is the examination of the witness by the party being called. Both examination must relate to the relevant facts.

GOALS OF CROSS EXAMINATION:

  • Authenticates the evidence of the state’s witnesses.
  • Acquires genuine acknowledgement that are supportive to the state’s case.
  • Lessen the defense case by impeachment of the witness on the stand.
  • Minimize the defense case by impeachment of other defense witnesses through the witness on the stand.

THE TEN COMMANDMENTS WHILE CROSS EXAMINING:

  • Be brief.
  • Use short questions with plain words.
  • Ask only leading and suggestive questions.
  • Never ask a question to which you do not know the answer.
  • Listen to the answers given by the witness.
  • Do not quarrel with the witness.
  • Don’t let the evidence (witness) clarify.
  • Do not ask the witness to repeat the testimony he/she gave in-chief.
  • Avoid long questions.
  • Save the explanation for summation.

IMPORTANCE OF CROSS EXAMINATION:

  • Builds credibility.
  • Creates clarity.
  • Constructs cracks.
  • Used for confirming and evaluating the truth of a person’s testimony, to develop the testimony further and to achieve other objectives.
  • The aim of a cross examination is to investigate the evidence and try the reliability of a witness who has been called and given proof in-chief.
  • Cross examination is an effective tool which is devised for the ascertainment of truth, discredit and impeach the testimony of a witness, assist evidences that are favorable to the side of a cross examiner and provides independent proofs beneficial to the position of a cross examiner.

It is a very powerful mechanism which testifies the veracity of a witness and the accuracy or completeness of what he/she says. It is a form of intensive questioning with the expected answers hinted to in such questions itself.

The cross examination is also known as” ex-adverso”, used to expose the inaccuracies of the evidence of a particular witness.

Section 146 in the Indian Evidence Act 1872 says that a witness during cross examination, may, in addition to the questions herein before referred to, be asked any questions which tend

  • To test his veracity.
  • To find out who he is and what his place in life is.
  • Shaking his reputation by damaging his character whether the answer to these questions may tend to criminalize him directly or indirectly, or may expose him or tend to expose him to a fine or revocation, directly or indirectly.

 

Q.) GIVE A LIST OF DO’S AND DON’TS FOR AN EFFECTIVE CROSS EXAMINATION.

Ans) DO’S OF CROSS EXAMINATION:

1. Have a command on leading questions only-:

By doing so, the witness is permitted to answer only “yes” or “no”.

2. Restrictions on questions-:

Try to control the questions you ask. Use only simple and easy questions. Do not ask open ended questions, narrative generating questions, compound, and multipart questions.

3. Limit the level of flow-:

Conduct cross examination in an order and by subject matter. Always ask questions in a sequence that tells a story which is followed easily. Try to use descriptive words to highlight the significant points. Impeach the credibility of a witness before attacking the credibility of his/her testimony.

4. Restrain to the subject matter-:

Do not allow the witness to restate his straight testimony. Refuse to explore uncharted territory and use verbatim notes on the witness’s direct testimony.

5. Be polite and courteous-:

Create the impression of being friendly and fair. Do not be harsh and overbearing. This will make a pleasant courtroom atmosphere, appealing to jurors, and does not alienate them from your client.

6. Keep faith on questions that you ask-:

Have a good faith basis for questions that you ask, if challenged by your opponent and are unable to provide sufficient proof for your question.

7. Begin with major points-:

When the jury finishes listening to the direct examination then start their thought processes to be in your favor by using extensive details.

8. Wind-up the cross examination with favorable points-:

Leave strong impression and thoughts on jury while ending the case using major points.

9. Organize the case-:

If the case is well ordered, the jury will not only appreciate the lawyer but also move the case rapidly and back to work sooner.

10. Use of space in the courtroom-:

Try to maintain distance and position. It keeps the jury more alert and contributes to emphasis and effect.

DON’TS OF CROSS EXAMINATION:

1. Never argue with a witness-:

The examiner should not argue or quarrel with the witness. Overcome the situation by eliciting the answers you need or use skillful questioning for the responses.

2. Don’t answer the questions for an opposing witness-:

The examiner should never respond to the opposing party, as by doing so the control goes in the favor of the witness rather to the lawyer/examiner.

3. Never argue with the judge-:

Try to make strongest argument while being respectful towards the judge. Do not argue in front of the jury and audience as it will lead to a mortal sin against self-control. Be a loyal examiner during the case as much as you can.

4. Don’t persecute yourself by your opponent-:

The opposing counsel will try to bait you so that you may lost your control and make mistakes. Do not be tempted and baited (mostly used in depositions, courtrooms and even over phones). Stay confident and play your game without losing self-control and concentration at your client’s expense.

5. Don’t let the jury know and see that your case has been hurt by an answer-:

Never reform your demeanor, expression, and the pace of your cross examinations. If you do, you risk revealing that your case has been hurt and damaged. This time requires utmost self-control. 

6. Don’t “KILL” a witness unless the jury wants him to be demolished-:

Never try to stab the bull. If the examiner plunges the sword into the body of the bull, the crowd might feel sorry for that bull.

7. Don’t cross examine if there is no impact-:

If no result, outcome, and impression appears by cross examining then announce that you have no questions. This itself will carry a big impact.

8. Don’t re-affirm points that hurt you-:

Avoid giving opportunity to the witness who has given direct testimony against you and to repeat that testimony.

9. Don’t rely on the judge to discipline the witness-:

Try to control the witness by your own. Do not call upon judge as it will lead to the weakness of courtroom. Judge should be used as a last resort.

10. Don’t be cocky and arrogant-:

Try to keep the tone in a respectful and truth-seeking manner. Ask as many killing questions as you can but do not let the jury to see you as a conceited examiner.

CLAT 2017 Previous Year Paper

CLAT 2017

Q. 1 Fill in the blank by choosing the most appropriate option.

We shall fail ________ we are industrious.

A. whether

B. unless

C. until

D. though

 

Q. 2 Fill in the blank by choosing the most appropriate option.

Sunita decided to set ______ some time every day for prayers.

A. up

B. aside

C. on

D. in

 

Q. 3 Fill in the blank by choosing the most appropriate option.

If they want to succeed, they ______ have to work very hard

A. ought

B. should

C. must

D. will

 

Q. 4  Fill in the blank by choosing the most appropriate option.

She stood ______ Amit, but could not utter a single word for quite some time.

A. for

B. to

C. before

D. about

 

Q. 5 Fill in the blank by choosing the most appropriate option.

The passengers were very happy _______ the friendly and warm treatment.

A. from

B. to

C. about

D. by

 

Q. 6 Fill in the blank by choosing the most appropriate option.

The doctor advised him to go ______ several medical tests.

A. through

B. about

C. under

D. into

 

Q. 7 Fill in the blank by choosing the most appropriate option.

Would anybody ______ a mother have risked her life for the baby?

A. however

B. rather

C. than

D. but

 

Q. 8 Fill in the blank by choosing the most appropriate option

The minister flew ______ the flooded areas in a helicopter

A. in

B. about

C. over

D. along

 

Q. 9 Fill in the blank by choosing the most appropriate option.

Kanak is endowed _______ many great qualities.

A. of

B. in

C. by

D. with

 

Q. 10  Fill in the blank by choosing the most appropriate option

You have played a great role, for _______ your help I possibly would have landed myself into a problem.

A. despite

B. although

C. after

D. without

 

Questions: 11 – 15

Direction for Questions 11 – 15: Read the given passage carefully and choose the most appropriate option to the questions given below.

The World Trade Organisation (WTO) was created in the early 1990s as a component of the Uruguay Round negotiation. However, it could have been negotiated as part of the Tokyo Round of the 1970s, since negotiation was an attempt at a ‘constitutional reform’ of the General Agreement on Tariffs and Trade (GATT). Or it could have been put off to the future, as the US government wanted. What factors led to the creation of the WTO in the early 1990s? One factor was the pattern of multilateral bargaining that developed  late in the Uruguay Round. Like all complex international agreements, the WTO was a product of a series of trade-offs between principal actors and groups. For the United States, which did not want a new organization, the disputed settlement part of the WTO package achieved its longstanding goal of a more effective and more legal dispute settlement system. For the Europeans, who by the 1990s had come to view GATT dispute settlement less in political terms add more as a regime of legal obligations, the WTO package was acceptable as a means to discipline the resort to unilateral measures by the United States. Countries like Canada and other middle and smaller trading partners were attracted by the expansion of a rulebased system and by the symbolic value of a trade organization, both of which inherently support the weak against the strong. The developing countries were attracted due to the provisions banning unilateral measures. Finally, and perhaps most important, many countries at the Uruguay Round came to put a higher priority on the export gains than on the import losses that the negotiation would produce, and they came to associate the WTO and a rule-based system with those gains. This reasoning – replicated in many countries – was contained in U. S. Ambassador Kantor’s defence of the WTO, and it announced to a recognition that international trade and its benefits cannot be enjoyed unless trading nations accept the discipline of a negotiated rule-based environment. A second factor in the creation of the WTO was pressure from lawyers and the legal process. The dispute settlement system of the WTO was seen as a victory of legalists but the matter went deeper than Aglasem Admission that. The GATT, and the WTO, are contract organizations based on rules, and it is inevitable that an organization creating a further rule will in turn be influenced by legal process. Robert Hudee has written of the ‘momentum of legal development’, but what is this precisely? Legal development can be defined as promotion of the technical legal values of consistency, clarity (or certainty) and effectiveness; these are values that those responsible for administering any legal system will seek to maximize. As it played out in the WTO, consistency meant integrating under one roof the whole lot of separate agreements signed under GATT auspices; clarity meant removing ambiguities about the powers of contracting parties to make certain decisions or to undertake waivers; and effectiveness meant eliminating exceptions arising out of grandfather-rights and resolving defects in dispute settlement procedures and institutional provisions. Concern for these values is inherent in any rule-based system of co-operation, since without these value rules would be meaningless in the first place, therefore, create their own incentive for fulfilment. The moment of legal development has occurred in other institutions besides the GATT, most notably in the European Union (EU). Over the past two decades the European Court of Justice (ECJ) has consistently rendered decisions that have expanded incrementally the EU’s internal market, in which the doctrine of ‘mutual recognition’ handed down in Cassis de Dijon case in 1979 was a key turning point. The court is now widely recognized as a major player in European integration, even though arguably such a strong role was not originally envisaged in the Treaty of Rome, which initiated the current European Union. One means the Court used to expand integration was the ‘teleological method of interpretation’, whereby the actions of member states were evaluated against ‘the accomplishment of the most elementary goals set forth in the Preamble to the (Rome) treaty. The teleological method represents an effort to keep current policies consistent with slated goals, and it is analogous to the effort in GATT to keep contracting party trade practices consistent with slated rules. In both cases legal concerns and procedures are an independent force for further co-operation. In the large part the WTO was an exercise in consolidation. In the context of a trade negotiation that created a near-revolutionary expansion of international trade rules, the formation of the WTO was a deeply conservative act needed to ensure that the benefits of the new rules would not be lost. The WTO was all about institutional structure and dispute settlement: these are the concerns of conservatives and not revolutionaries, that is why lawyers and legalists took the lead on these issues. The WTO codified the GATT institutional practice that had developed by custom over three decades, and it incorporated a new dispute settlement system that was necessary to keep both old and new rules from becoming a sham. Both the international structure and the dispute settlement system were necessary to preserve and enhance the integrity of the multilateral trade regime that had been built incrementally from the 1940s to the 1990s. 

 

Q. 11 In the statement ‘… it amounted to a recognition that international trade and its benefits cannot be enjoyed unless trading nations accept the discipline of a negotiated Aglasem Admission rule-based environment’, it refers to:

A. .The export gains many countries came to associate with a rule-based system.

B. The higher priority on export gains placed by many countries at the Uruguay The higher priority on export gains placed by many countries at the Uruguay Round.

C. The provision of a rule-based system by the WTO.

D. Ambassador Kantor’s defence of the WTO.

 

Q. 12 What would be the closest reason why WTO was not formed in 1970s?

A. The US government did not like it.

B. Important players did not find it in their best interest to do so.

C. Lawyers did not work for the dispute settlement system.

D. The Tokyo Round negotiations was an attempt at constitutional reform

 

Q. 13 In the method of interpretation of the European Court of Justice:

A. Actions against member states needed to be evaluated against the said community goals.

B. Enunciation of the most elementary community goals needed to be emphasized

C. Current policies need to be consistent with stated goals.

D. Contracting party trade practices need to be consistent with stated rules.

 

Q. 14 According to the passage, WTO promoted the technical legal values partly Through. 

A. Integrating under one roof the agreements signed under GATT.

B. Rules that create their own incentive for fulfilment.

C. Ambiguities about the powers of contracting parties to make certain decisions.

D. Grandfather-rights exceptions and defects in dispute settlement procedures

 

Q. 15 The most likely reason for the acceptance of the WTO package by nations was that:

A. It has the means to prevent the US from taking unilateral measures.

B. Its rule-based system leads to export gains.

C. It settles disputes more legally and more effectively

D. They recognized the need for a rule-based environment to protect the benefits of increased trade.

 

Q. 16 In each of the following sentences, some part of the sentence or the entire sentence is underlined. Beneath each sentence, you will find four ways of phrasing the underlined part. Choose the most appropriate option given in each of the sentences given below that is the best version than the underlined part of the sentence

A. will allow laws on be proposed, passed, as well as repealed by voters

B. allows voters to propose, pass and to repeal laws

C. will allow voter to propose, pass, as well as to repeal laws

D. allows voters to propose to pass, and repeal laws

 

Q. 17 In each of the following sentences, some part of the sentence or the entire sentence is underlined. Beneath each sentence, you will find four ways of phrasing the underlined part. Choose the most appropriate option given in each of the sentences given below that is the best version than the underlined part of the sentence

A. The mountainous regions are dotted with hundreds of monasteries, many of them chiselled from solid rock centuries ago.

B. The mountainous regions are dotted with hundreds of monasteries, many of which are chiselled from solid rock centuries ago

C. chiselled from solid rock centuries ago, the mountainous regions are dotted with many hundreds of monasteries,

D. Hundreds of monasteries, many of them chiselled from solid rock centuries ago, are dotting the mountainous regions.

 

Q. 18 Direction for questions 16 – 20: In each of the following sentences, some part of the sentence or the entire sentence is underlined. Beneath each sentence, you will find four ways of phrasing the underlined part. Choose the most appropriate option given in each of the sentences given below that is the best version than the underlined part of the sentence

A. even though it sounds plausible, the weakness of the hypothesis

B. the weakness of the hypothesis which sounds plausible.

C. though the hypothesis sounds plausible, its weakness

D. though plausible, the hypothesis’ weakness

 

Q. 19 Direction for questions 16 – 20: In each of the following sentences, some part of the sentence or the entire sentence is underlined. Beneath each sentence, you will find four ways of phrasing the underlined part. Choose the most appropriate option given in each of the sentences given below that is the best version than the underlined part of the sentence 

A. With everyone attending not knowledgeable.

B. For everyone in attendance did not know.

C. For not everyone attending knew.

D. With everyone in attendance not knowing.

 

Q. 20 Direction for questions 16 – 20: 

In each of the following sentences, some part of the sentence or the entire sentence is underlined. Beneath each sentence, you will find four ways of phrasing the underlined part. Choose the most appropriate option given in each of the sentences given below that is the best version than the underlined part of the sentence

A. he other one a deficit of six, occur in Nature and contribute

B. the other with a deficit of six, occur in Nature and contribute

C. the other with a deficit of six, occurs in Nature and contributes

D. one with six occurs in Nature and contributes

 

Q. 21 Choose the correct spellings in options given below

A. Bleshphemy

B. Blasphamy

C. Blasphemy

D. Blashphemy

 

Q. 22 Choose the correct spellings in options given below

A. Gallows

B. Ghellows

C. Gellows

D. Ghallows

 

Q. 23 Choose the correct spellings in options given below

A. Remuneration

B. Remunaration

C. Renumeration

D. Renumaration

 

Q. 24 Choose the correct spellings in options given below

A. Accommedation

B. Accomadation

C. Accommodation

D. Accomedation

 

Q. 25 Choose the correct spellings in options given below

A. Annulment

B. Annulmant

C. Anulment

D. Annulement

 

Q. 26 Choose the correct spellings in options given below

A. Gratuitous

B. Gratuitus

C. Gratitious

D. Gratetious

 

Q. 27 Choose the correct spellings in options given below

A. Hypothecation

B. Hyphothecation

C. Hypathecation

D. Hypthacation

 

Q. 28 Choose the correct spellings in options given below

A. Interrogation

B. Interagation

C. Interogetion

D. Interogation

 

Q. 29 Choose the correct spellings in options given below

A. Maratorium

B. Moretorium

C. Moratarium

D. Moratorium

 

Q. 30 Choose the correct spellings in options given below

A. Abeyance

B. Abhayence

C. Abeyence

D. Abheyance

 

Q. 31 Directions for questions 31 – 40: In each of the following sentences four words or phrases are underlined. If there is any mistake with regard to grammar or usage, it is in the underlined part only. Identify the incorrect part

A. A

B. D

C. B

D. C

 

Q. 32 Directions for questions 31 – 40: In each of the following sentences four words or phrases are underlined. If there is any mistake with regard to grammar or usage, it is in the underlined part only. Identify the incorrect part

A. A

B. B

C. D

D. C

 

Q. 33 Directions for questions 31 – 40: In each of the following sentences four words or phrases are underlined. If there is any mistake with regard to grammar or usage, it is in the underlined part only. Identify the incorrect part

A. A

B. B

C. C

D. D

 

Q. 34 Directions for questions 31 – 40: In each of the following sentences four words or phrases are underlined. If there is any mistake with regard to grammar or usage, it is in the underlined part only. Identify the incorrect part

A. A

B. D

C. B

D. C

 

Q. 35 Directions for questions 31 – 40: In each of the following sentences four words or phrases are underlined. If there is any mistake with regard to grammar or usage, it is in the underlined part only. Identify the incorrect part

A. A

B. D

C. C

D. B

 

Q. 36 Directions for questions 31 – 40: In each of the following sentences four words or phrases are underlined. If there is any mistake with regard to grammar or usage, it is in the underlined part only. Identify the incorrect part

A. A

B. D

C. C

D. B

 

Q. 37 Directions for questions 31 – 40: In each of the following sentences four words or phrases are underlined. If there is any mistake with regard to grammar or usage, it is in the underlined part only. Identify the incorrect part

A. C

B. A

C. B

D. D

 

Q. 38 Directions for questions 31 – 40: In each of the following sentences four words or phrases are underlined. If there is any mistake with regard to grammar or usage, it is in the underlined part only. Identify the incorrect part

A. A

B. D

C. B

D. C

 

Q. 39 Directions for questions 31 – 40: In each of the following sentences four words or phrases are underlined. If there is any mistake with regard to grammar or usage, it is in the underlined part only. Identify the incorrect part

A. C

B. B

C. D

D. A

 

Q. 40 Directions for questions 31 – 40: In each of the following sentences four words or phrases are underlined. If there is any mistake with regard to grammar or usage, it is in the underlined part only. Identify the incorrect part

A. C

B. A

C. B

D. D

 

Q. 41 Choose the most appropriate option.

Donald Trump is _____________ President of the United States.

A. 46th

B. 43rd

C. 45th

D. 44th

 

Q. 42 Choose the most appropriate option

With the development of Terahertz (THz) transmitter, it is expected to be faster than 5G

mobile networks by:

A. Four Times

B. Five Times

C. Two TImes

D. Ten TImes

 

Q. 43 Choose the most appropriate option

Among the following professors of Indian origin who has received Knighthood for the work as a co-inventor of next generation DNA Sequencing called Solexa Sequencing is?

A. P. C. Mahalanobis

B. Mehnad Saha

C. Shankar Balasubramanian

D. Satyendra Nath Bose

 

Q. 44 Choose the most appropriate option

Which of the following country enacted a law during August 2016 providing for the right to register the marriages of Hindus?

A. Pakistan

B. Iran

C. Saudi Arabia

D. Afghanistan

 

Q. 45 Choose the most appropriate option

The top destination for domestic tourists in India for the past three consecutive years has been: 

A. Orissa

B. Rajasthan

C. Kerala

D. Tamil Nadu

 

Q. 46 Choose the most appropriate option

The Japanese Prime Minister who offered ‘sincere and everlasting condolences’ to the people of the United States for killing more than 2,400 soldiers in the attack on Pearl harbour was:

A. Hayato Ikeda

B. Kakuei Tanaka

C. Juniciro Koizumi

D. Shinzo Abe

 

Q. 47 Choose the most appropriate option

NASA rediscovered India’s lunar spacecraft that was lost in the space during the past eight years known as:

A. Mangalayaan – II

B. Mangalayaan – I

C. Chandrayan – II

D. Chandrayan – I

 

Q. 48 Choose the most appropriate option

India’s third largest trading partner during 2016 is

A. Kuwait

B. Saudi Arabia

C. Dubai

D. U.A.E.

 

Q. 49 Choose the most appropriate option

Who called the immigration the ‘Trojan horse of Terrorism’?

A. Theresa May, Prime Minister of U.K

B. Angela Merkel, Chancellor of Germany

C. Viktor Orban, Prime Minister of Hungary

D. Donald Trump, President of the U.S

 

Q. 50 Choose the most appropriate option

The first country to have announced euthanasia of a child is:

A. Belgium

B. Norway

C. Denmark

D. Finland

 

Q. 51 Choose the most appropriate option

The bowler who has claimed the fastest 250 wickets in Cricket test matches is:

A. Dennis Likee

B. Rangana Herath

C. Anil Kumble

D. Ravichandran Ashwin

 

Q. 52 Choose the most appropriate option

Volvo has launched the world’s largest bus that can carry up to:

A. 300 passengers

B. 150 passengers

C. 260 passengers

D. 320 passengers

 

Q. 53 Choose the most appropriate option

According to the Survey Report released by Transparency International during March 2017 on India, the most corrupt are

A. Police

B. Local Councilors

C. Business Executives

D. Government Officials

 

Q. 54 Choose the most appropriate option

Name the President elect of France who is likely to take the oath on 14 May 2017.

A. Emmanuel Macron

B. Marine Le Pen

C. Francois Holland

D. Francois Fillon

 

Q. 55 Choose the most appropriate option

The 2016 Nobel Peace Prize was won by the President of :

A. South Africa

B. United States of America

C. Sri Lanka

D. Columbia

 

Q. 56 Choose the most appropriate option

Japan is threatening to drag India to W.T.O on issues relating to the export of its:

A. Steel

B. Electronic Goods

C. Tea

D. Small Ships

 

Q. 57 Choose the most appropriate option

In terms of steel production in the world during 2015 – 2016, India stood at:

A. Second

B. Third

C. Fourth

D. Fifth

 

Q. 58 Choose the most appropriate option

India’s voting rights at the International Monetary Fund increased from 2.3 % to

A. 2.5%

B. 2.8%

C. 2.6%

D. 2.4%

 

Q. 59 Choose the most appropriate option

The first elected civilian President in Myanmar is:

A. Aung San Suu Kyi

B. Khin Ayi

C. Htin Kyaw

D. Aung San

 

Q. 60 Choose the most appropriate option

Prithvi Defence Vehicle is

A. The name of an amphibious Naval vessel

B. The name of a newly developed tank

C. The name of a single seated aircraft developed by HAL

D. The name of India’s Nuclear Intercepter Missile

 

Q. 61 Choose the most appropriate option

Which shoe company in the United States of America has won an IPR dispute against China recently for using their logo?

A. Reebok

B. Adidas

C. Nike

D. New Balance

 

Q. 62 Choose the most appropriate option

Who has been selected for 2016 BC Roy Award?

A. Dr. N. Bhaskaran

B. Dr. J. Rajendra

C. Dr. Jagat Ram

D. Dr. P. Raghu Ram

 

Q. 63 Choose the most appropriate option

The world’s first artificial intelligence lawyer, a robot, is named as:

A. IDM-LaBrain

B. Boss

C. Ross

D. Watson

 

Q. 64 Choose the most appropriate option

The Hubble telescope of NASA is located in

A. NASA Headquaters

B. Canada

C. Space

D. Iceland

 

Q. 65 Choose the most appropriate option

Which court has stayed the execution of death sentence of Kulbhushan Jadhav in May 2017?

A. International Court of Justice

B. Supreme Court of Pakistan

C. Supreme Court of India

D. International Criminal Court

 

Q. 66 Choose the most appropriate option

Till the end of 2016, the total number of UNESCO’s World Heritage Sites in India is:

A. 18

B. 21

C. 42

D. 35

 

Q. 67 Choose the most appropriate option

Highest number of open prisons in India as on 2015 are in

A. Rajasthan

B. Kerala

C. Tamil Nadu

D. Maharastra

 

Q. 68 Choose the most appropriate option

Among the following, who has won the maximum number of titles?

A. Gopichand

B. P. V. Sindhu

C. Saina Nehwal

D. Srikant

 

Q. 69 Choose the most appropriate option

How many billionaires India has lost since demonetization on November 8, 2016?

A. Two

B. Eleven

C. Eighteen

D. Eight

 

Q. 70 Choose the most appropriate option

Juno is the name of a:

A. Hydrogen fuelled submarine

B. Hydrogen fuelled space craft

C. Solar powered space craft

D. Atomic powered submarine

 

Q. 71 Choose the most appropriate option

The Union Cabinet has recently approved the setting up of a Permanent Tribunal for resolving:

A. Complaints from three services

B. Inter-state water disputes

C. Inter-state boundary disputes

D. Election disputes

 

Q. 72 Choose the most appropriate option

World’s longest rail tunnel is about:

A. 23 kms

B. 35 kms

C. 47 kms

D. 57 kms

 

Q. 73 Choose the most appropriate option

The first Commercial Court and Commercial Disputes Resolution Centre was inaugurated at

A. Visakapatinam, Andhra Pradesh

B. Mumbai, Maharastra

C. Raipur, Chattisgarh

D. Ahmedabad, Gujarat

 

Q. 74 Choose the most appropriate option

Solar Impulse–2 is:

A. Hybrid airplane

B. Solar powered ship

C. Impact of climate change for overall increase of 2º C a year

D. Solar powered airplane that completed the first around the world

 

Q. 75 Choose the most appropriate option

During 2017, which SAARC country has notified the Right to Information Act?

A. Afghanistan

B. Bhutan

C. Nepal

D. Sri Lanka

 

Q. 76 Choose the most appropriate option

‘World’s longest-all women Non-stop flight’ from New Delhi to San Francisco covering

14,500 kms was operated by:

A. Jet Airways

B. Air India

C. United AIrlines

D. American Airlines

 

Q. 77 Choose the most appropriate option

‘Scorpion kick’ is a phrase used in

A. Kung Fu

B. Kabaddi

C. Kalari Fight

D. Kick Boxing

 

Q. 78 Choose the most appropriate option

Which country offered asylum seekers 1,200 Euros to leave by withdrawing their application for protection?

A. Denmark

B. France

C. Germany

D. Italy

 

Q. 79 Choose the most appropriate option

The Happiness Index Department or a Wing has been established in the states of:

A. Andhra Pradesh and Madhya Pradesh

B. Madhya Pradesh and Goa

C. Mashya Pradesh and Tamil Nadu

D. Andhra Pradesh and Sikkim

 

Q. 80 Choose the most appropriate option

Which of the following individuals was called a ‘deceptive actor’ by China’s foreign ministry during March 2017?

A. Narendra Modi

B. Dalai Lama

C. Donald Trump

D. Sirisena

 

Q. 81 Choose the most appropriate option

The World Bank had cut India’s GDP growth for 2016 – 2017 to:

A. 8%

B. 7.6%

C. 8.6%

D. 7%

 

Q. 82 Choose the most appropriate option

Among the following who was crowned as ‘Miss Supernational’ during 2016?

A. Srinidhi Shetty

B. Shilpa Shetty

C. Alia Bhat

D. Aishwarya Rai

 

Q. 83 Choose the most appropriate option

COIN, a software programme developed by J. P. Morgan supports:

A. Interpreting Commercial Documents

B. Financial Accounting

C. Robotic Surgery

D. Bitcoin

 

Q. 84 Choose the most appropriate option

The first country in the world to have begun shutting down the entire Frequency Modulation (FM) radio network to be replaced by Digital Audio Broadcasting is: 

A. China

B. United States of America

C. Norway

D. Switzerland

 

Q. 85 Choose the most appropriate option

Among the following M.L.As, who was disqualified by the Governor during January 2017 under Article 192 of the Constitution for undertaking government contracts?

A. Srinivas Prasad of Karnataka

B. Jayalalithaa of Tamil Nadu

C. Uma Shankar Singh of Bihar

D. P.C. George of Kerala

 

Q. 86 Choose the most appropriate option

India emerged as ___________ largest holder of the U.S Government Securities at the end of 2016:

A. Twelfth

B. Twenty Eight

C. Twenty Fourth

D. Fifteenth

 

Q. 87 Choose the most appropriate option

The first statue of a woman in Parliament Square in England is that of:

A. Margaret Thatcher

B. Queen Elizabeth – II

C. Mother Theresa

D. Millicent Fawcett

 

Q. 88 Choose the most appropriate option

Which country 3D – Printed a home of 37 sq.mts?

A. Spain

B. Russia

C. South Korera

D. Taiwan

 

Q. 89 Choose the most appropriate option

As on 31st January 2016, the highest number of law colleges were present in:

A. Madhya Pradesh

B. Andhra Pradesh

C. Maharashtra

D. Uttar Pradesh

 

Q. 90 Choose the most appropriate option

Immediately before Antonio Guterres was appointed the U.N Secretary General in October 2016, he was:

A. Commissioner General of UNRWA

B. United Nations High Commissioner for Refugees

C. The Prime Minister of Portugueses

D. United States High Commissioner for Human Rights

 

Q. 91 Age of father 10 years ago was three times the age of his son. After 10 years, father’s age is twice that of his son. The ratio of their present ages is:

A. 11:7

B. 7:4

C. 9:5

D. 7:3

 

Q. 92 A clock was set correct at 12 O’ clock. It loses 10 minutes per hour.What will be the angle between the hour and minute hands of the clock after one hour?

A. 90º

B. 85º

C. 75º

D. 105º

 

Q. 93 The Banker’s discount on a sum of money for 18 months is Rs. 600 and the true discount on the same sum for 3 years is Rs. 750/-. The rate percentage is:

A. 10%

B. 20%

C. 15%

D. 12%

 

Q. 94 The average weight of three men ‘X’, ‘Y’ and ‘Z’ is 75 kgs. Another man ‘A’ joins the group and the average weight now becomes 80 kgs. If another person ‘B’ whose weight is 5 kgs more than ‘A’ replaces ‘X’, then the average weight of ‘Y’, ‘Z’, ‘A’ and ‘B’ will be 85 kgs. What is the weight of ‘X’?

A. 84 kgs.

B. 82 kgs.

C. 78 kgs.

D. 80 kgs.

 

Q. 95 In an office, 1/3 of the workers are Men, ½ of the men are married and 1/3 of the married men have children. If ¾ of the women are married and 2/3 of the 2/3 of the married women have children, then the part of workers without children are:

A. 11/18

B. 17/36

C. 5/18

D. 4/9

 

Q. 96 The difference between simple interest and compound interest at the same rate for rupees 5,000 for two years is rupees 98. The rate of interest is:

A. 14%

B. 10%

C. 10 ½ %

D. 12%

 

Q. 97 A vessel contains a mixture of milk and water in the ratio of 5:3 respectively. How much of the mixture must be siphoned off and replaced with water, so that the mixture may be half milk and half water?

A. 1/7

B. 1/4

C. 1/5

D. 1/3

 

Q. 98 Praveen has Rs. 4,662 in the form of 2, 5 and 10 rupee notes. If these notes are in the ratio of 3:5:8, the number of five rupees notes with him is:

A. 336

B. 250

C. 84

D. 210

 

Q. 99 ‘A’ and ‘B’ complete a work in 12 days, ‘B’ and ‘C’ in 8 days and ‘C’ and ‘A’ in 16 days. ‘A’ left after working for 3 days. In how many days more will ‘B’ and ‘C’ finish the remaining work?

A. 7 ¾

B. 6 ⅚

C. 4 ¾

D. 3 ¾

 

Q. 100 A train ‘X’ leaves station ‘A’ at 3 p.m and reaches station ‘B’ at 4.30 p.m., while another train ‘Y’ leaves station ‘B’ at 3.00 p.m and reaches station ‘A’ at 4.00 p.m. These two trains cross each other at:

A. 3.36 p.m.

B. 3.30 p.m

C. 3.20 p.m.

D. 3.40 p.m

 

Q. 101 A trader sells rice at a profit of 20% and uses weights which are 10% less than the correct weight. The total gain earned by him is:

A. 22 2/9%

B. 35%

C. 33 1/3%

D. 30%

 

Q. 102 Keerthi’s father gave him some money to buy books. He spent half of the money equally to buy books and entertaining his friends. Whatever amount left with him, he deposited half in his savings account and gave Rs. 5 to a poor person as charity. Finally, Keerthi was left with Rs. 20 which he returned to his father. What amount did his father give him initially? 

A. Rs. 160

B. Rs. 120

C. Rs. 200

D. Rs. 100

 

Q. 103 Taps ‘A’ and ‘B’ can fill a tank in 37 ½ minutes and 45 minutes respectively. Both taps are opened and after some time tap ‘B’ is turned off.The tank is filled completely in exactly 30 minutes, if tap ‘B’ is turned off after:

A. 15 minutes

B. 12 minutes

C. 9 minutes

D. 10 minutes

 

Q. 104 A man rows to a place 45 k.ms distant and back in 12 hours. He realises that he can row 5 k.ms downstream in the same time as 3 k.ms against the stream. The velocity of the stream is:

A. 4 k.ms/hr

B. 1 k.m/hr

C. 2 k.ms/hr

D. 1.5 k.ms/hr

 

Q. 105 There are two urns. One contains two white balls and four red balls, the other contains three white and nine red balls. All balls are of the same shape and size. From each urn, one ball is drawn. What is the probability of getting both the balls of the same colour?

A. 1/2

B. 1/12

C. 7/12

D. 1/24

 

Q. 106 A boat travels upstream from A to B and back from B to A in 5 hours.The speed of the boat in still water is 8 km/hour and the speed of the current is 4 km/hour. Then, the distance from A to B is:

A. 9 kms.

B. 10 kms.

C. 12 kms

D. 15 kms

 

Q. 107 Two men and seven boys can do a work in 14 days. Three men and eight boys can do the same work in 11 days. Further eight men and six boys can do three times the amount of this work in:

A. 18 days

B. 30 days

C. 24 days

D. 21 days

 

Q. 108 A piece of cloth costs rupees 75. If the piece is four meters longer and each meter costs rupees 5 less, the cost remains unchanged. What is the length of the piece?

A. 12 meters

B. 8 meters

C. 10 meters

D. 6 meters

 

Q. 109 A can do a piece of work in 8 days and B alone can do the same work in 10 days. A and B agreed to do the work together for Rs. 720. With the help of C, they finished the work in 4 days. How much C is to be paid?

A. Rs. 72

B. Rs. 82

C. Rs. 70

D. Rs. 80

 

Q. 110 Gold and copper are as heavy as water by 19 and 9 times respectively. The ratio in which these two metals be mixed so that the mixture is 17 times as heavy as water is:

A. 2:3

B. 3:4

C. 3:2

D. 4:1

 

Q. 111 Questions numbers 111-145 consists of legal proposition(s)/ principle(s) (hereinafter referred to as ‘principle’) and facts. Such principles may or may not be true in the real and legal sense, yet you have to conclusively assume them to be true for the purposes of this Section. In other words, in answering these questions, you must not rely on any principle except the principles those are given herein below for every question. Further, you must not assume any facts other than those stated in the question. The objective of this section is to test your interest towards study of law, research aptitude and problem solving ability, even if the ‘most reasonable conclusion’ arrived at may be absurd or unacceptable for any other reason. It is not the objective of this section to test your knowledge of law.Therefore, to answer a question, principle is to be applied to the given facts and to choose the most appropriate option.

Principle: Acceptance of a proposal must be absolute and unqualified.

Facts: ‘A’ made a proposal to sell his motorcycle to ‘B’ for rupees 25,000/-. ‘B’ agreed to buy it for rupees 24,000/-. ‘A’ sold his motorcycle to ‘C’ for 26,000/- the next day. ‘B’ sues ‘A’ for damages

A. ‘B’ can proceed against ‘C’

B. ‘B’ will get the difference of rupees 1,000/- only

C. ‘B’ will not get any damages from ‘A’

D. ‘B’ will get damages from ‘A’

 

Q. 112 Principle: When a person consented to an act to be done by another, he cannot claim any damages resulting from doing that act, provided the act done is the same for which consent is given.

Facts: ‘P’ submitted a written consent to a surgeon ‘S’ for undergoing a surgical operation

for removal of appendicitis. The surgeon while doing surgery also removed the gall bladder of ‘A’:

A. ‘P’ can claim damages from ‘S’

B. ‘P’ is not bound to pay expenses of the surgery

C. ‘P’ is required to pay expenses for surgery for Appendicitis but not for Gall Bladder

D. ‘P’ cannot claim damages from ‘S’

 

Q. 113 Principle: Penal laws provide that whoever voluntarily has carnal intercourse against the order of nature with any man or woman, shall be punished for rape.

Facts: A Police Officer found a man engaged in carnal intercourse with an animal. The Police Officer arrested the man and produced him before the Court.

A. Court will punish the man for rape

B. Court will punish the police officer.

C. Court will not punish the man for rape.

D. Court will not punish the police officer.

 

Q. 114 Principle: A person is said to do a thing fraudulently, if he does that thing with intent to defraud, but not otherwise.

Facts: ‘A’ occasionally hands over his ATM card to ‘B’ to withdraw money for ‘A’. On one occasion ‘B’ without the knowledge of ‘A’, uses’A’s ATM card to find out the balance in ‘A’s account, but does not withdraw any money.

A. ‘B’ has committed misappropriation

B. ‘B’ has committed the act fraudulently

C. ‘B’ has committed breach of faith

D. ‘B’ has not committed the act fraudulently

 

Q. 115 Principle: An offer made by one party when accepted by another makes it a contract. 

Transactions:

1 P offered to sell his house for Rs. 20 lakhs to R; R told P that he was interested to buy a house for 15 lakhs only.

2 C was looking for a house for not more than 25 lakhs; P informed Cthat his house was

available for 20 lakhs.

3 K wanted to buy some old furniture; L told K that he would sell his furniture for Rs. 10,000.

4 R advertised to sell his old car for a price of Rs. Three lakhs; S found the advertisement and offered to buy it for Rs. 2 lakhs 50 thousand;R agrees to sell it to S.

Which among the above is actually a contract?

A. Situation 4 only is a contract

B. Situation 3 only is a contract

C. Situations 1 and 2 are contracts

D. Situations 2 and 4 are contracts

 

Q. 116 Principle: There are legal provisions to give authority to a person to use necessary force against an assailant or wrong-doer for the purpose of protecting one’s own body and property as also another’s body and property when immediate aid from the state machinery is not readily available; and in so doing he is not answerable in law for his deeds. 

Facts: X, a rich man was taking his morning walk. Due to the threat of robbers in the locality, he was carrying his pistol also. From the opposite direction, another person was

coming with a ferocious looking dog. All of a sudden, the dog which was on a chain held by the owner,started barking at X. The owner of the dog called the dog to be calm.They crossed each other without any problem. But suddenly, the dog started barking again from a distance. X immediately took out his pistol. By seeing the pistol the dog stopped barking and started walking with the owner. However, X shot at the dog which died instantly. The owner of the dog files a complaint against X, which in due course reached the Magistrate Court. X pleads the right of private defence. Decide

A. Shooting a fierce dog is not to be brought under the criminal law. So the case should be dismissed. 

B. As there was no guarantee that the dog would not bark again, sh ooting it was a precautionary measure and hence within the right available to X under law.

C. There was no imminent danger to X as the dog stopped barking a nd was walking with the owner. Hence, shooting it amounted to excessive use of the right of private defence and hence liable for killing the dog.

D. The right of private defence is available to persons against assail ants or wrong-doers only and a dog does not fall in this category.

 

Q. 117 Principle: In criminal law, misappropriation is the intentional, illegal use of the property or funds of another person for one’s own use or other unauthorized purpose, particularly by a public official, a trustee of a trust, an executor or administrator of a dead person’s estate or by any person with a responsibility to care for and protect another’s assets. Embezzlement is misappropriation when the funds involved have been lawfully entrusted to the embezzler. On the contrary, theft is the illegal taking of another person’s property or services without that person’s permission or consent with the intent to deprive the rightful owner of it.

Facts: A went for swimming at the Municipal Swimming Pool. A handed over all his valuables, including some cash to X, the guard on duty for safe custody, as notified by the Municipality. After swimming for an hour, A came out and searched for X. He found another guard on duty and that guard informed A that X had gone home after completing his shift and did not hand over anything to be given to A. A register ed a complaint with the police. X was traced but he told the police that he sold all the valuables and the entire cash was used for drinking liquor. What offence, if any, was/were committed by X?

A. If at all X is liable, it is for criminal misappropriation only.

B. X is liable for theft as he took A’s property without X’s permission.

C. X is liable for criminal misappropriation and embezzlement.

D. X is not guilty of criminal misappropriation as he did not make an y personal gain out of those items with him.

 

Q. 118 Principle: According to the law of trade unions in India, no suit or other legal proceeding shall be maintainable in any civil court against any registered trade union or any officer or member thereof in respect of any act done in contemplation or in furtherance of a trade dispute.

Facts: Soloman, the Secretary of a registered Trade Union took a loan from a Bank for the higher education of his daughter. Soon after completing the course she was married to an NRI Engineer. Solomon did not repay the loan. The Bank demanded the payments from Soloman and warned him that the Bank will take suitable legal action against him. Identify the legal position in this regard.

A. As Soloman did not use the loan amount for his use and hence, n o action can be initiated against him.

B. The Bank cannot initiate any action against Soloman as he is the Secretary of a Registered Trade Union.

C. The Bank can recover the loan amount from the Trade Union as Soloman is the Secretary of the Union.

D. The Bank can file a suit for recovery of the loan amount against S oloman as he took the loan for a personal purpose and in such case no im munity will work.

 

Q. 119 Principle: When a person falsifies something with the intent to deceive another person or entity is forgery and is a criminal act. Changing or adding the signature on a document, deleting it, using or possessing the false writing is also considered forgery. In the case of writing/painting to fall under the definition, the material included must have been fabricated or altered significantly in order to represent something it is actually not.

Facts: David made a living traveling from city to city, selling paintings that he claimed were done by great artists. Since the artists’ signatures were in place, many people fell for them and purchased the paintings. One of these artists saw three of his alleged paintings in a City gallery containing his name. He knew these were not his works and he complained to the police. Police traced David and initiated legal proceedings. Is David guilty of any offence?

A. David is guilty of forgery as the addition of the signature was with an intention to make people believe that those were the paintings of the gr eat artists.

B. David is not guilty of any offence as he was selling the art pieces for his living.

C. There is no point in taking legal action against David as the signa ture has not done any alteration to the art work.

D. Those who buy the art pieces from David ought to have been car eful in checking it and ensuring that they were originals before purchasing it.

 

Q. 120 Principle: It is a case of fraud where a party to a contract knows or believes a fact to be true, but conceals it actively from the other party with a view to induce that person to enter into the contract.

Facts: While taking a life insurance policy, in reply to questions by the insurance company during the inquiry into his proposal, Zameer deliberately concealed the fact of his medical treatment for a serious ailment, which he had undergone only a few weeks ago

A. The concealment of fact by Zameer amounted to fraud.

B. The act of Zameer did not amount to fraud, as disclosing the fact would have resulted in exposure of his privacy.

C. The act of Zameer did not amount to any misrepresentation.

D. The act of Zameer amounted to innocent misrepresentation.

 

Q. 121 Principle: Where one of the parties to a contract was in a position to dominate the decision of the other party, the contract is enforceable only at the option of the party who was in a position to dominate the decision of the other party.

Facts: A doctor asked his patient to make a payment of rupees Ten Lakh for treatment of his fever. The patient paid an amount of rupees Five Lakh and promised to pay the remaining amount after the treatment. After treatment the patient recovered from fever.

The doctor demanded the remaining amount from the patient. The patient refused to pay.

A. he contract is not enforceable as doctor was in dominating positi on.

B. The contract is enforceable against the patient by the doctor.

C. The contract is enforceable against the doctor.

D. The contract is not enforceable without the consent of the patient.

 

Q. 122 Principle: Ownership in property consists of right to possess, right to use, right to alienate and right to exclude others. Sale is complete when property gets transferred from the seller to the buyer on sale.

Facts: ‘A’ sold his car to ‘B’. After this, ‘B’ requested ‘A’ to keep the car in his care on behalf ‘B’ for one month. ‘A’ agreed.

A. Sale of car is complete.

B. Sale of car is complete.Sale will be automatically completed after the expiry of one Month 

C. Sale of car is not complete

D. Sale will be completed when ‘B’ will take the delivery of the car.

 

Q. 123 Principle: Every agreement, by which any party is restricted absolutely from enforcing his right in respect of any contract, by the usual legal proceedings in the ordinary Tribunals, is void to that extent. The law also provides that nobody can confer jurisdiction to a civil court by an agreement between parties.

Facts: A and B entered into a valid contract for rendering certain service. A clause in the contract was that in case of any dispute arose out of the contract; it shall be referred to for Arbitration only. Is the contract valid?

A. The parties were trying to confer jurisdiction to some authority to decide a dispute and hence the clause would be invalid.

B. Arbitration is also a valid dispute settlement machinery recognize d by law and hence the entire contract is valid.

C. The contract is valid but the clause regarding Arbitration is void.

D. Arbitrator cannot be termed as an ordinary Tribunal. Hence, the a greement is void and would be unenforceable.

 

Q. 124 Principle: According to Sec. 2 of the Industrial Disputes Act, 1947, ‘Industrial dispute means any dispute or difference between employers and employers or between employers and workmen or between work men and workmen, which is connected with the employment or non-employment or the terms of employment or with the conditions of labour of any person’.

Facts: The employees of DK Enterprises met the management and requested half a day leave to allow them to celebrate a lunar eclipse, which was going to happen two days later. The management refused the request. Does this situation amount to an ‘industrial dispute’?

A. As the difference of opinion between the employees and employe r is on declaration of holiday it amounts to an issue connected with employ ment or with the terms of employment and hence, an industrial dispute.

B. No as Lunar eclipse is unconnected with employment.

C. Yes, because there is some difference of opinion it would be an i ndustrial dispute.

D. No as declaring holidays is a prerogative of the employer. So no i ndustrial dispute.

 

Q. 125 Principle: According to law, a person who find goods belonging to another and takes them into his custody, is subject to the same responsibility as a bailee. Bailee is a person or party to whom goods are delivered for a purpose, such as custody or repair, without transfer of ownership. The finder of the goods legally can sell the goods found by him under certain circumstances including the situation that the owner refuses to pay the lawful charges of the finder.

Facts: P, a college student, while coming out of a Cricket stadium found a necklace, studded with apparently precious diamonds. P kept itfor two days thinking that the owner would notify it in a local newspaper. Since he did not notice any such notification, P published a small classified advertisement in a local newspaper. In two days’ time, P was contacted by a film actor claiming that it was her Necklace and requested P to return it to her. P told her that she should compensatehim for the advertisement charges then only he would return it otherwise he will sell it and make good his expenses. The film star told P that she had advertised in a national newspaper about her lost Necklace which was lost somewhere in the Cricket Stadium. The advertisement was published for three consecutive days incurring a large expenditure for her. Mentioning all this she refuses to pay P and claims the Necklace back. Which among the following is the most appropriate answer to this? 

A. The film star was right in refusing P, as she did not offer any reward for anyone who would return the Necklace.

B. As it was wrong on the part of P to bargain over a property belon ging to a celebrity and he should have accepted some gift which might hav e been given by the film star and returned the Necklace instead of threatening her that he would sell it.

C. As the film star had notified in the newspaper, P ought to have re ad it and contacted her instead of publishing another notification. So he cannot claim any compensation.

D. P was requesting the film star for the actual expenditure incurred by him before returning the Necklace. This request is legally sustainable.

 

Q. 126 Principle: The concept of natural justice is against bias and for the right to a fair hearing. While the term natural justice is often retained as a general concept, and it has largely been replaced and extended by the general ‘duty to act fairly’.

Fact: ‘X’, a male employee of a company was dismissed by the employer just on the basis of a complaint by ‘Y’, a female employee of the company that ‘X’ was trying to be too friendly with her and often requested her to accompany him to the canteen.Is the dismissal of ‘X’ valid?

A. No, because in the modern times this type of behaviour is common.

B. Yes, because men are not supposed to behave improperly with women and hence there is no violation of any principles of law

C. Yes, moral law is antique and therefore, not applicable in modern times, therefore the termination is valid and no violations of the principles of natural justice occurred

D. No, because the employer did not give a chance to ‘X’ to explain his side, thereby violated the principles of natural justice.

 

Q. 127 Principle: The Constitution of India guarantees certain fundamental rights to its citizens. The Constitution also provides that these rights cannot be taken away by state even by a law. For violation of this, the person adversely affected by the law may approach the High Court or the Supreme Court for the issuance of an appropriate writ. One of these rights includes the freedom to form association that implies the right to join an association or not to join such an association.

Facts: Owing to some industrial disturbances created by XATU, oneof the several trade unions in AB Chemicals (Pvt) Ltd., the Company issued a circular to all its employees that as far as possible the employees may disassociate with XATU. Navin is an employee of AB Chemicals and the current General Secretary of XATU. Aggrieved by this circular, which affected the fundamental rights of his and other members of the Union, approaches the High Court of the state for a relief. Identify the most reasonable legal proposition. 

A. The Company’s circular is illegal and has to be quashed by the Court.

B. Circular issued by a Company amounts to law in the constitutional sense and hence the High Court can issue a writ as pleaded for by Navin

C. The prohibition against any imposition of restriction against a fundamental right is not applicable to anybody other than the state and hence Navin will not get any relief from the High Court

D. The circular interferes with the freedom guaranteed by the Constitution and hence the High Court can issue an appropriate right.

 

Q. 128 Principle: Assault is causing bodily injury to another person by use of physical force. 

Facts: Rustum while entering into compartment of a train raised his fist in anger towards a person Sheetal, just in front of him in the row, to get way to enter into the train first, but did not hit him. Rustum has:

A. Rightly showed his anger

B. committed an assault on Sheetal

C. insulted Sheetal

D. not committed an assault on Sheetal

 

Q. 129 Principle: Every agreement, of which the object or consideration is opposed to public policy, is void. An agreement which has the tendency to injure public interest or public welfare is one against public policy. What constitutes an injury to public interest or public welfare would depend upon the times and the circumstances.

Facts: ‘A’ promises to obtain for ‘B’ an employment in the public service, and ‘B’ promises to pay rupees 5,00,000/- to ‘A’.

A. The agreement is void because rupees 5,00,000/- is excessive.

B. The agreement is valid, as it is with consideration for public service.

C. The agreement is valid, as it is a contract between two parties with their free consent.

D. The agreement is void, as the object and consideration for it is opposed to public policy.

Q. 130 Principle: According to law, a person is deemed to have attained the age of majority when he completes the age of 18 years, except in the case of a person where a guardian of a minor’s person or property has been appointed under the Guardians and Wards Act, 1890 or where the superintendence of a minor’s property is assumed by a Court of Wards. Indian law expressly forbids a minor from entering into a contract. Hence, any contract entered into by a minor is void-ab-initio regardless of whether the other party was aware of his minority or not. Further, though a minor is not competent to contract, nothing in the Contract Act prevents him from making the other party bound to the minor. 

Facts: Lal executed a promissory note in favour of Gurudutt, aged 16years stating that he would pay Gurudutt a sum of Rs. 2 Lakhs when he attains the age of majority. On attaining the age of 18, Gurudutt demanded the amount from Lal, who refused to pay. Gurudutt wants to take legal action against Lal. Identify the most appropriate legal position from the following:

A. Lal was not aware of the fact that Gurudutt was a minor.

B. Gurudutt should not have entered into a contract with Lal when he was a minor

C. A promissory note duly executed in favour of a minor is not void and can be sued upon by him, because he though incompetent to contract,may yet accept a benefit

D. Lal argues that as per the Guardians and Wards Act, 1890, Gurudutt can claim the money only after he attains the age of 21.

 

Q. 131 Principle: When a person interferes with peaceful possession of another person without the permission of the person in possession of those premises, commits trespass to land.

Facts: ‘T’ just walked over the land of ‘P’ to reach his house as it was a short cut. ‘P’ had displayed a notice that it is not a thoroughfare. ‘P’ did not cause any damage to the land.

A. T’ has committed trespass to land.

B. ‘T’ has created nuisance for ‘P’.

C. ‘T’ has not committed any trespass on the land of ‘P’.

D. ‘T’ has violated privacy of ‘P’.

 

Q. 132 Principle: When a person falsifies something with the intent to deceive another person or entity is forgery and is a criminal act. Changing or adding the signature on a document, deleting it, using or possessing the false writing is also considered forgery. In the case of writing to fall under the definition, the material included must have been fabricated or altered significantly in order to represent something it is actually not.

Facts: John was a publisher of ancient books and papers. In one of his books on the World Wars, he gave photograph of some letters written by famous historic personalities. A researcher in history noted that in the pictures of some of the letters printed in the book, John had added some words or sentences in his own handwriting to give completeness to the sentences, so that the readers will get a clear picture of the writer’s intention. The researcher challenges the originality of those pictures and claims that the book containing the forged letters should be banned. Examine the validity of the researcher’s demand. 

A. The additions in the letters were made by the publisher in his own handwriting would have made material alteration to the original meaning and hence amounted to forgery.

B. As forgery amounts to adding or deleting anything from an original document, the demand of the researcher is valid.

C. Allowing forged publications to be circulated among the public is as good as committing fraud on the public, so the publication should be banned.

D. The additions were made to give clarity to the original document and did not in any sense change the contents of the documents and hence there is no forgery as alleged by the researcher.

 

Q. 133 Principle: Negligence is actionable in law. In simple terms, negligence is the failure to take proper care over something.

Facts: A, a doctor, conducted a hysterectomy sincerely on B and left a small cotton swab inside the abdomen. As a consequence of which B developed some medical problems and had to undergo another surgery. Is A liable?

A. A is not liable as he did not foresee any consequences at the time of surgery.

B. Liability for negligence does not arise here as A performed the operation sincerely

C. A is liable for the negligence as he failed to take proper care during the surgery.

D. As only a small swab was left in the abdomen, there was no negligence.

 

Q. 134 Principle: When a person makes such a statement which lowers other person’s reputation in the estimation of other persons, is liable for committing defamation.

Facts: ‘A’ writes a letter to ‘B’ in which he uses abusive language against ‘B’ and also states that ‘B’ is a dishonest person. ‘A’ put the letter in a sealed envelope and delivered it to ‘B’.

A. A’ has committed a moral wrong

B. ‘A’ has not committed defamation

C. A’ has not committed moral wrong

D. ‘A’ has committed defamation

 

Q. 135 Principle: A master shall be liable for the fraudulent acts of his servants committed in the course of employment. However, the master and third parties must exercise reasonable care in this regard.

Facts: Rahul was a door to door salesman with United Manufacturing Company (the Company). The Company was manufacturing Water Purifiers. Rahul, along with the Company’s products, used to carry Water Purifiers manufactured by his Cousin in a local Industrial Estate. He used to sell the local product at a lower rate giving the impression to the buyers that he is offering a discount on the Company’s product. The Company Management detected the fraudulent activity of Rahul and dismissed him from service. Rahul still continued to carry on with his activity of selling the local product pretending that he was still a salesman of the Company. Several customers got cheated in this process. The fraud was noticed by the Company when the customers began to complain about the product. The customers demanded the Company to compensate their loss.

A. The Company is liable to compensate all the customers as it did not inform the public about Rahul’s fraudulent conduct and the subsequent dismissal.

B. The Company is not liable as Rahul was dismissed by the Company.

C. The Company is liable to the customers who purchased the local product from Rahul only till he remained as a salesman of the Company.

D. The liability rests with the local manufacturer as it was a defective product.

 

Q. 136 Principle: Contract is a written or spoken agreement, with specific terms between two or more persons or entities in which there is a promise to do something in return for a valuable benefit known as consideration. Such an agreement is intended to be enforceable by law. A unilateral contract is one in which there is a promise to pay or give other consideration in return for actual performance.

Facts: A Toilet Soap Manufacturing Company in India in order to promote the sale of their product, published an advertisement in all the Newspapers on January 1, 2017 that the Company has kept a model ignition key of an Audi A3 Car. The advertisement also stated that whoever gets the said key before December 31, 2017 from a soap bar will be gifted with the Audi A3 Car. Mr. Martin, a foreigner who came to India as a Tourist who was staying in a Hotel found a Key similar to same Car Ignition Key. Mr. Martin brought this matter to the notice of the Hotel Manager. The Manager informed Mr. Martin about the Company’s advertisement on January 1, 2017. Mr. Martin wants to claim the Car. Will he succeed?

A. The Hotel Manager who could legally claim the Car as he was the one actually purchased the soap for the use in the Hotel.

B. No. The Soap Company has not entered into a contract with Mr.Martin as he was not in India on January 1, 2017 when the advertisement was published.

C. No. Actual intention of the Company was to promote the sale of the Soap.

D. Mr. Martin obtained the Key before the stipulated date from the Soap Bar. So he is covered by the offer of the Soap Company and can claim the car.

 

Q. 137 Principle: Under the Employees Compensation Act, 1923, an employer is liable to pay compensation to his workmen for injuries sustained by them by an accident arising out of and in the course of employment.

Facts: M, the Manager of SRK Industries asked his secretary S to submit a report at the Government Labour Office. ‘S’ submitted the report as directed. On his way back S met one of his class mates. He then decided to have a cup of tea together on a way side restaurant. Sometime later, ‘S’ got a message from his office to report back as it was long time since he left the office. ‘S’ rushed back on his Motor Cycle. On his way back a Truck which was coming from a side road hit ‘S’. He was admitted in a nearby hospital with multiple injuries. He claims compensation under the Employees Compensation Act from his employer.

A. The Employer is liable as S had to rush back to the office, because of the message from the office.

B. The Employer is not liable as the truck driver was negligent.

C. The Employer is not liable as he was admitted in a private hospital and not a Government Hospital.

D. The Employer is liable to pay compensation as the accident took place arising out of and in the course of employment.

 

Q. 138 Principle: An agreement, the terms of which are not certain, or capable of being made certain, is void.

Facts: Sunder agreed to take Bhola’s penthouse on rent for three years at the rate of rupees 12, 00, 000/- per annum provided the house was put to thorough repairs and the living rooms were decorated according to contemporary style.

A. There is no valid contract because it has vague and uncertain terms, as the term ‘present style’ may mean one thing to Sunder and another to Bhola.

B. There is a valid contract because there is an offer from Sunder and acceptance from Bhola.

C. It is voidable contract at the option of Bhola.

D. There is a valid contract because all the terms of contract are certain and not vague as the rent is fixed by both of them and the term ‘present style’ only can be interpreted to mean the latest style.

 

Q. 139 Principle: Whoever takes away with him any minor less than sixteen years of age if a male, or less than eighteen years of age if a female,out of the custody of parents of such minor without the consent of such parents, is said to commit no offence.

Facts: ‘A’, a man, took away a girl below sixteen years to Mumbai without informing the parents of the girl.

A. ‘A’ committed an offence against the girl as well as her parents

B. ‘A’ committed an offence against the girl.

C. ‘A’ committed no offence against the parents of the girl

D. ‘A’ committed no offence against the girl as well as her parents

 

Q. 140 Principle: Section 34 of Indian Penal Code provides that ‘When a criminal act is done by several persons in furtherance of the common intention of all, each of such persons is liable for that act in the same manner as if it were done by him alone.’

Facts: Three vagabonds, Sanju, Dilbag and Sushil decided to commit burglary. In the night, Sushil opened the lock and they broke into a rich man’s house when the entire family was on a pilgrimage. Sanju had gone to that house earlier in connection with some cleaning job. There was only a servant lady in the house. Hearing some sounds from the master bed room, the servant switched on the lights and went upto the room from where she heard the sound. Noticing that the servant was going to cry for help, Sanju grabbed her and covered her mouth with his hands and dragged her into the nearby room. The other two were collecting whatever they could from the room. When they were ready to go out of the house, they looked for Sanju and found him committing rape on the servant. They all left the house and the servantreported the matter to the police and identified Sanju. Subsequently,all three were arrested in connection with the offences of house breaking, burglary and rape. Identify the legal liability of the three.

A. All three are liable for all the offences as there was common intention to commit the crimes.

B. Sanju will be liable only for housebreaking and rape as he did not participate in the burglary.

C. Only Dilbag and Sushil are liable for burglary in looting the house, and all three will be liable for housebreaking and rape as they did not stop Sanju from committing the offence and hence were accomplice to the offence.

D. Only Sanju will be liable for rape as he was the one who actually committed the offence.

 

Q. 141 Principle: When a person who has made a promise to another person to do something does not fulfill his promise, the other person becomes entitled to receive, from the person who did not fulfill his promise,compensation in the form of money.

Facts: ‘X’ made a promise to ‘Y’ to repair his car engine. ‘Y’ made the payment for repair. After the repair, ‘Y’ went for a drive in the same car. While driving the car, ‘Y’ met with an accident due to bursting of atyre.

A. ‘X’ will be entitled to receive compensation from ‘Y’ in the form of money.

B. ‘X’ will not be entitled to receive compensation.

C. ‘Y’ will be entitled to receive compensation from ‘X’ in the form of money.

D. ‘Y’ will not be entitled to receive compensation from ‘X’.

 

Q. 142 Principle: If a party to a contract agrees to it under undue influence of any other party then the party under the undue influence may refuse to perform in accordance with the agreement.

Facts: A, a rich youngster became a member of a religious group and soon he was appointed by P the head of the group as his personal secretary. As per the rules of the group, all officials and staff of the group were supposed to stay in the group’s official premises itself. Some days later, A was asked by P to execute a Gift deed in favour of P, in which it was mentioned that all immovable properties in his name are being gifted to P. A was unwilling to execute the deed, but he was forcefully restrained by P and his body guards in P’s office and made A sign the gift deed. Soon after this A left the group and refused to hand over the property as agreed to in the gift deed. Is A’s action valid?

A. It is illegal for religious groups acquire property from its members.

B. A executed the deed, under compulsion and undue influence, and was right in withdrawing from the contract.

C. As the gift deed was executed by A, he cannot refuse.

D. As Gift is also a contract, the consent of A was not obtained by P while executing the deed.

 

Q. 143 Principle: A contract would be invalid and unlawful, if the contract is for an immoral or illegal purpose.

Facts: P, was a young and helpless widow, living on the pavement. R,a neighbour gave her a house, registered in her name, on the condition that she should allow R to keep his smuggled goods and drugs in her house. After the registration was done, according to the condition in the contract, R’s agents went to keep some packets in her house, she refused. R told her the condition under which the house was given to her. She still refused. Is P justified in her action?

A. As R was making the contract for illegal activities, P’s stand is valid in law.

B. R can take back the house by cancelling the transfer deed.

C. P is not justified as she did not have the right to deny R’s request.

D. P is right as she did not like smuggled goods to be kept in her house.

 

Q. 144 Principle: Nothing is an offence which is done in the exercise of the right of private defence. Facts: ‘A’, under the influence of madness, attempts to kill ‘B’. ‘B’ to save his life kills ‘A’.

A. ‘A’ has not committed an offence because he was mad.

B. ‘B’ has committed an offence.

C. ‘B’ has not committed any offence.

D. ‘A’ has committed the offence of attempt to murder.

 

Q. 145 Principle: A violation of a legal right of someone, whether results in a legal injury or not, gives rise to an action in tort for compensation. At the same time, an action by someone, which results in some loss or damage to somebody else is not actionable, if there is no violation of a right of that somebody.

Facts: AB Coaching Centre was a popular CLAT coaching academy with several good trainers. A lot of aspirants used to attend its coaching classes from all over and was making good profit. This was going on for the past several years. During a session, T, one of the very good and popular trainers of ABCC, had some difference of opinion with the owner of ABCC and left the coaching centre. In August 2016, T started another Entrance Coaching Centre closer to ABCC which resulted in a substantial drop in its students and huge financial loss. The owner of ABCC wants to file a case against T for the loss sustained by ABCC. What do you think is the right legal position?

A. T will be liable to compensate the loss to ABCC.

B. T has not violated any of ABCC’s legal right though they sustained some financial loss, and not legally bound to compensate ABCC.

C. ‘T’ should have consulted ABCC before starting his coaching centre.

D. T started the new coaching centre near ABCC intentionally, and s hall be liable to compensate the loss of ABCC.

 

Q. 146 Bona vacantia

A. Order of the court for eviction

B. Vacant land

C. Goods that have no owner

D. Vacant building

 

Q. 147 In pari delicto

A. Where the lawyer is at fault

B. Where the judge is at fault

C. Where the petitioner is at fault

D. Where both parties to a dispute are equally at fault

 

Q. 148 Sine die’ means:

A. Adjourned without fixing any date for the next meeting

B. Adjourned for the day and scheduled to meet next day again.

C. Adjourned for the day and meet after one week

D. Adjourned for the day and meet after one month.

 

Q. 149 Malus animus

A. Good intention

B. Bad intention

C. Physical force

D. Animal farm

 

Q. 150 Lis pendens

A. Pending suit

B. Decided case

C. No legal issues involved

D. Facts of case proved

 

Q. 151 Per incuriam

A. Mistaken decision

B. Supremacy of law

C. Mistaken identity

D. Supremacy of the Constitution

 

Q. 152 Autrefois convict means

A. Formerly convicted

B. To be convicted

C. Failed prosecution

D. Doubtful conviction

 

Q. 153 Lex loci means

A. Law of a place

B. Latin regualtions

C. Domestic laws

D. Italian laws

 

Q. 154 Animus posssidendi’ means:

A. Intent to contract

B. Intention to harm

C. Intention to return

D. Intention to possess

 

Q. 155 Caveat venditor means

A. Seller beware

B. Manufacturer beware

C. Transporter beware

D. Buyer beware

 

Q. 156 Pari passu means

A. On an unequal status

B. On equal footing

C. Diverse nature

D. Supremacy of law

 

Q. 157 Faux pas means

A. Cheating

B. Tactless mistake

C. Pausing for a while

D. Passage of time

 

Q. 158 ‘Punctum Temporis’ means

A. Functional authority

B. Temporary position

C. Timely assistance

D. Point of time

 

Q. 159 ‘Jus Gentium’ means:

A. Global administrative law

B. Law of Societies

C. Law among Nations

D. Global justice

 

Q. 160 Turpis arbiter’ means:

A. Inefficient lawyer

B. Inefficient judge

C. Corrupt prosecutor

D. Corrupt judge

 

Questions: 161 – 163

Direction for Question Numbers 161 – 163 Read the following information carefully and choose the appropriate option in the questions given below.

i. There is a group of five persons – A, B, C, D and E.

ii. One of them is a Singer, one is a Dancer, one is a Painter, one is a Teacher and one is a Doctor.

iii. Three of them – A, C and Doctor prefer rice to chapatti and two of them – B and the Painter prefer chapatti to rice.

iv. The Teacher, D and A are friends to one another but two of these prefer chapatti to rice.

v. The Singer is C’s brother.

 

Q. 161 Who is a Singer?

A. C

B. D

C. A

D. B

 

Q. 162 Who is a Teacher?

A. D

B. C

C. E

D. B

 

Q. 163 Who is a Dancer?

A. E

B. C

C. A

D. D

 

Questions: 164 – 166

Direction for Question Numbers 164 – 166: Read the following information carefully and choose the most appropriate option in the questions given below

i. Six flats on a floor in two rows facing North and South are allotted to P, Q, R, S, T and U.

ii. Q gets a North-facing flat and is not next to S.

iii. S and U get diagonally opposite flats.

iv. R, next to U gets a South-facing flat and T gets a North-facing flat.

 

Q. 164 Which of the following combination gets South-facing flats?

A. U, P ,T

B. Q, T, S

C. data inadequate

D. U, R, P

 

Q. 165 Whose flat is between Q and S?

A. U

B. R

C. P

D. T

 

Q. 166 If the flats of T and P are interchanged, who’s flat will be next to that of U?

A. T

B. P

C. R

D. Q

 

Questions: 167 – 168

Which alternative applies to the following Statement or Assumptions? Choose the

most appropriate option.

 

Q. 167 ‘There is no man that is not naturally good’ is equivalent to the proposition:

A. No men are good.

B. Some men are naturally good.

C. Some men are not naturally good.

D. All men are naturally good.

 

Q. 168 ‘Only ignorant people believe in witchcraft’ is equivalent to:

A. Some ignorant persons are not those who believe in witchcraft.

B. No ignorant persons are those who do not believe in witchcraft.

C. All persons who believe in witchcraft are ignorant.

D. There is no link between ignorance and witchcraft.

 

Q. 169 Find the odd one out from the following:

A. Clearly visible

B. Exact estimate

C. Open secret

D. Only choice

 

Q. 170 Find the odd one out from the following:

A. Expedition

B. Crusade

C. Campaign

D. Cruise

 

Q. 171 Crumb : Bread is as

A. Splinter : Wood

B. Inch : Unit

C. Powder : Face

D. Water : Vessel

 

Q. 172 wave: crest as _________ : peak.

A. Water

B. land

C. River

D. Mountain

 

Q. 173 If in a certain code, the word MILITARY is written as 12324567, then in the same code, the word TAIL will be written as:

A. 2345

B. 3254

C. 4523

D. 5432

 

Q. 174 If 27th March, 2011 was Sunday, what was the day on 27th June, 2011?

A. Tuesday

B. Saturday

C. Monday

D. Sunday

 

Q. 175 ————– is a hater of knowledge and learning.

A. Misologist

B. Moroccan

C. Misogynist

D. Mystique

 

Q. 176 ‘Some of the valuable books are seldom read’, means:

A. All the valuable books are not read.

B. All the valuable books are read.

C. Some of the valuable books are read.

D. Some of the valuable books are not read.

 

Q. 177 A girl introduced a boy as the son of the daughter of the father of her uncle. The boy is girl`s:

A. Son

B. Uncle

C. Son-in-law

D. Brother

 

Q. 178 When Ravi saw Ramesh, he recalled, ‘He is the son of the father of my daughter’. Who is Ramesh?

A. Cousin

B. Brother-in-law

C. Uncle

D. Brother

 

Q. 179 Pointing to a girl in the photograph, Ram said, ‘Her mother`s brother is the only son of my mother’s father’. How is the girl`s mother related to Ram?

A. Mother

B. Grandmother

C. Sister

D. Aunt

 

Q. 180 Sunil’s school bus is facing North when it reaches his school. After starting from Sunil’s house, it turned right twice and then left before reaching the school. What direction the bus was facing when it left the bus stop in front of Sunil’s house?

A. North

B. South

C. West

D. East

 

Q. 181 What is meant by ‘Alliteration’?

A. Act of literary modification.

B. The occurrence of the same letter or sound at the beginning of adjacent or closely connected words.

C. Acts of an environmentally conscious person.

D. Words which sound alike but have different meanings.

 

Q. 182 There were twelve dozens of chocolates with a shopkeeper. Ten chocolates were distributed by the shopkeeper to the children of his colony. The shopkeeper then added two more dozens of chocolates in his stock. If the shopkeeper divided the total chocolates equally in two different packets, then how many chocolates were there in each packet?

A. 158

B. 89

C. 152

D. 79

 

Q. 183 Identify the statement which cannot be accepted

A. Almost one third of the human body is made up of water

B. Odyssey is an ancient epic

C. Human race will become extinct sooner or later

D. The earth revolves around the sun in 366 days

 

Q. 184 Ravi was showing a photograph to his friend, Gopi. Pointing at a boy in the photograph, Ravi said: ‘The boy sitting at the left is the son of the wife of the only son of the grandmother of my younger brother’.What is the relation between the boy in the photograph and Ravi?

A. First Cousins

B. Brothers

C. Nephew and uncle

D. Ravi’s brother-in-law

 

Q. 185 How many times from 4 pm to 10 pm, the hands of a clock are at right angles?

A. 10

B. 11

C. 9

D. 6

 

Q. 186 John wants to go the university. He starts from his house which is in the East and comes to a crossing. The road to his left ends in a theatre, straight ahead is the hospital. In which direction is the University?

A. East

B. South

C. North

D. West

 

Q. 187 Coding and decoding 9: 72 : : 8 : ?

A. 34

B. 18

C. 64

D. 43

 

Q. 188 Two ladies and two men are playing bridge and seated at North, East,South and West of a table. No lady is facing East. Persons sitting opposite to each other are not of the same sex. One man is facing South.Which direction are the ladies facing to?

A. North and West

B. South and East

C. East and West

D. None of these.

 

Q. 189 A person who renounces religious or political belief or principle is called:

A. Apostle

B. Antiquarian

C. Ascetic

D. apostate

 

Q. 190 If in a code language, ‘ABANDON’ is written as ‘aramoim’; ‘BORE’ is written as ‘rits’ and ‘BASIL’ is written as ‘rabut’, then what is the original word for the code: ‘bituo’?

A. SOFIA

B. SOLID

C. NASIA

D. SOMAD

 

Q. 191 In a company, 60 % workers are males. If the number of female workers in the company is 800, what is the number of male workers in the company?

A. 1200

B. 1600

C. 1400

D. 1900

 

Q. 192 The birthday of Ms. Y was celebrated six days before Ms. X, who was born on 4th October 1999. The independence day of that year fell on Sunday. On which day did Ms. Y celebrate her birthday, if it was not a leap year?

A. Tuesday

B. Monday

C. Wednesday

D. Sunday

 

Q. 193 In a military secret service map, South-East is shown as North, North-East as West and so on. What will West become?

A. North-East

B. South-West

C. North-West

D. South-East

 

Q. 194 Vaishnavi prefers Economics to Maths, English to Social science, and Political Science to History. If she prefers Maths to History, and Social science to Maths, which is Vaishnavi’s least preferred subject?

A. History

B. Social science

C. Maths

D. Economics

 

Q. 195 A man walks 1 km. towards East and then he turns to South and walks 5 kms. Again he turns to East and walks 2 kms. After this he turns to North and walks 9 kms. Now, how far is he from his starting point?

A. 9 kms

B. 4 kms.

C. 10 kms

D. 5 kms

 

Q. 196 Mare is to Horse as –

A. Deer is to Buck

B. Sow is to Boar

C. Geese is to Duck

D. Pony is to Donkey

 

Q. 197 Pointing to a photograph, Prakash said, ‘She is the daughter of my grandfather’s only son’ How is Prakash related to the girl in the photograph?

A. Uncle

B. Cousin

C. Father

D. Brother

 

Q. 198 In the series of alphabets given below, which is the missing alphabet series?

AX, DU, GR, ____, ML

A. HQ

B. IK

C. JO

D. JN

 

Q. 199 If South-East becomes North, North-East becomes West and so on, what will West become? 

A. North-West

B. North

C. South-East

D. East

 

Q. 200 No parrots are black.

All crows are black.

From the above premises which one of the following conclusions is true?

A. No crows are parrots.

B. Some parrots are not crows

C. No conclusion can be drawn.

D. Some crows are not parrots.

 

Answer Sheet 
Question 1 2 3 4 5 6 7 8 9 10
Answer B B D C C A D C D D
Question 11 12 13 14 15 16 17 18 19 20
Answer A B A A D B A C C C
Question 21 22 23 24 25 26 27 28 29 30
Answer C A A C A A A A D A
Question 31 32 33 34 35 36 37 38 39 40
Answer B C D D D C C D D D
Question 41 42 43 44 45 46 47 48 49 50
Answer C D C A D D D D C A
Question 51 52 53 54 55 56 57 58 59 60
Answer D A A A D A B C C D
Question 61 62 63 64 65 66 67 68 69 70
Answer D D C C A D A C B C
Question 71 72 73 74 75 76 77 78 79 80
Answer B D C D D B B C A B
Question 81 82 83 84 85 86 87 88 89 90
Answer D A A C C A D B D B
Question 91 92 93 94 95 96 97 98 99 100
Answer D B B D A A C D C A
Question  101 102 103 104 105 106 107 108 109 110
Answer C D C C C D D D A D
Question  111 112 113 114 115 116 117 118 119 120
Answer C A C D A C C D A A
Question 121 122 123 124 125 126 127 128 129 130
Answer B A B A D D C D D C
Question 131 132 133 134 135 136 137 138 139 140
Answer A D C B A D D A D D
Question 141 142 143 144 145 146 147 148 149 150
Answer D B A C B C D A B A
Question 151 152 153 154 155 156 157 158 159 160
Answer A A A D A B B D C D
Question 161 162 163 164 165 166 167 168 169 170
Answer C B D D D C D C A D
Question 171 172 173 174 175 176 177 178 179 180
Answer A D C C A D D B D C
Question 181 182 183 184 185 186 187 188 189 190
Answer B D A B B C C A D B
Question 191 192 193 194 195 196 197 198 199 200
Answer A A D A D B D C C A

 

CLAT 2016 Previous Year Paper

CLAT 2016

Q. 1 I like reading journals _______ novels.

A. The Best

B. more than

C. most than

D. the less than

 

Q. 2 There isn’t _________ water in the overhead tank.

A. Lot of

B. any

C. little

D. something

 

Q. 3 They always give the available seats to _______ comes first

A. Whomever

B. whom

C. whichever

D. whoever

 

Q. 4 A fire broke _____ in the neighbourhood.

A. Off

B. out

C. away

D. from

 

Q. 5 If you promise _______ angry with me, I’ll tell you what I broke.

A. not to get

B. get out

C. not getting

D. not get

 

Q. 6 A thief does not ______ the door.

A. Knock at

B. knock on

C. knock to

D. knock

 

Q. 7 There is a lot of work ______ hand. Let’s cancel ______ picnic.

A. Upon,the

B. in,a

C. in,the

D. on,a

 

Q. 8 It’s unfortunate that he died _____ cancer.

A. By

B. of

C. whom

D. with

 

Q. 9 Professor Ahmed ____ teaching us _____ August, 2012.

A. Was,for

B. had been,since

C. has been,since

D. has been,for

 

Q. 10 The method and practice of teaching is called ____.

A. Paediatrics

B. pedagogy

C. training

D. philately

 

Q. 11 Her parents have arrived _____ the airport.

A. To

B. at

C. on

D. in

 

Q. 12 They returned home _____ a taxi.

A. In

B. from

C. on

D. with

 

Q. 13 I have never come _______ nay one as rude as him.

A. Across

B. to

C. around

D. at

 

Q. 14 I can’t bear ____ late.

A. been

B. be

C. to

D. being

 

Q. 15 Mani, along with his friends, _____ for basket ball practice every morning.

A. Were going

B. are going

C. goes

D. go

 

Q. 16 Choose the correct spelling

A. Sacribegous

B. saereligious

C. sacrilegious

D. sacrilegiuos

 

Q. 17 Choose the correct spelling

A. Deceive

B. deceeve

C. diecieve

D. decieve

 

Q. 18 Choose the correct spelling

A. Collaborate

B. colaborate

C. colabborate

D. collaborrate

 

Q. 19 Choose the correct spelling

A. Integrity

B. integerety

C. intigerity

D. integerity

 

Q. 20 Choose the correct spelling

A. Berrister

B. barrister

C. barisster

D. berister

 

Q. 21 “Faux pas” means:

A. Expected to happening

B. flase

C. social blunder

D. fake identity

 

Q. 22 “Ab initio” means:

A. In continuation with

B. from the beginning

C. in defence of a comment

D. in connection with the future

 

Q. 23 “To bury the hatchet” means:

A. To end a feud with an enemy

B. to cremate carcass of an animal

C. to plant grass in the field

D. to hid some treasure

 

Q. 24 “Amicus curiae” means:

A. One of the judges of the court

B. A friend of the court

C. One among the parties to the dispute

D. One among the expert witnesses in a case.

 

Q. 25 Choose the correct option.

A. Please stop interfering into my romantic life.

B. Please stop interfering in my romantic life.

C. Please stop interfering with my romantic life.

D. Please stop interfering for my romantic life.

 

Q. 26 Choose the correct option

A. The Titanic did not reach their destination

B. The Titanic did not reach her destination

C. The Titanic did not reach his destination

D. The Titanic did not reach it’s destination

 

Q. 27 Choose the correct option

A. The Film Star climbed off their car and smiled at the people.

B. The Film Star come out of the car and smiled at the people.

C. The Film Star got out of the car and smiled at the people.

D. The Film Star got off the car and smiled at the people.

 

Q. 28 Choose the correct option.

A. Your grammar is very good, but you need to work on correcting your pronunciation.

B. Your grammar is very good, but you need to work on managing your pronunciation.

C. Your grammar is very good, but you need to work on repairing your pronunciation.

D. Your grammar is very good, but you need to work on modifying your pronunciation.

 

Q. 29 When _____ is bliss, it is _____ to be wise.

A. Romance, boring

B. ignorance, folly

C. knowledge, better

D. bachelorhood, single

 

Q. 30  ______ waters run _____.

A. Hot, fast

B. Muddy, slow

C. Stagnant, leisurely

D. Still, deep

 

Questions: 31 – 40

Direction for Question 31-40: Read the given passage carefully and answer the questions given after the passage:

1. Often, we passionately pursue matters that in the future appear to be contradictory to our real intention or nature; and triumph is followed by remorse or regret. There are numerous examples of such a trend in the annals of history and contemporary life.

2. Alfred Nobel was the son of Immanuel Nobel, an inventor who experimented extensively with explosives. Alfred too carried out research and experiments with a large range of chemicals; he found new methods to blast rocks for the construction of roads and bridges; he was engaged in the development of technology and different weapons; his life revolved around rockets and cannons and gun powder. The ingenuity of the scientist brought him enough wealth to buy the Bofors armament plant in Sweden.

3. Paradoxically, Nobel’s life was a busy one yet he was lonely; and as he grew older, he began suffering from guilt of having invented the dynamite that was being used for destructive purposes.He set aside a huge part of his wealth to institute Nobel Prizes. Besides honouring men and women for their extraordinary achievements in physics, chemistry, medicine and literature, he wished to honour people who worked for the promotion of peace.

4. It’s strange that the very man whose name was closely connected with explosives and inventions that helped in waging wars willed a large part of his earnings for the people who work for the promotion of peace and the benefit of mankind. The Nobel Peace Prize is intended f or a person who has accomplished the best work for fraternity among nations, for abolition or reduction of war and for promotion of peace.

5. Another example that comes to one’s mind is that of Albert Einstein. In 1939, fearing that the Nazis would win the race to build the world’s first atomic bomb, Einstein urged President Franklin D Roosevelt to launch an American programme on nuclear research. The matter was considered and a project called the Manhattan Project was initiated. The project involved intense nuclear research the construction of the world’s first atomic bomb. All this while, Einstein had the impression that the bomb would be used to protect the world from the Nazis. But in 1945, when Hiroshima was bombed to end World War II, Einstein was deeply grieved and he regretted his endorsement of the need for nuclear research. 

6. He also stated that had he known that the Germans would be unsuccessful in making the atomic bomb, he would have probably never recommended making one. In 1947, Einstein began working for the cause of disarmament. But, Einstein’s name still continues to be linked with the bomb.Man’s fluctuating thoughts, changing opinions, varying opportunities keep the mind in a state of flux. Hence, the paradox of life: it’s certain t hat nothing is certain in life.

 

Q. 31 The Manhattan Project was intiated ______.

A. in honour of Einstein.

B. to protect the Nazis.

C. to bomb Hiroshima.

D. to carry out nuclear research

 

Q. 32 Alfred eastablished the Nobel Prizes to ______.

A. remind people of his achievements.

B. ease his guilt and promote work for the betterment of mankind.

C. use his wealth for hard working people.

D. honour only those people who are intelligent

 

Q. 33 In paragraph 4, the word ‘accomplished’ means ___________.

A. completed successfully.

B. worked hard.

C. won awards

D. made an effort to do something

 

Q. 34 In the fifth paragraph, the word ‘endorsement’ means

A. expressing one’s opposition.

B. expressing one’s approval or support.

C. making a promise to do something.

D. expressing one’s regret.

 

Q. 35 Working with arms and ammunition helped Alfred to amass _______.

A. enemies

B. intelligence

C. wealth

D. popularity

 

Q. 36 Immanuel’s interest in dynamites influenced Alfred’s inclination for working ___________.

A. for humanity

B. with explosives

C. for the Nobel Peace Prize

D. with contradiction

 

Q. 37 One of the paradoxes in Alfred’s life was that he was ___________.

A. lonely yet rich

B. hard working but a failure

C. intelligent yet lonely

D. occupied yet lonely

 

Q. 38 Einstein had the impression that the Germans would __________.

A. bomb Hiroshima.

B. be successful in making the world’s first atomic bomb.

C. be unsuccessful in making the atomic bomb.

D. work for humanity

 

Q. 39 The passage is _____________.

A. an argumentative essay

B. an expository essay.

C. a process essay.

D. a descriptive essay

 

Q. 40 The paradox, ‘it’s certain that nothing is certain in life’, indicates the writer’s

A. hatred for scientists

B. analytical mind

C. scientific mind

D. persuasive nature

 

Q. 41 India signed the “Paris Agreement on Climate Change” in April, 2016 at:

A. New York

B. Paris

C. New Delhi

D. Geneva

 

Q. 42 The country which cloned a buffalo calf first is:

A. India

B. U. S. A.

C. Sweden

D. Denmark

 

Q. 43 Mars is also known as the:

A. Blue planet

B. Brown Planet

C. Red Planet

D. Green Planet

 

Q. 44 In 2015, Prime Minister Mr. Narendra Modi launched a new campaign, “Start up India, Stand up India”. The campaign is aimed at:

A. Making India Digital

B. Promoting bank financing for start ups and offer incentives to boost entrepreneurship and job creation.

C. Promoting Swatch Bharat

D. Promoting free education to all children below the age of 14 years and assuring them jobs after obtaining higher education.

 

Q. 45 Who is the author of “Crime & Punishment”?

A. Geoffrey Chaucer

B. Fyodor Dostoevsky

C. Harold Joseph Laski

D. Karl Marx

 

Q. 46 Power to summon the Houses of the Parliament is vested with:

A. Vice President

B. President

C. Prime Minister

D. Speaker and Vice President

 

Q. 47 Which country is the first developed nation to default on debt of IMF?

A. USA

B. UK

C. Japan

D. Greece

 

Q. 48 Which State of India was declared in January, 2016 as the “first organic farming State of India”?

A. Punjab

B. Sikkim

C. Kerala

D. Haryana

 

Q. 49 Punjabi has become the ______________ most common language in the Parliament of Canada.

A. fourth

B. fifth

C. second

D. third

 

Q. 50 Which State has become the first State to introduce compulsory gender education at the graduate level?

A. Telangana

B. Kerala

C. Punjab

D. Andhra Pradesh

 

Q. 51 The highest peace time gallantry award Ashok Chakra was awarded posthumously during 2016 to:

A. Mohan Nath Goswami

B. Mahendra Singh

C. Jagdish Chand

D. Rajesh Atra

 

Q. 52 The Constitution (One Hundredth Amendment) Act, 2015 was enacted to give effect to: 

A. the transfer of certain territories by India to Bangladesh and transfer of certain

territories from Bangladesh to India

B. the acquiring of territories by India from Bangladesh.

C. National Judicial Appointment Commission

D. transfer of certain territories by India to Bangladesh.

 

Q. 53 The Currency of Thailand is:

A. Baht

B. Rupee

C. Ringgit

D. Peseta

 

Q. 54 Parliament of India consists of:

A. Council of States

B. House of the People

C. House of the People and Council of States

D. President, House of the People and Council of States

 

Q. 55 The direction to hold floor test to prove majority in the Legislative Assembly of Uttarakhand , to be held on 10th May, 2016 has been give n on 6th May, 2016 , by 

A. The Supreme Court of India

B. The High Court of Uttarakhand

C. The President of India

D. The Governor of Uttarakhand

 

Q. 56 Which organization has the motto ‘Be Prepared’?

A. National Disaster Management Authority

B. National Cadet Corps

C. Boys’ Scout

D. National Service Scheme

 

Q. 57 Under the Constitution of India the official language of the Union is:

A. English and Hindi

B. English

C. Hindi

D. Hindi in Devanagari Script

 

Q. 58 At the Asian Indoor Athletics Championships held at Doha in February, 2016, who among women won the gold medal in Long Jump?

A. Irina Ektova of Khazagstan

B. Mayooka Johny of India

C. Prajusha of India

D. Olga Rypakova of Khazakstan

 

Q. 59 The “Paris Agreement” was adopted in the twenty first session of Conference of Parties in the month of:

A. December, 2015

B. January, 2016

C. November, 2015

D. February, 2016

 

Q. 60 Who was conferred the Rajiv Gandhi Khel Ratna award during August, 2015?

A. Dhoni

B. Virat Kohli

C. Saina Nehwal

D. Sania Mirza

 

Q. 61 Who is the President of Sri Lanka?

A. Jayavardane

B. K. Sripavan

C. Ranil Wickramasinghe

D. Maithripala Sirisena

 

Q. 62 Who has scored the fastest century in a Test Cricket match?

A. A B de Villiers, South Africa

B. Misbah Ul Haq, Pakistan

C. Brendon McCullum, New Zealand

D. Vivian Richards, West Indies

 

Q. 63 Who is the Chairperson of NITI Aayog?

A. Ms. Sushma Swaraj

B. Sh. Narendra Modi

C. Sh. Arun Jately

D. Ms. Sushmita Mahajan

 

Q. 64 Who is the Chairperson of National Human Rights Commission?

A. Justice B.S. Chauhan

B. Justice Balakrishnan

C. Justice D.K. Jain

D. Justice H.L. Dattu

 

Q. 65 Who was the Constitutional Advisor to the Constituent Assembly in framing the Indian Constitution?

A. Dr. Rajendra Prasad

B. Dr. B.R. Ambedkar

C. Sir B.N. Rau

D. Pandit Jawaharlal Nehru

 

Q. 66 In April 2016, The Duke and Duchess of Cambridge visited India. Their names are: 

A. Prince Robert and Kateler

B. Prince Charles and Cathy

C. Prince William and Kate Middleton

D. Prince Hillery and Victoria.

 

Q. 67 In case the President of India wants to resign, he shall address his resignation to the:

A. Chief Justice of India

B. Vice President

C. Chief Election Commissioner

D. Prime Minister

 

Q. 68 Name the Indian Gymnast who has become the first female Indian Gymnast to qualify for the Olympics.

A. Dipa Karmakar

B. Anjubala

C. Bisweshwari Nandi

D. Rupinder Kaur

 

Q. 69 In case of death of both the President and Vice-President of India, who shall act as the President of India?

A. Prime Minister

B. The Parliament will nominate from among its Members

C. Chief Justice of India

D. Attorney General of India

 

Q. 70 Who is the Union Finance Minister of India?

A. Mr. D.V. Sadananda Gowda

B. Ms. Sushma Swaraj

C. Mr. Arun Jaitly

D. Mr. Rajnath Singh

 

Q. 71 Who is the Chairman of National Green Tribunal?

A. Justice Balakrishna Reddy

B. Justice R.C. Lahoti

C. Justice Swatanter Kumar

D. Justice Bhandhari

 

Q. 72 Which State has the largest number of foreign students in India?

A. Uttar Pradesh

B. Maharashtra

C. Haryana

D. Karnataka

 

Q. 73 Who lit the torch from the sun’s rays reflected in a parabolic mirrorduring the Olympic flame lighting ceremony for the Rio 2016 Olympic Games at the site of ancient Olympia in Greece on 22nd April, 2016?

A. Katerina Lehou

B. Ronaldo

C. Katerina Kaif

D. Joanie Laureh aka Chyna

 

Q. 74 The Parliament of which country became first Parliament in the world to run entirely on Solar Power?

A. UK

B. India

C. Pakistan

D. Japan

 

Q. 75 Prime Minister, Mr. Narendra Modi, in March, 2016, launched an ambitious programme “Setu Bharatam”. The programme is aimed at:

A. Linking all major cities and towns by rail, air and road by 2019.

B. Making all National Highways railway level crossing free by 2019

C. Providing travel concession to senior citizens throughout India.

D. Providing travel concession to women and children throughout India

 

Q. 76 How many Indian Universities/Institutes figure among the top 200 in BRICS rankings?

A. 6

B. 16

C. 26

D. 20

 

Q. 77 Which day is celebrated as “World Consumer Rights Day”?

A. 15th March

B. 5th September

C. 5th June

D. 15th July

 

Q. 78 Who was crowned as the winner of Femina Miss India 2016 and who will represent India at the Miss World pageant?

A. Sushruthi Krishna

B. Pankhuri Gidwani

C. Sushobita Kapoor

D. Priyadarshini Chatterjee

 

Q. 79 ___________ is the fastest planet to revolve around the Sun.

A. Mercury

B. Mars

C. Jupiter

D. Venus

 

Q. 80 Who has written the book “Indomitable Spirit”?

A. Dr. A.P.J. Abdul Kalam

B. Justice Krishna Iyer

C. Ms. Arundhati Roy

D. Sh. Narendra Modi

 

Q. 81 The Rajya Sabha in April, 2016 passed two Bills, which had already been passed by the Lok Sabha earlier, repealing certain outdated/old l aws. The Bills intended to repeal:

A. 315 laws

B. 1053 laws

C. 153 laws

D. 513 laws

 

Q. 82 “Beyond the Lines – An Autobiography” is authored by:

A. Mr. Arun Shourie

B. Mr. Kuldip Nayar

C. Justice Krishna Iyer

D. Mr. Soli Sarab Ji

 

Q. 83 The maximum number of ‘Smokeless Villages’ are found in the State of:

A. Punjab

B. Rajasthan

C. Telangana

D. Karnataka

 

Q. 84 In an attempt to curb black money, the Government has made PAN mandatory for all financial transactions exceeding Rupees:

A. 4 lakhs

B. 3 lakhs

C. 2 lakhs

D. 1 lakhs

 

Q. 85 In which of the following States / Union Territories, the Election Commission has decided to hold election in a single phase on May 16, 2016?

A. Assam, Kerala and Pondicherry.

B. West Bengal, Kerala and Tamil Nadu.

C. Kerala, Tamil Nadu and Pondicherry.

D. West Bengal, Kerala and Assam.

 

Q. 86 Name the country that has six Deputy Prime Ministers

A. Nigeria

B. Nicaragua

C. Nepal

D. Maldives

 

Q. 87 Country’s first ‘visually challenged friendly’ railway station is:

A. New Delhi

B. Varanasi

C. Tirupathi

D. Mysuru

 

Q. 88 The largest diaspora in the world is from:

A. Mexico

B. South Africa

C. India

D. China

 

Q. 89 A Japanese maglev, which is the fastest passenger train in the world, has broken its own record in 2015. The train reached ___________ kmph in the test run.

A. 503

B. 403

C. 603

D. 453

 

Q. 90 In January, 2016, at the first stage, the Union Ministry of Urban Development unveiled the list of ________ cities for smart cities mission out of 98 shortlisted cities:

A. 30

B. 15

C. 20

D. 25

 

Q. 91 The number of ‘three digit numbers’ which are multiples of 9 are:

A. 98

B. 101

C. 100

D. 99

 

Q. 92 The value of a machine depreciates every year at the rate of 10% on its value at the beginning of that year. If the present value of the machine is Rs. 729, its worth three years ago was:

A. Rs. 947.70

B. Rs. 1,000

C. Rs. 750.87

D. Rs. 800

 

Q. 93 The angle subtended by the Minor segment of a circle at the center is __________ the angle subtended by the Major segment at the center of the circle.

A. greater than

B. not related to

C. equal to

D. lesser than

 

Q. 94 What is the sum of all the natural numbers from 1 to 100?

A. 5000

B. 5050

C. 6000

D. 5052

 

Q. 95 A part of monthly hostel charges is fixed and the remaining depends on the number of days one has taken food in the mess. When a student A takes food for 20 days, she has to pay Rs. 1000 ashostel char ges whereas a student B, who takes food for 26 days, pays Rs. 1180 as hostel charges.Find the fixed charges and the cost of food per day.

A. 300, 30

B. 400, 40

C. 200, 20

D. 400, 30

 

Q. 96 A library has an average of 510 visitors on Sundays and 240 on other days. What is the average number of visitors per day in the month of June beginning with a Sunday?

A. 250

B. 280

C. 285

D. 276

 

Q. 97 12 defective pens are accidentally mixed with 132 good ones. It is not possible to just look at a pen and tell whether or not it is defective. One pen is taken out at random from this lot. Determine the probability that the pen taken out is a good one.

A. 7/12

B. 11/12

C. 10/12

D. 9/12

 

Q. 98 A man earns Rs. 20 on the first day and spends Rs. 15 on the next day. He again earns Rs. 20 on the third day and spends Rs. 15 on the fourth day. If he continues to save in this way, how soon will he have Rs. 60 in hand?

A. on 27th day

B. on 24th day

C. on 12th day

D. on 17th day

 

Q. 99 60 kg of an alloy X is mixed with 100 kg of an alloy Y. If alloy X has lead and tin in the ratio of 3:2 and alloy Y has tin and copper in the ratio of 1:4, then the amount of tin in the new alloy is

A. 53 kgs

B. 80 kgs

C. 36 kgs.

D. 44 kgs.

 

Q. 100 60 kg of an alloy X is mixed with 100 kg of an alloy Y. If alloy X has lead and tin in the ratio of 3:2 and alloy Y has tin and copper in the ratio of 1:4, then the amount of tin in the new alloy is

A. 53 kgs.

B. 80 kgs

C. 36 kgs

D. 44kgs

 

Q. 101 The traffic lights at three different signal points change after every 45 seconds, 75 seconds and 90 seconds respectively. If all change simultaneously at 7:20:15 hours, then they will change again simultaneously at

A. 7:27:30 hours

B. 7:28:00 hours

C. 7:27:50 hours

D. 7:27:45 hours

 

Q. 102 A circular park, 42 m in diameter, has a path 3.5 m wide running around it on the outside. Find the cost of gravelling the path at Rs. 4 per m2

A. Rs. 1672

B. Rs. 1652

C. Rs. 2002

D. Rs. 2048

 

Q. 103 A train which is moving at an average speed of 40 kmph, reaches its destination on time. When its average speed reduces to 35 kmph, then it reaches its destination 15 minutes late. The distance travelled b y the train is:

A. 80 kms

B. 40 kms

C. 70 kms

D. 30 kms

 

Q. 104 The mean of 72 items was found to be 63. If two of the items were misrecorded as 27 and 9 instead of 72 and 90 respectively, find the correct mean.

A. 64.75

B. 64.25

C. 63.25

D. 65.75

 

Q. 105 Principle: Defamation is the publication of a statement which tends to lower reputation of a person in the estimation of other members of the society generally.

Facts: ‘A’ writes a highly offensive and derogatory letter about ‘B’, and send it directly to ‘B’ in a sealed cover.

A. ‘A’ is liable to ‘B’ for defamation, as the letter is highly offensive and derogatory and is directly sent to ‘B’.

B. A’ is not liable to ‘B’ for defamation, since there is no publication to any other person in whose estimation the reputation of ‘B’ could be brought down.

C. ‘A’ is liable to ‘B’ for defamation, as the letter is highly offensive a nd derogatory.

D. ‘A’ is liable to ‘B’ for defamation, as it has hurt his (B’s) self-esteem.

 

Q. 106 A patient in a hospital is given soup daily in a cylindrical bowl of dia meter 7 cm. If the bowl is filled with soup to a height of 4 cm, how much soup the hospital has to prepare daily to serve 250 patients?

A. 40 litres

B. 38 litres

C. 38.5 litres

D. 39.5 litres

 

Q. 107 The angles between the hands of a clock when the time is 4:25 am is:

A. 14½ degrees

B. 12½ degrees

C. 17½ degrees

D. 13½ degrees

 

Q. 108 A shop gives 15% discount on the purchase of a T.V. If paid for in cash immediately, a further discount of 12% is given. If the marked price is Rs. 15,000, what is the price of the T.V if cash purchase is made?

A. Rs. 12,750

B. Rs. 11,220

C. Rs. 10,950

D. Rs. 11,475

 

Q. 109 If a boy is standing at the top of the tower and another boy is at the ground at some distance from the foot of the tower, then the angle of elevation and depression between the boys when both look at each other will be

A. Angle of elevation will be greater

B. Cannot be predicted for relation

C. Angle of depression will be greater

D. Equal

 

Q. 110 If the product of zeroes of the polynomial ax2 – 6x – 6 is 4, find the value of ‘a’

A. -3/2

B. -1/2

C. 3/2

D. 1/2

 

Question 111

Section : Legal Aptitude 

This section consists of Fifty (50) questions. Follow the instructions carefully and choose the most appropriate option:

The question number 111-145 in this section consist of legal proposition(s) (hereinafter referrred to as ‘principle’) and facts. Such principles may or may not be true in hte real and legal sense, yet you have to conclusively assume them to be true for the purpose of this section. In other words, in answering the following question. Further, you must not assume any facts other than those stated in the question. The objective of this section is to test your interest towards study of law, research aptitude and problem solving ability, even if the ‘most reasonable conclusion’ arrived at may e absurd or unacceptable for any other reason. To answer a question, the given principle is to be applied to the given facts so as to arrive at most reasonable conclusion.

 

Q. 111 Principle : Intentional application of force to another person is action able in law.

Facts : ‘P’ and ‘D’ are unknown to each other. When ‘P’ is about to sit on a chair, ‘D’ intentionally pulls it away as a result of which ‘P’ falls on the floor and is injured.

A. ‘D’ is not liable as ‘P’ is not seriously injured.

B. ‘D’ is liable as he intentinally caused injury to P.

C. ‘D’ is not liable as such jokes are common in the society.

D. ‘D’ is not liable as the injury is not directly caused.

 

Q. 112 Principle :

1. Wagering agreement are void.

2. Collateral agreement to wagering contracts are valid.

Facts : XYZ Bank lends Rs. 40,000 to Sabu in order to enable him to award as prize to

Randeep who is the winner of horse race. Later Sabu refused to pay the prize stating that horse racing is wagering agreement. Can XYZ Bank recover money from Sabu.

A. Yes it is only a collateral agreement to horse racing and there fore the bank can recover the money from Sabu.

B. Horse racing is illegal and therefore XYZ Bank cannot recover anything from Sabu.

C. No, as it is as wagering contract.

D. Bank can recover money from Sabu so that payment of prize money can be made to Randeep.

 

Q. 113 Principle : Where a person lawfully does anything for another person, or delivers anything to him, not intending to do so or to provide gratuitously, and such other person takes the benifits of that; the latter is bound to compensate the former for somethind done or thing provided, or ot restore, the thing so delivered.

Facts: Trader ‘A’ delivers certain eatables at ‘B’s house by mistake. ‘B’ consmed the eatables without asking anything. Which of the following derivations is correct?

A. ‘B’ is bound to pay ‘A’ for the eatables.

B. ‘B’ can be made liable to pay for the eatables, only if ‘A’ establish as an express contract between ‘A’ and ”B’.

C. ‘B’ is not bound to pay ‘A’ for the eatables.

D. It is the discretin of ‘B’ to make payment to ‘A’

 

Q. 114 Principle : Consent is a good defence in a civil action for tort but the act should be the same for which consent was given.

Facts : ‘B’ was formally invited by ‘A’ to his house. ‘B’ after sitting for some time in drawing room, moved to the bed room of the house. ‘A’ sued ‘B’ for trespass.114

A. ‘B’ has interfered with privacy of ‘A’.

B. ‘B’ has committed no trespass as he entered the house with ‘A’s consent.

C. ‘B’ has offended ‘A’ by moving to bed room.

D. ‘B’ has committed trespass as there was no consent of ‘A’ for entry in the Bed room.

 

Q. 115 Principle : Whoever takes away any moveable thing form the land of any person without that person’s consent, he is said to have committed theft.

Facts : During his visit to the house of ‘C’, ‘A’ asked ‘B’ the son of ‘C’, to accompany ‘A’ to the forest. Neither ‘A’ nor ‘B’ informed ‘C’ in this regard. ‘B’ accompanied ‘A’ to the forest. 

A. ‘A’ has committed theft as soon as he entered the house of ‘C’.

B. ‘A’ has not committed theft.

C. ‘ A’ has committed theft

D. ‘A’ has not committed theft till ‘B’ did not accompany him.

 

Q. 116 Principle: The communication of a proposal is complete when it comes to the knowledge of the person to whom it is made.

Facts : ‘A’ sent a letter making a proposal to ‘B’ to purchase the hous e of ‘B’.

A. The communication of proposal is complete when ‘B’ reads the letter.

B. The communication of proposal is complete when A sent the letter.

C. The communication of proposal is complete when B’s wife received it.

D. The communication of proposal is complete when B’s wife handed over the letter to ‘B’.

 

Q. 117 Principle: Law does not penalise for wrongs which are of trivial nature.

Facts: In the course of a discussion, ‘A’ threw a file of papers at the t able which touched the hands of ‘B’.

A. ‘A’ is liable for his act, as the file touched ‘B’s hand.

B. ‘A’ is liable for his act, as it assaulted ‘B’.

C. ‘A’ is liable for insulting ‘B’.

D. ‘A’ is not liable for his act, as it was of trivial nature.

 

Q. 118 Principle: Copyright law protects only work. ‘Work’ means cinemato graphic film but does not include performance by an actor in a cine matographic film.

Facts: Alia Bhatt acted in a movie

A. The acting of Alia Bhatt can be protected under copyright law as professional work.

B. The acting of Alia Bhatt cannot be protected under copyright law.

C. The acting of Alia Bhatt can be protected under copyright law only as an artistic work

D. The acting of Alia Bhatt can be protected as film producer’s work.

 

Q. 119 Principle: Import means bringing some consignment into India from a foreign country. 

Facts: A consignment from Sri Lanka entered the territorial waters of India. However, this consignment never crossed the Indian custom barrier nor did it enter into the stream of commerce in India.

A. The consignment will only be imported into India when it enters into the stream of commerce in India.

B. The consignment was not imported into India.

C. The consignment was imported into India.

D. The consignment will only be imported into India when it crosses the Indian custom barrier

 

Q. 120 Principle: A person is said to have committed assault when an apprehension is caused in the mind of a person that he is about to use ph ysical force against his body.

Facts: ‘A’ abuses ‘B’ while he was sitting in a moving train, by aggressively shaking his fists when ‘B’ was standing on the railway platform at a distance.

A. ‘A’ has caused apprehension of assault in the mind of ‘B’.

B. ‘A’ has committed assault against ‘B’.

C. A has not committed assault against ‘B’.

D. ‘A’ has caused fear of assault in the mind of ‘B’

 

Q. 121 Principle: Sale of liquor is illegal. All agreements relating to prohibit ed items do not exist in the eyes of law.

Facts: ‘A’ entered into an agreement with ‘B’ for the sale of liquor. ‘A’ failed to supply the agreed quantity of liquor to ‘B’.

A. ‘B’ can bring a legal action against ‘B’.

B. ‘A’ can bring a legal action against ‘B’.

C. ‘B’ cannot bring any legal action against ‘A’.

D. ‘A’ and ‘B’ can initiate appropriate legal proceeding against each other.

 

Q. 122 Principle: Nothing is an offence which is done by a child under twelve years of age, who has not attained sufficient maturity of understanding to judge the nature and consequences of his conduct on that occasion.

Facts: Himesh, 11 years old boy, picks up a gold ring worth Rs 5000/-lying on a table in his friend’s house and immediately sells it for Rs 2000/-, and misappropriates the money. 

A. Himesh would not be protected under the principle stated above because, irrespective of the age, stealing is an offence.

B. Himesh would be protected under the principle stated above because he is below 12 years of age.

C. Himesh would be protected under the principle stated above because his acts show that he was not sufficiently mature to understand the nature and consequences of his conduct.

D. Himesh would not be protected under the principle stated above because his acts show that he was sufficiently mature to understand the nature and consequences of his conduct.

 

Q. 123 Principle:One who dishonestly misappropriates or converts to his own use or sells any movable property belonging to another, is guilt y of the offence of misappropriation. 

Facts: ‘A’ takes property belonging to ‘Z’ out of Z’s possession, in good faith, believing when he takes it, that the property belongs to him self. Subsequently, ‘A’, on discovering his mistake, without disclosin g the actual facts, dishonestly sells the property to a stranger.

A. ‘A’ may be guilty of theft but not for misappropriation.

B. ‘A’ is guilty of an offence of misappropriation.

C. ‘A’ is not guilty because when he took the property, he believed in good faith that it belonged to him.

D. ‘A’ is not guilty as the property can be recovered from the stranger.

Q. 124 Principle: Letters or words not describing quality of things can be registered as a trade mark.

Facts: Ram made an application for registration of alphabet ‘B’ written in a fancy style as trade mark to be applied on packets and cartons of shoes manufactured by him.

A. The alphabet ‘B’ can be registered as trade mark because it describes the quality of things.

B. The alphabet ‘B’ cannot be registered as trade mark because it is an English letter.

C. The alphabet ‘B’ can be registered as trade mark.

D. The alphabet ‘B’ cannot be registered as trade mark because it belongs to humanity.

 

Q. 125 Principle: Defamation is the publication of a statement which tends to lower reputation of a person in the estimation of other members of the society generally.

Facts: ‘A’ writes a highly offensive and derogatory letter about ‘B’, and sends it directly to ‘B’ in a sealed cover.

A. ‘A’ is liable to ‘B’ for defamation, as the letter is highly offensive and derogatory and is directly sent to ‘B’.

B. A’ is not liable to ‘B’ for defamation, since there is no publication to any other person

in whose estimation the reputation of ‘B’ could be brou ght down.

C. ‘A’ is liable to ‘B’ for defamation, as the letter is highly offensive a nd derogatory.

D. ‘A’ is liable to ‘B’ for defamation, as it has hurt his (B’s) self-estee m.

 

Q. 126 Principle: Existence of all the alleged facts is relevant, whether they occurred at the same time and place or at different times and places.

Facts: ‘A’, a citizen of England, is accused of committing murder of ‘B’ in India by taking part in a conspiracy hatched in England.

A. Only the fact that ‘A’ citizen of England is accused of committing murder of ‘B’ in India is relevant.

B. ‘A’ citizen of England cannot be tried in India.

C. Only the fact that ‘A’ is accused of conspiracy hatched in England is relevant.

D. The facts that ‘A’ citizen of England is accused of commission of murder in India and of conspiracy hatched in England are relevant facts.

 

Q. 127 Principle: An agreement without free consent can be enforces only at the option of the party whose consent was not free.

Facts: A obtains the consent of ‘B’ to enter into an agreement by put ting a gun on the head of B’s girl friend.

A. ‘B’ can enforce the agreement.

B. ‘B’ cannot enforce the agreement.

C. Neither ‘A’ nor ‘B’ can enforce the aggrement.

D. ‘A’ can enforce the agreement.

 

Q. 128 Principle: Acceptance of proposal must be the exact mirror image of the proposal. 

Facts: ‘A’ made a proposal to ‘B’ to sell a chair for Rs. 500. ‘B’ expressed his desire to buy the said chair for Rs. 400.

A. It is not clear whether A made a proposal to ‘B’.

B. ‘B’ has accepted the proposal of ‘A’.

C. ‘B’ has not accepted the proposal of ‘A’.

D. It is not clear whether ‘B’ has accepted the proposal of ‘A’ or not.

 

Q. 129 Principle: Mere silence as to facts lakely to affect the decision of a person to enter into a contract is not fraud.

Facts: ‘A’ sells to ‘B’ (A’s daughter who is minor) a horse which ‘A’ knows to be unsound. ‘A’ says nothing to ‘B’ about the unsoundness of the horse.

A. ‘B’ can take plea of fraud because she is minor.

B. There can not be a contract between a father and daughter.

C. ‘A’ has not committed fraud.

D. ‘A’ has committed fraud.

 

Q. 130 Principle: A person, who is usually of unsound mind, but occasionally normal, may make a contract when he is not of unsound mind.

Facts: ‘A’ generally remains in the state of unsound mind and rarely becomes capable of understanding the things.

A. ‘A’ can make a contract when normal.

B. ‘A’ can make a contract only for his own benefit.

C. ‘A’ can never make a contract.

D. ‘A’ can make a contract at any time whenever he pleases.

 

Q. 131 Principle: In case where there is an infringement of legal right even without any actual loss or damage, the person whose right is Infringed has a cause of action.

Facts: ‘P’ was wrongfully prevented by the Returning officer from exercising his vote in an assembly election. Still he (‘P’) brought an action claiming damages. Which of the following derivations is correct?

A. ‘P’ would succeed in his action, as it is mandatory to cast vote.

B. ‘P’ would succeed in his action, as he was wrongfully prevented from exercising his legal right of voting in that election.

C. ‘P’ would not succeed in his action, as he did not suffer any loss in that election.

D. ‘P’ would not succeed in his action, as the candidate for whom he wanted to give his vote won the election.

 

Q. 132 Principle: There are certain acts which, though harmful, are not wrongful in law; therefore, do not give legal right to bring action in law, to the person who suffers from such acts. 

Facts: ‘Prakash’ has a rice mill. His neighbour, Shanti, sets up another rice mill and offers a tough competition to Prakash. As a consequence, Prakash’s profits fall down. He brings a suit against Shanti for damages.

A. Prakash can succeed in his claim as it is a case of actual damages.

B. Prakash cannot succeed in his claim for damages, as it is a case of damage without infringement of any legal right.

C. Prakash may succeed in his claim for damages, as it is a case of loss to his business.

D. Prakash can succeed in his claim for damages, as it is a case of damage as a result of infringement of his legal right.

 

Q. 133 Principle: A condition to a contract can also be complied with after the happening of the event to which such a condition is attached.

Facts: ‘A’ promises to pay Rs. 5000 to ‘B’ on the condition that he shall marry with the consent of ‘C’, ‘D’ and ‘E’. ‘B’ marries without the consent of ‘C’, ‘D’ and ‘E’, but obtains their consent after the Marriage. 

A. ‘B’s marriage is not valid.

B. ‘B’ has not fulfilled the condition.

C. The condition is illegal

D. ‘B’ has fulfilled the condition.

 

Q. 134 Principle: Killing is not murder if the offender, whilst deprived of the power of self-control by intense and sudden provocation, causes the death of the person who gave the provocation.

Facts: ‘A’, a man found his girl friend sleeping, in her own bed room, with another man

named ‘B’. ‘A’ did not do anything but went to his home, picked a gun and cartridges, returned to the girl friend’s bed room with loaded gun but found the place empty. After fifteen days he saw his girl friend dining in a restaurant. Without waiting for even a second, ‘A’ fired five bullets at his girl friend who died on the spot.

A. ‘A’ could have killed ‘B’ instead of his girl friend.

B. ‘A’ could have killed both ‘B’ and his girl friend.

C. ‘A’ did not kill his girl friend under intense and sudden provocation.

D. ‘A’ killed his girl friend under intense and sudden provocation

 

Q. 135 Principle: Whoever by words or writing conveys to others any imputation concerning any person’s reputation is said to defame that person.

Facts: During a marriage ceremony, ‘A’ circulated a pamphlet saying that ‘S’, sister of the bride, is a thief, she has stolen the shoes of the bridegroom.

A. ‘A’ has defamed the bridegroom.

B. ‘A’ defamed the bride.

C. ‘A’ has defamed ‘S’.

D. ‘A’ did not defame ‘S’ as he never intended it.

 

Q. 136 Principle: Causing of an effect partly by an act and partly by an omission is an offence. 

Facts: ‘A’ confined her daughter ‘D’ in a room. ‘A’ also did not provide any food to her daughter ‘D’. Consequently, ‘D’ died of starvation.

A. ‘A’ committed the offence of confining ‘D’.

B. ‘A’ committed the offence of causing death of ‘D.

C. ‘A’ committed no offence.

D. ‘A’ committed the offence of not providing food to ‘D’.

 

Q. 137 Principle: Whoever does not arrest the killer and report the matter to the concerned authorities commits an offence.

Facts: ‘A’, a woman, sees ‘B’ , another woman, killing a third woman ‘C’. ‘A’ neither

attempted to arrest ‘B’ nor informed the concerned authorities.

A. ‘A’ has not committed an offence.

B. ‘A’ has committed an offence.

C. ‘B’ has not committed an offence.

D. ‘B’ has committed an offence.

 

Q. 138 Principle: False imprisonment is a tort (wrong) which means the total restraint of a person’s liberty without lawful justification.

Facts: A part of a public road had been closed for spectators of a boat race. ‘P’ wanted to

enter but hewas prevented by ‘D’ and other policemen because he had not paid the admission fee. ‘P’ was able toenter the enclosure by other means but was unable to go where he wanted to go. The policemenrefused access to where he wanted to go but allowed him to remain where he was or to go back. ‘P’ remained within the enclosure and refused to leave. Subsequently, ‘P’ sued ‘D’ for false imprisonment.

A. It was a case of false imprisonment, but ‘D’ could not be made liable for it.

B. ‘D’ could not be made liable for false imprisonment as he has not touched him.

C. ‘D’ could be made liable for false imprisonment, as he did restrict P’s movements.

D. ‘D’ could not be made liable for false imprisonment, as he did not totally restrict P’s movements

 

Q. 139 Principle: An independent contractor is one who is employed to do some work of his employer. He is engaged under a contract for services. He undertakes to produce a given result, and in the actual execution of the work, he is not under the direct control or

following directions of hi semployer. He may use his own discretion in execution of the work assigned.In general, an employer is not liable for the torts (wrongful acts) of his independent contractor. But,the employer may be held liable if h e directs him to do some careless acts.

Facts: Ramesh hired a taxi-cab to go to Delhi Airport. As he started l ate from his home, he kept on urging the taxi-driver to drive at a high speed and driver followed the directions; and ultimately due to high speed an accident took place causing injuries to a person.

A. Ramesh would be held liable for damages as he exercised the control by giving directions to the driver.

B. Ramesh would not be held liable for damages because the drive r was an independent contractor and not his servant.

C. Ramesh would not be held liable for damages because Ramesh did not know the consequences of such rash driving.

D. Ramesh would not be liable as car was not owned by him.

 

Q. 140 Principle: Nothing is an offence, which is done by accident or misfortune, and without any criminal intention or knowledge in the doing of a lawful act in a lawful manner by lawful means and with proper car e and caution.

Facts: ‘A’ takes up a gun, not knowing whether it is loaded or not, points it playfully at ‘B’ and pulls the trigger. Consequently, ‘B’ falls dead.

A. ‘B’s death is not accidental, as there was want of proper care and caution on the part of ‘A’.

B. ‘B’s death is accidental, as ‘A’ had no intention to kill ‘B’.

C. ‘B’s death is accidental, as ‘A’ was just pointing the gun playfully at ‘B’.

D. ‘B’s death is accidental, as ‘A’ did not have the knowledge that the gun is loaded.

 

Q. 141 Principle: An agreement may be entered into orally or in writing, or by conduct.

Facts: ‘A’ went to the shop of ‘B’ and picked a toothbrush and gave a cheque of Rupees twenty to ‘B’ and left the shop. 

A. Payment of toothbrush cannot be made through a cheque

B. ‘A’ did not enter into an agreement with ‘B’.

C. ‘A’ should have carried a currency note of Rupees twenty to make the payment.

D. There was an agreement between ‘A’ and ‘B’.

 

Q. 142 Principle: Law never enforces an impossible promise.

Facts: ‘A’ made a promise to ‘B’ to discover treasure by magic.

A. Law will not enforce the promise .

B. Law will enforce the promise.

C. Law will enforce the promise only at the option of ‘A’.

D. Law will enforce the promise only at the option of ‘B’

 

Q. 143 Principle: Consent is a good defence for civil action in tort. But consent must include both knowledge of risk and assumption of risk, i.e, readiness to bear harm.

Facts: A lady passenger was aware that the driver of the cab, in which she opted to travel was little intoxicated. The cab met with an accident and lady got injured.

A. Driver can take the plea that he was lightly intoxicated.

B. Lady is not entitled to claim compensation as she had knowledge of the risk.

C. Lady is entitled to claim compensation as she only knew about risk and there was no assumption of risk.

D. Lady can refuse to pay the fare as she had suffered injuries.

 

Q. 144 Principles: • A person is said to abet the doing of a thing when he instigates any other person to do that thing. • Mere acquiescence, however, does not amount to instigation. 

Facts: ‘A’ says to ‘B’: I am going to kill ‘C’.” And, ‘B’ replies: “Do as you wish and take the consequences”; whereafter ‘A’ kills ‘C’.

A. ‘B’ has not abetted ‘A’ to kill ‘C’.

B. ‘B’ has abetted ‘A’ by conspiracy.

C. ‘B’ abetted ‘A’ to kill ‘C’.

D. ‘B’ is jointly liable with ‘A’ for killing ‘C’.

 

Q. 145 Principles: • A servant is one who is employed to do some work for his employ er (master). He is engaged under a contract of service. He works directly under the control and directions of his master. • In general, the master is vicariously liable for those torts (wrongful acts) of his servant which are done by the servant in the course of his employment.

Facts: ‘M’ appointed ‘D’ exclusively for the purpose of driving his tourist vehicle. ‘M’ also appointed ‘C’ exclusively for the purpose of performing the work of a conductor for the tourist vehicle. During one trip, at the end of the journey, ‘C’, while ‘D’ was not on the driver’s seat, and apparently for the purpose of turning the vehicle in the right direction for the next journey, drove it through the street at high speed, and negligently injured ‘P’. 

A. ‘M’ could be made liable for the act of ‘C’, as his (C’s) act of driving the vehicle was within his scope of employment.

B. ‘M’ is not liable as he was not present at the time of accident.

C. ‘M’ could not be made liable for the act of ‘C’, as his (C’s) act of d riving the vehicle was not in the course of his employment.

D. ‘M’ could be made liable for the act of ‘C’, as ‘C’ was employed nder a contract of service.

 

Q. 146 ‘alibi’ means a plea by an accused person that he –

A. was facing trial.

B. was present elsewhere

C. underwent preventive detention.

D. remained in judicial custody

 

Q. 147 Under the Constitution of India restriction on freedom of religion can not be placed on the ground of –

A. Public order

B. Social justice

C. Morality

D. Health

 

Q. 148 If an authority is holding information about another in a ‘fiduciary capacity’, the

information under the Right to Information Act, 2005 may not be obtainable. ‘Fiduciary relationship’ is based on: 

A. Authority

B. Law

C. Trust

D. Contract

 

Q. 149 Which one of the following is not a Directive Principle of State Policy under Part IV of the Constitution of India?

A. Promotion of adult education.

B. Promotion of International peace and security.

C. Organisation of village panchayats.

D. Provision for just and humane conditions of work and maternity relief.

 

Q. 150 ‘audi alteram partem’ means –

A. Not connected to facts.

B. Giving opportunity of hearing of the other side.

C. Following the substantive law.

D. A transferee cannot retransfer

 

Q. 151 Which among the following was described by Dr. B. R. Ambedkar as the “heart and soul of the Constitution of India”?

A. Right to Constitutional Remedies

B. Right to equality

C. Freedom of Religion

D. Right to move throughout the territory of India

 

Q. 152 obiter dicta’ means –

A. Direction by a judge.

B. Basis of judicial decision.

C. Judgment of a court in the case before it.

D. An opinion given by the court not necessary for the decision.

 

Q. 153 Under the Constitution of India ‘Right to Pollution Free Environment’ has emerged as a fundamental right from the right to

A. Life and personal liberty under Article 21

B. Freedom of movement under Article 19

C. Conserve culture under Article 29.

D. Equality under Article 14

 

Q. 154 ‘persona non grata’ means –

A. Non-person

B. An unacceptable person

C. Non-performance of promise.

D. Ungrateful person

 

Q. 155 The object of which one of the following writs is to prevent a person to hold public office which he is not legally entitled to hold ?

A. Quo warranto

B. Mandamus

C. Prohibition

D. Certiorari

 

Q. 156 Which among the following does not belong to the ‘right to freedom of religion’?

A. Freedom of conscience and free profession, practice and propagation of religion.

B. Freedom from attending religious instruction or religious worship in certain educational institutions.

C. Freedom from payment of taxes for promotion of any particular religion .

D. Freedom of speech and expression .

 

Q. 157 ‘lis pendens’ means –

A. A pending suit

B. On the basis of evidence

C. Awaited information

D. Decision awaited.

 

Q. 158 Which Indian State has prescribed minimum educational qualification for candidates contesting panchayat polls?

A. Gujarat

B. Kerala

C. Punjab

D. Haryana

 

Q. 159 As per law the minimum age for the marriage of a boy and a girl in India is

A. 21 years in both cases

B. 18 years and 21 years respectively

C. 21 years and 18 years respectively

D. 18 years in both cases

 

Q. 160 The Supreme Court of India has struck down the Constitution (Ninet yninth Amendment) Act, 2014 as unconstitutional. It is related to –

A. National Judicial Appointment Commission

B. Religious Rights

C. Land Exchange between India and Bangladesh

D. Jallikattu (Bull Fighting)

 

Questions: 161 – 163

Direction for Questions 161 – 163

Read the following information carefully and choose the most appropriate option:

A and B are good in driving bus and car. C and A are good in driving car and scooter. C, D and E are good in driving scooter and tractor. E and C are good in driving scooter and auto rickshaw. D and B are good in driving bus and tractor. 

 

Q. 161 Who is good in driving scooter, tractor and auto rickshaw but not good in driving car?

A. A

B. E

C. D

D. C

 

Q. 162 Who is good in driving scooter, tractor and bus?

A. C

B. D

C. B

D. A

 

Q. 163 Who is good in driving tractor, scooter, car and autorickshaw but no t bus?

A. C

B. D

C. B

D. A

 

Questions: 164 – 166

Direction for Questions 164 – 166

Read the following information carefully and choose the most appropriate option:

In each question, there is a statement and two assumptions numbered as I and II. Read the statement and find which of the given assumptions is implicit:

(A) If assumption I is implicit.

(B) If assumption II is implicit.

(C) If neither assumption I nor Assumption II is implicit.

(D) If both Assumption I and Assumption II are implicit.

 

Q. 164 Statement: The next meeting of the executive board of a company will be held after six months.

Assumptions:

I. Existing executive board will be dissolved before six months

II. The company will remain in function after six months

A. Neither assumption I nor Assumption II is implicit.

B. Assumption II is implicit.

C. Both Assumption I and Assumption II are implicit.

D. Assumption I is implicit.

 

Q. 165 Statement: In the State of Zuminisia, people prefer to travel by X airline instead of Y airline, as X airline has advanced German security system and 99% on time operations. Assumptions:

I. Airline X with advanced German security system and record on time operation is perceived better than airline Y.

II. Had advanced German security system and on time performance record of Y airline been implemented, it would have been preferred over airline X.

A. Assumption I is implicit.

B. Neither assumption I nor Assumption II is implicit.

C. Assumption II is implicit.

D. Both Assumption I and Assumption II are implicit.

 

Q. 166 Statement:To attend a convocation ceremony scheduled to be held on Thursday at GM University, Chennai, Mr X left for Chennai on Tuesday by train.

Assumptions:

I. Mr X may reach home on Saturday

II. Mr X may reach the University on Wednesday

A. Assumption I is implicit.

B. Assumption II is implicit.

C. Both Assumption I and Assumption II are implicit.

D. Neither assumption I nor Assumption II is implicit

 

Questions: 167 – 168

Direction for Questions 167 – 168

Read the following information carefully and choose the most appropriate option:

In a joint family, A, B, C, D, E, and F are the members. B is the son of C. A and C are husband and wife. C is not the mother of B. E is the brother of C. D is the daughter of A and F is the brother of B. 

 

Q. 167 Which of the following is a pair of females in the family?

A. AE

B. AD

C. DF

D. BD

 

Q. 168 Who is the mother of B?

A. D

B. F

C. A

D. E

 

Questions: 169 – 170

Read the following information carefully and choose the most appropriate option:

In each of the questions, two statements are numbered as I and II. There may be a cause and effect relationship between the two statements. The statements maybe the effect of the same cause or an independent cause. The statements may be independent causes without having any relationship.

Read both the statements and mark your answer as:

(A) If statement I is the cause and statement II is its effect.

(B) If statement II is the cause and statement I is its effect.

(C) If both statements I and II are independent causes.

(D) If both statements I and II are effects of independent causes.

 

Q. 169 Statement I: In last two years, there is a considerable reduction in c ncellation of flights due to fog in North India.

Statement II: In last two years, there is a considerable improvement in passenger amenities on all airports of North India.

A. Both statements I and II are effects of independent causes.

B. Statement I is the cause and statement II is its effect.

C. Statement II is the cause and statement I is its effect.

D. Both statements I and II are independent causes

 

Q. 170 Statement I: The Government, by legislation has decided to make all public information available to general public.

Statement II: Before passing of legislation, general public did not have access to public information.

A. Both statements I and II are independent causes.

B. Statement I is the cause and statement II is its effect.

C. Statement II is the cause and statement I is its effect.

D. Both statements I and II are effects of independent causes.

 

Q. 171 Identify the statement which cannot be false.

A. Democracy is the best form of government.

B. Water evaporates at 100° C.

C. All radii of any given circle are of equal length.

D. Myopia is a congenital disorder.

 

Q. 172 An old woman decided to divide her gold among her daughter and d aughters-in-law. She first kept exactly half of the gold for her daughter. Then she divided the rest of her gold among her daughters-in-law. The eldest one got 26 grams more than the youngest daughter-in-law. The middle one got wice as the youngest one. If the eldest daughter-in-law got 66 grams of gold, how much was recieved by the daughter.

A. 198 grams

B. 172 grams

C. 186 grams

D. 194 grams

 

Q. 173 Find the odd one out from the following group.

WINDSHIELD, SPARK PLUG, CLUTCH PEDAL, CAR, ENGINE

A. Car

B. Engine

C. Windshield

D. Spark Plug

 

Q. 174 Four statements are given below. Group two of them in such a way that one is logically incorrect and the other is verifiable as a matter of fact

(a) The Sun does not rise in the East.

(b) A straight line is the shortest distance between any two points.

(c) Every circle has a centre.

(d) The maximum duration of a total solar eclipse is about 7.5 minute s

A. [a,c]

B. [c,d]

C. [a,d]

D. [a,b]

 

Q. 175 This question consists of a related pair of words, followed by four pairs of words. Choose the pair that best represents a similar relations hip as the one expressed in the given pair of words:

SANDERLING : BIRD

A. Mastiff: Dog

B. Cat : Mare

C. Frog : Toad

D. Water : Fish

 

Q. 176 From among the given options, identify the statement which means t he same as the statement ‘The dual nature of light is an enigma’.

A. Two contradictory descriptions of light presuppose a third description.

B. Light has distinct properties which makes it unique.

C. Light is mysterious

D. The nature of light is an enigma.

 

Q. 177 Which word in the following group DOES NOT belong to the others?

17PROSPER, EXCITE, THRIVE, FLOURISH

A. Prosper

B. Excite

C. Flourish

D. Thrive

 

Q. 178 Examine the following numbers and identify the next number:

45; 43; 40; 36; 31; 25; ….

A. 23

B. 29

C. 17

D. 18

 

Q. 179 f it is true that ‘All humans are imperfect’, then which one of the following is necessarily true?

A. No humans are imperfect.

B. Some humans are not imperfect.

C. Every human is imperfect.

D. All imperfect beings are humans.

 

Q. 180 If it is false that ‘Men always pray to God’, then which one of the following statements is true?

A. Men seldom pray to God.

B. Men always pray to God.

C. Some men pray to God.

D. All men pray to God.

 

Q. 181 Geeta is twice the present age of Seema. If age of Seema is 20 years now, how many years ago Geeta was three times Seema’s age?

A. 10

B. 8

C. 12

D. 9

 

Q. 182 In certain code MAIL is written as ZNVY then how will FILM be written in that code? 

A. SVYZ

B. MLIF

C. NORY

D. XLMP

 

Q. 183 You are given a 60inch long ribbon, and you are instructed to cut 60 1-inch long strips out of this ribbon. The time taken to cut one strip i s one second. So how long will it take to cut 60 strips?

A. 1 minute

B. 1 minute and 1 second

C. 30 seconds

D. 59 seconds

 

Q. 184 If it is true that ‘Religious fundamentalism is dangerous to the society’, then which one of the following statements can also be true?

A. Disrespect for other religions is dangerous to the society.

B. Religious pluralism is dangerous to the society.

C. Religion without reason is dangerous to the society.

D. Belief in any religion is dangerous to the society.

 

Q. 185 If it is true that ‘Good governance implies law and order in society’ t hen identify the statement which has to be accepted along with the given statement?

A. An able government very effectively uses laws to promote peace.

B. A healthy society is governed by maximum number of laws.

C. A strong government uses force to impose laws.

D. Law is indispensable for the society

 

Q. 186 If it is false that ‘There is at least one octogenarian in the room’, then which one of the following is probable?

A. No one in the room is an octogenarian.

B. All those who are in the room are octogenarians.

C. One person in the room is not an octogenarian.

D. Some people in the room are not octogenarians.

 

Q. 187 A, B, C and D have got some money with them. If A gives 8 rupees to B, B will have as much as C has and A will have 3 rupees less than what C has. Also if A takes 6 rupees from C, A will have twice as much money as D. If B and D together have 50 rupees, how much money does A and B have respectively ?

A. 29, 32

B. 27, 40

C. 32, 29

D. 40, 27

 

Q. 188 Who among the following is the odd one in the following group of persons?

Chief Justice of India, Attorney General of India, Solicitor General, Advocate General

A. Chief Justice of India.

B. Advocate General

C. Solicitor General

D. Attorney General of India

 

Q. 189 In a bag, there are some diamonds. In another bag, there are one fourth the number more has the number of diamonds in the first bag. If the difference in the number of diamonds in the first and second bag is 3, how many diamonds are there in the first bag?

A. 10

B. 16

C. 12

D. 8

 

Q. 190 Mr. X, the President of a club arrived in a meeting at 10 minutes to 12 30 hrs. Mr X came earlier by 20 minutes than the other participating members in the meeting, who arrived late by 30 minutes. At what time was the meeting scheduled?

A. 1210 hrs

B. 1220 hrs

C. 1240 hrs

D. 1250 hrs

 

Q. 191 In a class, student X has 8th position from the top and 84th from the bottom. How many students are there in the class?

A. 96

B. 98

C. 91

D. 88

 

Q. 192 There is some relationship between the figures given in the series. Find out the missing one from the alternatives below:

A. A

B. B

C. C

D. D

 

Q. 193 If Oceans are Deserts, then Waves are:

A. Powerful

B. Ripples

C. Sand Dunes

D. Water

 

Q. 194 Assume that both premises, ‘No innocent person should be punished’ and ‘Socrates is innocent’ are true. Then which one of the following options is necessarily true?

A. Socrates may not be punished.

B. Socrates is not punished.

C. It is not proper to punish Socrates.

D. Socrates should not be punished

 

Q. 195 A 2100 member team consisting of Team Leaders and Athletes is attending a National Athletic Meet. For every 20 Athletes, there is one Team Leader. How many Team Leaders would be there in the team?

A. 100

B. 105

C. 110

D. 95

 

Q. 196 Choose the pair of words from the options that best represents a similar relationship as the one expressed in the following pair of word s.

WAITER: RESTAURANT

A. Driver : Car

B. Teacher : School

C. Author : Book

D. Actor : Acting

 

Q. 197 Examine the series and identify the missing number:

46, 44, 40, 38, 34, …

A. 26

B. 32

C. 28

D. 30

 

Q. 198 Identify the argument which cannot be accepted

A. All unmarried women are spinsters. Therefore, all spinsters are women.

B. All wives are married. Therefore, all married people are wives.

C. All equilateral triangles are equiangular. Therefore, all equiangul ar triangles are equilateral.

D. All rainy days are wet days. Therefore, all wet days are rainy day s.

 

Q. 199 Identify the statement which cannot be true.

A. Snow is white.

B. All bachelors are faithful to their wives.

C. Black body radiation is a physical phenomenon.

D. Every natural number has a successor

 

Q. 200 If it is false that ‘Animals are seldom aggressive’, then which one of t he following statements conveys the same meaning?

A. All animals are always aggressive.

B. All animals are aggressive.

C. At least one animal is aggressive.

D. Sometimes animals are aggressive.

 

Answer Sheet 
Question 1 2 3 4 5 6 7 8 9 10
Answer B B D D A A C B C B
Question 11 12 13 14 15 16 17 18 19 20
Answer B A A D C C A A A B
Question 21 22 23 24 25 26 27 28 29 30
Answer C B A B B B C A B D
Question 31 32 33 34 35 36 37 38 39 40
Answer D B A B C B D B D B
Question 41 42 43 44 45 46 47 48 49 50
Answer A A C B B B D B D A
Question 51 52 53 54 55 56 57 58 59 60
Answer A A A D A C D B A D
Question 61 62 63 64 65 66 67 68 69 70
Answer D C B D C C B A C C
Question 71 72 73 74 75 76 77 78 79 80
Answer C D A C B B A D A A
Question 81 82 83 84 85 86 87 88 89 90
Answer B B D C C C D C C C
Question 91 92 93 94 95 96 97 98 99 100
Answer C B D B D C B D D B
Question  101 102 103 104 105 106 107 108 109 110
Answer D C C A B C C B D A
Question  111 112 113 114 115 116 117 118 119 120
Answer B A A D B A D B C C
Question 121 122 123 124 125 126 127 128 129 130
Answer C D B C B D A C C A
Question 131 132 133 134 135 136 137 138 139 140
Answer B B D C C B B D A A
Question 141 142 143 144 145 146 147 148 149 150
Answer D A C C C B B C A B
Question 151 152 153 154 155 156 157 158 159 160
Answer A D A B A D A D C A
Question 161 162 163 164 165 166 167 168 169 170
Answer B B A B A B B C A C
Question 171 172 173 174 175 176 177 178 179 180
Answer C C A C A D B D C A
Question 181 182 183 184 185 186 187 188 189 190
Answer A A D A A A D A C A
Question 191 192 193 194 195 196 197 198 199 200
Answer C D C D A B B B B D

 

CLAT 2015 Previous Year Paper

CLAT 2015

Questions: 1 – 3

In the question, there are five sentences. Each sentence has pairs of words/phrases that are italicized and highlighted. From the italicized and highlighted word(s)/phrase(s), select the most appropriate word(s)/phrases(s) to form correct sentences. Then, from the options, given choose the best one.

 

Q. 1 The further [A]/ farther [B] he pushed himself, the more disillusioned he grew.

For the crowd it was more of a historical [A]/ historic [B] event; for their leader it was just another day.

The old has a healthy distrust [A]/ mistrust [B] for all new technology.

The film is based on a worthy [A]/ true [B] story.

She coughed discreetly [A]/ discretely [B] to announce her presence.

A. BABAB

B. ABBBA

C. BAABA

D. BBAAB

 

Q. 2 Regrettably [A]/ Regretfully have to decline your invitation. The critics censored [A]/ censured [B] the new movie because of its social unacceptability.

He was besides [A]/ beside [B] himself with range when I told him what I had done.

Anita had a beautiful broach [A]/ assent [B] to surgical treatment.

A. BABBA

B. BBAAB

C. ABBBA

D. BABAB

 

Q. 3 The prisoner’s interment [A]/ internment [B] came to an end with his early release.

She managed to bite back the ironic [A]/ caustic[B] retort on the tip of her tongue. Jeans that flair[A]/ flare[B] at the bottom are in fashion these days.

They heard the bells peeling[A]/ pealing[B] far and wide.

The students baited[A]/ bated[B] the instructor with irrelevant questions.

A. BBABB

B. ABBBB

C. BABBA

D. BBBBA

 

Q. 4 Identify the incorrect sentence/ sentences.

A) I want to do an MBA before going into business.

B) Priti’s husband has been on active service for three months.

C) The horse suddenly broke into a buckle.

D) I need to file an insurance claim.

A. B and C

B. C only

C. B and D

D. B, C and D

 

Q. 5 Identify the incorrect sentence/ sentences.

A) I must run fast to catch up with him.

B) The newly released book is enjoying a popular run.

C)The doctor is on a hospital round.

D) You can’t run over him like that

A. A and C

B. D only

C. A, C and D

D. A only

 

Q. 6 Identify the incorrect sentence/ sentences.

A) The letter was posted to the address.

B) Your stand is beyond all reasons.

C) How do you deal with friend who doesn’t listen to a reason?

D) My wife runs a profitable business in this suburb.

A. A only

B. D only

C. B and C

D. C and D

 

Questions: 7 – 12

Fill in the blanks, numbered [1] [2] [3] [4] [5] and [6] in the passage given below with the most appropriate word from the options given for each blank. “Between the year 1946 and the year 1995, I did not file any income tax returns”. With the [1] statement, Soubhik embarked on an account of his encounter with the income tax department. “I originally owed Rs.20,000 in unpaid taxes. With [2] and [3], the 20,000 became 60,000. The Income Tax Department then went into action, and I learned first-hand just how much power the Tax Department wields. Royalties and trust funds can be [4]; automobiles may be [5], and auctioned off. Nothing belongs to the [6] until the case is settled”. 

 

Q. 7 Fill up the blank [1]

A. devious

B. blunt

C. tactful

D. pretentious

 

Q. 8 Fill in the blanks [2]

A. interest

B. taxes

C. principal

D. returns

 

Q. 9 Fill up the blank [3]

A. sanctions

B. refunds

C. fees

D. fines

 

Q. 10 Fill in the blank [4]

A. closed

B. detached

C. attached

D. inpounded

 

Q. 11 Fill in the blank [5]

A. smashed

B. seized

C. dismantled

D. frozen

 

Q. 12 Fill up the blank [6]

A. purchaser

B. victim

C. investor

D. offender

 

Questions: 13 – 14

Four alternatives summaries are given the text. Choose the option that best captures the essence of the text.

Q. 13 Some decisions will be fairly obvious – “no brainers”. Your bank account is low, but you have a two-week vacation coming up and you want to get away to some place warn to relax with your family. Will you accept your in-laws offer of free use of their Florida beach front condo? Sure. You like your employer and feel ready to move forward in your career. Will you step in for your boss for three weeks while she attend a professional development course? Of course.

A. Some decisions are obvious under certain circumstances. You may, for example, readily accept a relative’s offer of free holiday accommodation. Or step in for your boss when she is away.

B. Some decisions are no-brainers. You need not think when making them. Examples are condo offers from in-laws and job offers from bosses when your bank account is low or boss is away.

C. Easy decisions are called “no-brainers” because they do not require any cerebral activity. Examples such as accepting free holiday accommodation abound in our lives.

D. Accepting an offer from in-laws when you are short on funds and want a holiday is a nobrainer. Another no-brainer is taking the boss’s job when she is away.

A. A

B. B

C. C

D. D

 

Q. 14 Physically, inertia is a feeling that you just can’t move; mentally, it is a sluggish mind. Even if you try to be sensitive, if your mind is sluggish, you just don’t feel anything intensely. You may even see a tragedy enacted in front of your eyes and not be able to respond meaningfully.. You may see one person exploiting another, one group persecuting another, and not be able to get angry. Your energy is frozen. You are not deliberately refusing to act; you just don’t have the capacity.

A. Inertia makes your body and mind sluggish. They become insensitive to tragedies, exploitation, and persecution because it freezes your energy and decapitates it.

B. When you have inertia you don’t act although you see one person exploiting another or one group persecuting another. You don’t get angry because you are incapable.

C. Inertia is of two types – physical and mental. Physical inertia restricts bodily movements. Mental inertia prevents response to events enacted in front of your eyes.

D. Physical inertia stops your body from moving; mental inertia freezes your energy and stops your mind from responding meaningfully to events, even tragedies, in front of you.

A. A

B. B

C. C

D. D

 

Q. 15 For the word a contextual sentence is given. Pick the word from the alternatives given that is most inappropriate in the given context.

SPECIOUS : A specious argument is not simply a false one but one that has the ring of truth. 

A. Deceitful

B. Fallacious

C. Credible

D. Deceptive

 

Q. 16 For the word a contextual sentence is given. Pick the word from the alternatives given that is most inappropriate in the given context.

OBVIATE : The new mass transit system may obviate the need for the use of personal cars.

A. Prevent

B. Forestall

C. Preclude

D. Bolster

 

Q. 17 For the word a contextual sentence is given. Pick the word from the alternatives given that is most inappropriate in the given context.

DISUSE : Some words fall into disuse as technology makes objects obsolete.

A. Prevalent

B. Discarded

C. Obliterated

D. Unfashionable

 

Q. 18 For the word a contextual sentence is given. Pick the word from the alternatives given that is most inappropriate in the given context.

PARSIMONIOUS : The evidence was constructed from every parsimonious scraps of information.

A. Prevalent

B. Penurious

C. Thrifty

D. Altrustic

 

Q. 19 For the word a contextual sentence is given. Pick the word from the alternatives given that is most inappropriate in the given context.

FACETIOUS : When I suggested that war is a method of controlling population, my father remarked that I was being facetious.

A. Jovian

B. Jovial

C. Jocular

D. Joking

 

Questions: 20 – 21

Answer the question based on the following information. Indicate which of the statement given with that particular question consistent with the description of unreasonable man in the passage below. Unreasonableness is a tendency to do socially permissible things at the wrong time. The unreasonable man is the sort of person who comes to confide in you when you are busy. He serenades his beloved when she is ill. He asks a man who has just lost money by paying a bill for a friend to pa a bill for him. He invites a friend to go for a ride just after the friend has finished a long car trip. He is eager to offer services which are not wanted, but which cannot be politely refused. If he is present at an arbitration, he stirs up dissension between the two parties, who were really anxious to agree. Such is the unreasonable man.

 

Q. 20 The unreasonable man tends to

A. entertain women

B. be a successful arbitrator when dissenting parties are anxious to agree

C. be helpful when solicited

D. tell a long story to people who have heard it many times before

 

Q. 21 The unreasonable man tends to

A. bring a higher bidder to a salesman who has just closed a deal

B. disclose confidential information to others

C. sing the praise of the bride when he goes to a wedding.

D. sleep late and rise early

 

Questions: 22 – 24

In the following sentence, a part of the sentence is underlined. Beneath each sentence, four different ways of paraphrasing the underlined part are indicated. Choose the best alternative among the four options. 

 

Q. 22 The management can still hire freely BUT CANNOT SCOLD FREELY.

A. cannot scold at will

B. cannot give umbrage

C. cannot take decision to scold

D. cannot scold willfully

 

Q. 23 This government has given subsidies to the Navratnas but there is NO TELLING WHETHER THE SUBSEQUENT ONE WILL DO.

A. whether the subsequent government will do so

B. if the government to follow will accept the policy

C. if the government to follow will adhere to the policy

D. no telling whether the subsequent one will do so

 

Q. 24 The Romanians may be restive under Soviet direction but THEY ARE TIED TO MOSCOW BY IDEOLOGICAL AND MILITARY LINKS.

A. they are close to Moscow from ideological and military perspective

B. they are preparing for a greater revolution

C. secretly they rather enjoy the prestige of being protected by the mighty soviets

D. there is nothing they can do about it

 

Questions: 25 – 27

In the question, a related pair of words or phrases is followed by a pair of words or phrases. Select the pair that best expresses a relationship similar to the one expressed in the original pair.

 

Q. 25 Dulcet : Raucous

A. Sweet : Song

B. Crazy : Insane

C. Palliative : Exacerbating

D. Theory : Practical

 

Q. 26 Malapropism : Words

A. Anachronism : Time

B. Ellipsis : Sentence

C. Jinjanthropism : Apes

D. Catechism : Religion

 

Q. 27 Peel : Peal

A. Coat : Rind

B. Laugh : Bell

C. Rain : Reign

D. Brain : Cranium

 

Questions: 28 – 37

In view of this passage given below. Choose the best option for question. 

When talks come to how India has done for itself in 50 years of Independence, the world has nothing but praise for our success in remaining a democracy. On other fronts, the applause is less loud. In absolute terms, India has not done too badly, of course, life expectancy has increased. So has literacy. Industry, which was barely a fledgling, has grown tremendously. And as far as agriculture is concerned, India has been transformed from a country perpetually on the edge of starvation into a success story held up for others to emulate. But these are competitive times when changed is rapid, and to walk slowly when the rest of the world is running is almost as bad as standing still on walking backwards. Compared with large chunks of what was then the developing world South Korea, Singapore , Malaysia, Thailand, Indonesia, China and what was till lately a separate Hong Kong – India has fared abysmally. It began with a far better infrastructure than most of these countries had. It suffered hardly or not at all during the Second world war. It had advantages like an English speaking elite, quality scientific manpower (including a Nobel laureate and others who could be ranked among the world’s best) and excellent business acumen. Yet, today, when countries are ranked according to their global competitiveness, it is tiny Singapore that figures at the top. Hong Kong is an export powerhouse. So is Taiwan. If a symbol were needed of how far we have fallen back, note that while Korean Cielos ares old in India, no one is South Korea is rushing to buy an Indian car. The reasons list themselves. Topmost is economic isolationism. The government forgot that it itself could not create, but only squander wealth. Some of the manifestations of the old attitude have changed. Tax rates have fallen. Licensing has been all but abolished. And the gates of global trade have been opened wide. But most of these changes were first by circumstances partly by the foreign exchange bankruptcy of 1991 and the recognition that the govt. could no longer muster the funds of support the public sector, leave alone expand it. Whether the attitude of the government itself, or that of more than handful of ministers, has changed, is open to question. In many other ways, however, the government has not changed one with. Business still has to negotiate a welter of negotiations. Transparency is to see beyond their noses. A no-exit policy for labour is equivalent to a no-entry policy for new business. If one industry is not allowed to retrench labour, other industries will think a hundred times before employing new labour, In other ways too, the government hurts industries. Public sector monopolies like the department of telecommunications and Videsh Sanchar Nigam Ltd. make it possible for Indian business to operate only at a cost several times that of their counterparts abroad. The infrastructure is in a shambles party because it is unable to formulate a sufficiently remunerative policy for private business, and partly because it does not have the stomach to change market rates for services. After a burst of activity in the early nineties, the government is dragging its feet. At the rate it is going, it will be another fifty years before the government realizes that a pro- business policy is the pro-people policy. By then of course, the world would have moved even farther ahead. 

 

Q. 28 The writer’s attitude towards the Government is …

A. critical

B. ironical

C. sarcastic

D. derisive

 

Q. 29 The writer is surprised at the Government’s attitude its industrialists because….

A. the government did not heed to protect its industrialists.

B. the issue of competition was non-existent

C. the government looked upon its industrialists as crooks

D. the attitude was a conundrum

 

Q. 30 The Government was compelled to open the economy due to…

A. pressure from international market

B. pressure from domestic market

C. foreign change bankruptcy and paucity with the government

D. All of the above

 

Q. 31 The writer ends the passage on a note of…

A. cautious optimism

B. pessimism

C. optimism

D. pragmatism

 

Q. 32 According to the writer India should have performed better than the other Asian nations because…

A. it had adequate infrastructure

B. it had better infrastructure

C. it had better politicians who could take the required decisions.

D. All of the above

 

Q. 33 India was in better condition than the other Asian nations because…

A. it did not face the ravages of the Second World War.

B. it had an English speaking populace and good business sense.

C. it had enough wealth through its export.

D. Both (a) and (b) above.

 

Q. 34 The major reason for India’s poor performance is…

A. Economic isolationism

B. Economic mismanagement

C. Inefficient industry

D. All of these

 

Q. 35 One of the factors of the government’s projectionist policy was…

A. encouragement of imports

B. discouragement of imports

C. encouragement of exports

D. discouragement of exports

 

Q. 36 The example of the Korean Cielo has been presented to highlight…

A. India’s lack of stature in the international market.

B. India’s poor performance in the international market.

C. India’s lack of credibility in the international market.

D. India’s disrepute in the international market.

 

Q. 37 According to the writer….

A. India’s politicians are myopic in their vision of the country’s requirement.

B. India’s politicians are busy lining their pocket.

C. India’s politician’s are not conversant with the needs of the present scenario.

D. All of the above

 

Q. 38 Choose the option closest in meaning to the Capitalized word.

GRANDIOSE…

A. imposing

B. unpretentious

C. boring

D. lanky

 

Q. 39 Choose the option closest in meaning to the Capitalized word.

SPRY……

A. doubtful

B. nimble

C. prognosticate

D. leave

 

Q. 40 Choose the option closest in meaning to the Capitalized word.

FUDGE…..

A. to sweeten

B. smear

C. irritate

D. falsify

 

Q. 41 Attukal Pongal festival, which is figured in Guinness Book of world records is celebrated in…

A. Tamil Nadu

B. Kerala

C. Telangana

D. Goa

 

Q. 42 In February 2015, which Indian Cricket legend has been inducted into the ICC Hall of Fame?

A. Rahul Dravid

B. Anil Kumble

C. Sachin Tendulkar

D. Mohammed Azharuddin

 

Q. 43 Which of the following Acts formally introduced the principle of elections for the first time?

A. The Indian Councils Act, 1909

B. Government of India Act, 1919

C. The Government of India Act, 1935

D. India’s Independence Act, 1935

 

Q. 44 IRTCT has recently launched a new service called ‘Rupay prepaidcards’ which will enable passengers to book their tickets, do shopping and pay service bills online. This service was launched in collaboration with which bank?

A. Union Bank of India

B. State Bank of India

C. ICICI Bank

D. Bharatiya Mahila Bank

 

Q. 45 Garuda Shakti III is the military exercise between India and which country?

A. Nepal

B. Russia

C. Indonesia

D. China

 

Question 46

Match List – I with List – II and select the best option using the code given below the lists.

 

Q. 46 List 1 ( Organization / Centers) List – II (Locations)

A. High Altitude Warfare School               1. Chennai

B. Indian Air Force Training Center         2. Gulmarg

C. National Defense College                       3. New Delhi

D. Institute of National Recognition        4. Pune

A. A-2, B-1, C-3, D-4

B. A-1, B-2, C-3, D-4

C. A-1, B-2, C-4, D-3

D. A-2, B-1, C-4, D-3

 

Q. 47 Name India’s Beyond Visual Range (BVR) Air-to-missile which was successfully test fired on 19 March 2015 from a Sukhoi-30 fighter aircraft?

A. Astra

B. K-100

C. Mitra

D. Tejas

 

Q. 48 The ISRO has developed a “Flood Hazard Atlas” by mapping flood prone and vulnerable areas in which state?

A. Kerala

B. Maharashtra

C. Assam

D. Tripura

 

Q. 49 Which South East Asian country has recently banned surrogacy service to end its

flourishing rent-a-womb industry? 

A. Singapore

B. Laos

C. Thailand

D. Vietnam

 

Q. 50 Which of the following is the oldest share market in India?

A. Bombay

B. Madras

C. Calcutta

D. Delhi

 

Q. 51 The name is new Andhra Pradesh capital city is likely to be……

A. Amaravathi

B. Badrachala

C. Krishna Nagar

D. Varshavathi

 

Q. 52 The Ulfa city, where annual BRICS summit – 2015 is scheduled to be held is in which country? 

A. China

B. Russia

C. South Africa

D. Brazil

 

Q. 53 The “Ease of Doing Business Index” is prepared and published by…..

A. World Trade Organization

B. World Bank Group

C. United Nations

D. European Union

 

Q. 54 Prime Minister Modi has launched the “Give It Up” campaign for voluntarily giving up….. 

A. Use of tobacco products

B. LPG subsidy

C. Use of incandescent bulbs

D. Use of plastics

 

Q. 55 Which one of the following Railway Zones and the corresponding Headquarter pairs is not correctly matched?

A. North Eastern Railway: Gorakhpur

B. South Eastern Railway: Bhubaneshwar

C. Eastern Railway: Kolkata

D. South East Central Railway: Bilaspur

 

Q. 56 Which among the following is the world’s largest e- commerce company?

A. Amazon

B. eBay

C. Alibaba

D. Flipkart

 

Q. 57 Which committee was constituted by RBI to review governance of boards of banks in India?

A. P J Nayak Committee

B. H R Khan Comittee

C. Harsh Vardhan Comittee

D. K Subramanian Comittee

 

Q. 58 The recently announced Paramparagat Krishi Vikas Yojana aims to boost…?

A. Organic Farming

B. Drip Irrigation

C. Horticulture crops

D. Vegetable production

 

Q. 59 The winner of 2015 Malaysian Grand Prix is…?

A. Sebastian Vettel

B. Kimi Raikkonen

C. Lewis Hamilton

D. Jenson Button

 

Q. 60 Which of the following is essentially a solo dance nowadays performed in group as well?

A. Kuchipudi

B. Kathak

C. Manipuri

D. Mohiniattam

 

Q. 61 Who among the following was the author of Rajtarangini, commonly regarded as the first genuine history of India written by an Indian?

A. Banbhatta

B. Ravikirti

C. Pushpananda

D. Kalhana

 

Q. 62 Name the golfer who won the Indian Open title on 22 February 2015.

A. SSP Chowrasia

B. Anirban Lahiri

C. Siddikur Rahman

D. Daniel Chopra

 

Q. 63 Which space agency has successfully launched the world’s first all electric satellites in March, 2015?

A. Russia Federal Space Agency

B. China National Space Administration

C. SpaceX

D. European Space Agency

 

Q. 64 Who among the following 18th century Indian rulers has been called ‘Plato of his tribe’?

A. Sawai Jai Singh

B. Badam Singh

C. Suraj Mal

D. Guru Gobing Singh

 

Q. 65 Bhalchandra Nemade who has been selected for the 50th Jnanpith Award for 2014, on 6 february 2015, is a famous writer in which language?

A. Marathi

B. Oriya

C. Malayalam

D. Urdu

 

Q. 66 A Snickometer is associated with which sports?

A. Tennis

B. Cricket

C. Hockey

D. Football

 

Question 67

Which of the following is an incorrect option? 

 

Q. 67 Which of the following is an incorrect option?

A. Within the Arcitc and Antarctic Circles there is at least one day in the year during which the sun does not set and at least one day on which it never rises.

B. At the North Pole there is darkness for half the year.

C. At the summer solstice, the sun shines vertically over the Tropic of Capricorn.

D. The sun shines vertically over the Equator twice in the year.

 

Q. 68 What is the correct sequence of the following movements in chronological order?

1. Civil Disobedience Movement

2. Khilafat Movement

3. Home Rule Movement

4.Quit India Movement

A. 1, 2, 3, 4

B. 4, 3, 2, 1

C. 3, 2 ,4, 1

D. 2, 4, 3, 1

 

Q. 69 Recently, which country became the first member country to the UN Framework

Convention on Climate Change (UNFCCC) to submit its action plan on Intended Nationally Determined Contribution (INDC)?

A. India

B. Switzerland

C. Australia

D. Singapore

 

Q. 70 Lysosomes, which are known as suicide bags, are produced by which organelle?

A. Mitochondria

B. Golgi body

C. Ribosome

D. Peroxisome

 

Q. 71 Which is the single policy rate to unambiguously signal the stance of monetary policy as recently recommend by RBI?

A. PLR

B. Repo

C. Bank

D. CLR

 

Q. 72 Which city has become India’s first fully WiFi – enabled metro city on 5 February 2015?

A. Mumbai

B. Kolkata

C. Chennai

D. Delhi

 

Q. 73 Who among the following is India’s first chief of cyber security?

A. B. J. Srinath

B. Gulshan Rai

C. A. S. Kamble

D. Amardeep S. Chawla

 

Q. 74 Which one of the following is the online grievances monitoring portals launched by union government for Indians living abroad?

A. Madad

B. Sankalp

C. Mythri

D. Rakshan

 

Q. 75 Which of the following is a Direct tax?

A. Excise duty

B. Sales tax

C. Income tax

D. None of the above

 

Q. 76 Indian Space Research Organization was recently conferred ‘Space Pioneer Award’ by the National Space Society (NSS) of which country over the historic feat on successfully sending an orbit to Martian atmosphere in its very first attempt?

A. France

B. European Union

C. China

D. USA

 

Q. 77 Recently in which country did Indian Prime Narendra Modi inaugurate the first of the eight Coastal Surveillance Radar Systems (CSRS) being set up by India?

A. Mauritius

B. Maldives

C. Sri Lanka

D. Seychelles

 

Q. 78 Greece and Turkey are working to resolve their dispute over sovereignty and related rights in the area of which sea?

A. Black Sea

B. Sea of Marma

C. Aegean Sea

D. Mediterranean Sea

 

Q. 79 Name the Indian industrialists on whose 175th birth anniversary, Union government launched the commemorative stamp on 6 January 2015?

A. G D Birla

B. T V Sundaram Iyengar

C. Kasturbhai Lalbhai

D. Jamsetji Nusserwanji Tata

 

Q. 80 Which one of the following pairs is not correctly matched?

A. Kunal Bahl & Rohit Bansal – Snapdeal

B. Sachin Bansal & Binny Bansal – Red Bus

C. Deepinder Goyal & Pankaj Chaddah – Zomato

D. Bhavish Aggarwal & Ankit Bhati – Ola cab

 

Q. 81 Which state is ti host the 36th National Games in 2016?

A. Karnataka

B. Goa

C. Tamil Nadu

D. Telanga

 

Q. 82 Project Varsha, India’s new naval base under construction is near which of the following cities?

A. Kochi

B. Karwar

C. Vishakapatnam

D. Chennai

 

Q. 83 Young Indian shuttler K. Srikanth on 15 March 2015 won which of the following major badminton championship?

A. China Grand Prix Gold

B. Australian Grand Prix Gold

C. Swiss Grand Prix Gold

D. Indonesian Grand Prix Gold

 

Q. 84 What is the name given to the dedicated TV channel for farmers that was announced in the Union Budget for 2014-15 and Rs. 100 crore was set aside for its establishment? 

A. Kisan

B. Farmers Show

C. Krishi Channel

D. None of the above

 

Q. 85 The “Friends for Life” – an elephant conservation project has been launched by World Wide Fund for Nature India and…..

A. Aditya Birla Group

B. Muthoot Group

C. Manapuram Group

D. Reliance Ltd

 

Q. 86 The National Industrial Corridor (NIC) that was proposed to be established in the Union Budget of 2014-15, will have its headquarters at which city?

A. Pune

B. Bhubaneswar

C. Bangalore

D. Hyderabad

 

Q. 87 President Pranab Mukherjee on 6 January 2015 signed the ordinance to amend Citizenship Act, 1955. Which of the statements in this regard is/are right?

I. The ordinance exempts Person of Indian Origin (PIO) from appearing before the local police station on every visit.

II. It replaced the clause that says foreigners marrying Indians must continuously stay in the country for a period of six months before they get an Indian citizenship.

A. I Only

B. II Only

C. Both I and II

D. Neither I or II

 

Q. 88 Name the renowned Indian ecologist who has been chosen for the prestigious 2015 Tyler Prize for Environmental Achievement on 23 March 2015?

A. MS Swaminathan

B. Kasthuri Rangan

C. Jayaram Ramesh

D. Madhav Gadgil

 

Q. 89 How much Foreign Direct Investment (FDI) in country’s defence sector was proposed in the Union Budget 2014-15 presented on 10 July 2014?

A. 51%

B. 49%

C. 29%

D. 10%

 

Q. 90 The protein CA – 125 (Cancer Antigen) is used as bio-marker for detection of which type of cancer?

A. Ovarian Cancer

B. Bone Cancer

C. Lung Cancer

D. Oral Cancer

 

Questions: 91 – 95

Answer the questions based on the information given in the following table. A, B, C, D, E denote companies and 2006, 2007, 2008, 2009 and 2010 denote years. A B C D E

M S M S M S M S M S

2006 2.8 1.3 3.3 2.2 2.6 1.7 3.0 2.2 1.9 1.4

2007 3.2 2.0 2.4 1.6 2.2 1.5 2.5 1.9 2.0 1.7

2008 1.9 0.9 2.9 1.6 2.1 1.0 2.3 1.5 1.6 1.1

2009 1.0 0.4 2.4 1.3 2.8 1.4 1.1 1.2 3.2 2.5

2010 2.5 1.5 2.3 1.2 2.6 2.1 1.8 1.1 3.1 2.6

 

Q. 91 What is the total number of units manufactured by Company C over all the years together?

A. 1420

B. 1030

C. 1230

D. 1320

 

Q. 92 What is the approximate percent increase in the number of units sold b Company E in the year 2007 from the previous year?

A. 17

B. 36

C. 27

D. 21

 

Q. 93 The number of units sold by the company D in the year 2006 is what percent of the number of units manufactured by it in that year?(rounded off to two digits after decimal)

A. 52.63

B. 61.57

C. 85.15

D. 73.33

 

Q. 94 What is the respective ratio of total number of units manufactured by Company A and B together in the year 2009 to those sold by them in the same year?

A. 2:01

B. 3:02

C. 5:02

D. None of the above

 

Q. 95 What is the average number of units sold by Company D all the years together?

A. 166

B. 158

C. 136

D. 147

 

Q. 96 What is the value of x in the following equation?

x⁰‧⁴/16 = 32/x²‧⁶

A. 8

B. 9

C. 6

D. 7

 

Q. 97 The simplified value of [(0.111)³ + (0.222)³ – (0.333)³ + (0.333)² × (0.222)] is :

A. 0.999

B. 0.111

C. 0

D. 0.888

 

Q. 98 When 2 1/2 is added to a number and the sum is multiplied by 4 1/4 and the 3 is added to the product and then the sum is divided by 1 1/5, the quotient becomes 25. What is that number?

A. 2 1/2

B. 3 1/2

C. 4 1/2

D. 5 1/2

 

Q. 99 if x= (16³ + 17³ + 18³ + 19³), then x divided by 70 leaves a remainder of…

A. 0

B. 1

C. 69

D. 35

 

Q. 100 A man has 9 friends, 4 boys and 5 girls. In how many ways can he invite them, if there have to be exactly 3 girls in the invitees?

A. 320

B. 160

C. 80

D. 200

 

Q. 101 A group of 630 children is arranged in rows for a group photograph session. Each row contains three fewer children than the row in front of it. What number of rows is not possible?

A. 3

B. 4

C. 5

D. 6

 

Q. 102 A die is rolled twice. What is the probability that the sum of the numbers on the two faces is 5?

A. 3/13

B. 4/14

C. 6/13

D. 1/9

 

Q. 103 Two trains, one from Howrah to Patna and other from Patna to Howrah, start

simultaneously. After they meet, the trains reach their destinations after 9 hours and 16 hours respectively. The ratio of their speed is…

A. 2 : 3

B. 4 : 3

C. 6 : 7

D. 9 : 6

 

Q. 104 A watch which gains uniformly is 2 minutes slow at noon on Monday and is 4 minute 48 second fast at 2 p.m. on the following Monday. When was it correct?

A. 2 p.m. on Tuesday

B. 2 p.m. on Wednesday

C. 3 p.m. on Thursday

D. 1 p.m. on Friday

 

Q. 105 A speaks truth in 75% cases and B in 80% of the cases. In what percentage of cases are they likely to contradict each other, narrating the same incident?

A. 5%

B. 15%

C. 35%

D. 45%

 

Q. 106 The sum of all the natural numbers from 200 to 600 (both inclusive) which are neither divisible by 8 nor by 12 is:

A. 1, 23, 968

B. 1, 33, 068

C. 1, 33, 268

D. 1, 87, 332

 

Q. 107 In a tournament, there are n teams T₁,T₂,…..Tₙ with n>5. Each team consists of k players, k>3. The following pairs of teams have one player in common T₁ and T₂, T₂ and T₃,….., Tₙ₋₁ and Tₙ and T₁. No other pair of teams has any player in common. How many players are participating in the tournament, considering all the n teams together?

A. k(n-1)

B. n(k-2)

C. k(n-2)

D. n(k-1)

 

Q. 108 If n²=12345678987654321, what is n?

A. 12344321

B. 1235789

C. 111111111

D. 11111111

 

Q. 109 Along a road lie an odd number of stones placed at intervals of 10m. These stones have to be assembled around the middle stone. A person can carry only one stone at a time. A man carried out the job starting with the stone in the middle, carrying stones in succession, thereby covering a distance of 4.8 km. Then, the number of stones is: 

A. 35

B. 15

C. 31

D. 29

 

Q. 110 What are the last two digits of 7²⁰⁰⁸?

A. 01

B. 21

C. 61

D. 71

 

Questions: 111 – 125

The question consist of two statements, one labelled as PRINCIPLE and other as FACT. You are to examine the principle and apply it to the given facts carefully and

select the best option.

 

Q. 111 PRINCIPLE: Whoever intending to take dishonestly any movable property out of the possession of any person without that person’s consent moves that property, such taking is said to commit theft.

FACT: RAMU cuts down a tree on RINKU’s ground, with the intention of dishonestly taking the tree out of RINKU’s possession without RINKU’s consent. A could not take the tree away.

A. RAMU can be prosecuted for theft

B. RAMU cannot be prosecuted for theft

C. RAMU can be prosecuted for attempt to theft

D. RAMU has neither committed theft nor attempt to commit theft

 

Q. 112 PRINCIPLE: injuria sine damnum i.e. injury without damage.

FACT: SONU, who was returning officer at a polling booth, wrongly refused to register a duly tendered vote of MONU, though he was a qualified voter. The candidate, whom MONU sought to vote, was declared elected.

A. MONU can sue SONU on the ground that he was denied to cast vote, which is a fundamental right.

B. MONU can sue SONU on the ground that he was denied to cast vote, which is a legal right.

C. MONU cannot sure SONU because there is no injury or damage cause to MONU.

D. MONU cannot sue SONU because to whom he sought to vote was declared elected

 

Q. 113 PRINCIPLE: A person is said to be of sound mind for the purpose of making a contract if, at the time when he makes it, he is capable of understanding it and of forming a rational judgement as to its effect upon his interests.

FACT: Mr. X who is usually of sound mind, but occasionally of unsound mind enters into a contract with Mr. Y when he is of unsound mind. Y came to know about this fact afterwards and now wants to file a suit against Mr. X.

A. Mr. X cannot enter into contract because he is of unsound mind when he entered into contract

B. Mr. C can enter into contract but the burden is on the other party to prove that he was of unsound mind at the time of contract.

C. Mr. X can enter into a contract but the burden is on Mr. X to prove that he was of unsound mind at the time of contract

D. None of the above

 

Q. 114 PRINCIPLE: When one person signifies to another his willingness to do or to abstain from doing anything, with a view to obtaining the assent of that other to such act or abstinence, he is said to make a proposal.

FACT: “Ramanuj telegraphed to the Shyamsunder, writing: “will you sell me your Rolls Royce CAR? Telegram the lowest cash price.” Shyamsunder replied, too by telegram: ‘Lowest price for CAR is Rs. 20 lacs.’ Ramanuj immediately sends his consent through telegram stating: ‘I agree to buy the CAR for Rs. 20 lacs asked by you.’ Now Shyamsunder refused to sell the CAR.

A. He cannot refuse to sell the CAR because the contract has already been made.

B. He can refuse to sell the CAR because it was only invitation to offer and not the real offer.

C. It was not a valid offer because willingness to enter into a contract was absent.

D. None of the above

 

Q. 115 PRINCIPLE: A master is liable for the acts committed by his servant in the course of employment.

FACT: Sanjay is a driver working in Brookebond and Co. One day, the Manager asked him to drop a customer at the airport and get back at the earliest. On his was back from the airport, he happened to see his fiance Ruhina waiting for a bus to go home. He offered to drop her at home, which happened to be close to his office. She got into the car and soon thereafter; the car somersaulted due to the negligence of Sanjay. Ruhina was thrown out of the car and suffered multiple injuries. She seeks compensation from Brookebond and Co. 

A. Brookebond and Co., shall be liable, because Sanjay was in the course of employment at the time of the accident

B. Brookebond and Co., shall not be liable, Sanjay was not in the course of employment when he took Ruhina inside the car.

C. Ruhina got into the car at her own risk, and therefore, she cannot sue anybody.

D. None of the above

 

Q. 116 PRINCIPLE: Nuisance as a tort (civil wrong) means an unlawful interference with a person’s use or enjoyment of land, or some right over, or in connection with it.

FACT: During the scarcity of onions, long queues were made outside the defendant’s shop who having a license to sell fruits and vegetables used to only 1 Kg. of onion per ration card. The queues extended onto the highway and also caused some obstruction to the neighboring shops. The neighboring shopkeepers brought an action for nuisance against the defendant.

A. The defendant is liable for nuisance

B. The defendant was not liable for nuisance

C. The defendant was liable under the principle of strict liability

D. The plaintiff’s suit should be decreed in his favour

 

Q. 117 PRINCIPLE: Nothing is an offence which is done by a person who, at the time of doing it, by reason of unsoundness of mind, is incapable of knowing the nature of the act, or that he is doing what is either wrong or contrary to law.

FACT: A takes his son B who is three years old, for a bath to the well. He throws his son inside the well so that he could have a good bath. After 10 minutes he also jumped in the well to take a bath and take his son out of the well. Both were rescued by the villagers but his son was found dead.

A. A has committed culpable homicide not amounting to murder

B. A has committed murder

C. A has done no offence as he can plead the defence of unsoundness of mind

D. A’s family should be responsible for this incident to let him take child to the well

 

Q. 118 PRINCIPLE: ignorantia juris non excusat and ignorantia facti excusat.

FACT: George was a passenger from Zurich to Manila in a Swiss Plane. When the plane landed at the airport at Bombay on 28th Nov. 1962 it was found on search that George carried 34 kgs of gold bars in person and that he had not declared it in the ‘Manifest for transit’. On 26th Nov. 1962 Government of India issued a notification and modified its earlier exemption and now it is necessary that, the gold must be declared in the “Manifest” of the aircraft.

A. George cannot be prosecuted because he had actually no knowledge about the new notification issued only two days ago

B. George cannot be prosecuted because it is mistake of fact which is excusable

C. George’s will be prosecuted because mistake of law is not excusable

D. George liability would depend on the discretion of the court

 

Q. 119 PRINCIPLE: Everybody is under a legal obligation to take reasonable care to avoid an act or omission which he can foresee would injure his neighbor. The neighbor for this purpose, is any person whom he should have in his mind as likely to be affected by his act. 

FACT: Krishnan, while driving a car at high speed in a crowded road, knocked down a cyclist. The cyclist died on the spot with a lot of blood spilling around, Lakshmi, a pregnant woman passing by, suffered from a nervous shock, leading to abortion. Lakshmi filed a suit against Krishnan claiming damages

A. Krishna will be liable, because he owed a duty of reasonable care to everybody on the road including Lakshmi

B. Krishna will not be liable, because he could not have foreseen Lakshmi suffering from nervous shock as a result of his act.

C. Krishna will be liable to Lakshmi because he failed to drive carefully

D. None of the above

 

Q. 120 PRINCIPLE: Preparation is not an offence except the preparation of some special offences. 

FACT: Ramesh keeps poisoned halua in his house, wishing to kill Binoy whom he invited to a party and to whom he wishes to give it. Unknown to Ramesh, his only son takes halua and dies. In this case

A. Ramesh is liable for the murder

B. He is not liable for murder since it is a preparation alone

C. He is liable for culpable homicide

D. None of the above

 

Q. 121 PRINCIPLE: Agreements, the meaning of which is not certain, or capable of being made certain, are void.

FACT: A horse was bought for a certain price coupled with a promise to give Rs. 500 more if the horse proved lucky.

A. This is a valid agreement

B. This agreement is void for uncertainty because it is very difficult to determine what luck, bad or good, the horse had brought to the buyer.

C. The agreement is partially valid and partially void

D. None of the above

 

Q. 122 PRINCIPLE: Mere silence as to facts likely to affect the willingness of a person to enter into a contract is not fraud, unless the circumstances of the case are such great that, regard being had to them, it is duty of the person keeping silence to speak, or unless his silence is, in itself, equivalent to speech.

FACT: A sells, by auction, to B, a horse which A knows to be unsound. A says nothing to B about the horse’s unsoundness.

A. A can be held liable for fraud

B. A can be held liable for misrepresentation

C. A cannot be held liable, because he did not say anything positive about the soundness of the horse

D. A cannot be held liable because it is the buyer who must be aware of the things

 

Q. 123 PRINCIPLE: Any direct physical interference with the goods in somebody’s possession without lawful justification is called trespass to goods.

FACT: A purchased a car from a person who had no title to it and had sent it to a garage for repair. X, believing, wrongly, that the car was his, removed it from the garage.

A. X can be held responsible for trespass to goods

B. X cannot be held responsible for trespass to good as he was under a wrong belief

C. X has not committed any wrong

D. None of the above

 

Q. 124 PRINCIPLE: “Nobody shall unlawfully interfere with a person’s use or enjoyment of land, or some right over, or in connection with it. The use or enjoyment, envisaged herein, should be normal and reasonable taking into account surrounding situation.” 

FACT: Jeevan and Pavan were neighbors in a residential locality. Pavan started a typing class in a part of his house and his typing sound disturbed Jeevan who could not put up with any kind of continuous noise. He filed a suit against Pavan.

A. Pavan is liable, because he should not have started typing class in his house

B. Pavan is liable, because as a neighbour, he should have realised Jeevan’s delicate nature

C. Pavan is not liable, because typing sound did not disturb anyone else other than Jeevan

D. None of the above

 

Q. 125 PRINCIPLE: Doctrine of Double Jeopardy. No person shall be prosecuted and punished for the same offence twice.

FACT: Maqbool brought some gold into India without making any declaration to Custom department on the airport. The custom authorities confiscated the gold under the Sea Customs Act. Maqbool was later charged for having committed an offence under Foreign Exchange Regulation Act.

A. He cannot be prosecuted because it would amount to double jeopardy.

B. He can be prosecuted because confiscation of good by custom authorities does not

amount to prosecution by the Court

C. Maqbool ought to have known that can be stopped by the custom authorities.

D. None of the above

 

Questions: 126 – 135

The question consist of two statements, one labelled as Assertion (A) and other as Reason (R).

You are to examine the two statements carefully and select the best option.

 

Q. 126 Assertion: Custom per se is law, independent of prior recognition by the sovereign or the judge.

Reason: Custom is source of law but by itself is not law.

A. Both A and R are individually true and R is correct explanation to A

B. Both A and R are individually true but R is not correct explanation of A

C. A is true but R is false

D. A is false but R is true

 

Q. 127 Assertion: Idol is a person who can hold property.

Reason: Only human being can be called person not the lifeless things.

A. Both A and R are individually true and R is correct explanation to A

B. Both A and R are individually true but R is not correct explanation of A

C. A is true but R is false

D. A is false but R is true

 

Q. 128 Assertion: Laws are means of achieving an end namely social control.

Reason: The ultimate end of law is to secure greatest happiness to greatest number.

A. Both A and R are individually true and R is correct explanation to A

B. Both A and R are individually true but R is not correct explanation of A

C. A is true but R is false

D. A is false but R is true

 

Q. 129 Assertion: Every person should have the freedom of speech and expression.

Reason: If a person is stopped from speaking then mankind will lose the truth.

A. Both A and R are individually true and R is correct explanation to A

B. Both A and R are individually true but R is not correct explanation of A

C. A is true but R is false

D. A is false but R is true

 

Q. 130 Assertion: Attempt to commit an offence though does not result in any harm, should also be punished.

Reason: A person who tries to cause a prohibited harm and fails, is, in terms of moral

culpability, not materially different from the person who tries and succeeds.

A. Both A and R are individually true and R is correct explanation to A

B. Both A and R are individually true but R is not correct explanation of A

C. A is true but R is false

D. A is false but R is true

 

Q. 131 Assertion: In India, every state has a High Court in its territory.

Reason: The Constitution of India provides for a High Court in each state.

A. Both A and R are individually true and R is correct explanation to A

B. Both A and R are individually true but R is not correct explanation of A

C. A is true but R is false

D. A is false but R is true

 

Q. 132 Assertion: The Council of Ministers at the centre is collectively responsible both to the Lok Sabha and Rajya Sabha

Reason: The members of both Lok Sabha and Rajya Sabha are eligible to be ministers of the Union Government.

A. Both A and R are individually true and R is correct explanation to A

B. Both A and R are individually true but R is not correct explanation of A

C. A is true but R is false

D. A is false but R is true

 

Q. 133 Assertion: The reservation of 33% of seats for women in Parliament and State Legislature does not require Constitutional Amendment.

Reason: Political parties contesting elections can allocate 33% of seats they contest to

women candidates without any Constitutional Amendment.

A. Both A and R are individually true and R is correct explanation to A

B. Both A and R are individually true but R is not correct explanation of A

C. A is true but R is false

D. A is false but R is true

 

Q. 134 Assertion: We, the people of India, having solemnly resolved to constitute India into a Democratic Republic.

Reason: A republic will ensure we have a head of state that is democratically elected and accountable to voters. As a result the head of state will be a more effective constitutional safeguard.

A. Both A and R are individually true and R is correct explanation to A

B. Both A and R are individually true but R is not correct explanation of A

C. A is true but R is false

D. A is false but R is true

 

Q. 135 Assertion: Republic Day is celebrated on 26th January every year in the country.

Reason: The Constitution of India came into force on 26th January 1950.

A. Both A and R are individually true and R is correct explanation to A

B. Both A and R are individually true but R is not correct explanation of A

C. A is true but R is false

D. A is false but R is true

 

Questions: 136 – 138

Read the definition and elements of the attempt, apply them on the given fact situations and answer the question:-

Definition of Attempt: Lord Blackburn has said that “there is no doubt that there is difference between a preparation antecedent to an attempt and the actual attempt, but if the actual transaction has commenced which would have ended in the crime if not interrupted, there is clearly an attempt to commit the crime.” 

1. Fault element: Intention or knowledge requisite for committing an offence; and 

2. Conduct Element: does any act towards its commission and has crossed the stage of preparation. This ast is so closely connected with, proximate to the commission that it fails in object because of facts not known to him or because of circumstances beyond his control.

 

Q. 136 ‘RANI’ ran to a well stating that she would jump into it, and she started running towards the well but she was caught before she could reach it.

A. She is not guilty of attempt to commit suicide because she might have changed her mind before jumping into the well.

B. She is guilty of attempt to commit suicide

C. Right to life includes rights to die hence a person should not be held responsible for attempt to commit suicide.

D. None of the above

 

Q. 137 ‘SINY’ with an intention to pickpocket puts his hand into MINU’s pocket. MINU had a loaded pistol in his pocket. The thief touches the pistol and trigger goes on, whereby MINU is shot dead.

A. SINY will be liable only for attempting to pickpocket and not for killing because she cannot be treated differently from all other pickpockets who steal under exactly similar circumstances and same intention, with no risk of causing death and with no greater care to avoid it

B. SINY will be liable for attempting to murder

C. SINY will be liable for culpable homicide not amounting to murder as his intention was definitely not to kill

D. None of the above

 

Q. 138 ‘JAM’ denied food to his wife JANE for several days by keeping her confined in a room with an intention to accelerate her death. JANE ultimately managed to escape.

A. JAM is guilty for attempt to murder his wife.

B. JAM is not guilty for attempt to murder his wife and he was only doing preparation

C. JAM is not guilty to murder his wife as she always had the option to escape

D. None of the above

 

Questions: 139 – 142

Fill in the blanks. Choose the pair of words which complete the sentence to make logical sense:

 

Q. 139 The NDA led Government notified the ………….. and the National Judicial Appointments Commission Act, this ending the over two-decade-old ………….. of appointing judges of Supreme Court and high courts. Under the new law, a six-member panel headed by ……………. will select judges of apex court and state high courts.

A. 99th Constitutional (Amendment) Act 2015, collegium system, the Chief Justice of India

B. 121st Constitutional (Amendment) Act 2015, collegium system, the Union Law Minister

C. 121st Constitutional (Amendment) Act 2015, collegium system, the Prime Minister

D. 99th Constitutional (Amendment) Act 2015, cabinet system, the Prime Minister

 

Q. 140 The …………….. Legislative Assembly on 31st March 2015 passed a controversial Anti- Terrorism Law. Earlier, the passed bill was rejected two times by the then ……………. in 2004 and 2008

A. Bihar, Presidents

B. Madhya Pradesh, Governors

C. Gujarat, Presidents

D. Maharashtra, Governors

 

Q. 141 The Union Government on the recommendation of the ……………… under the chairmanship of ……………. has decided to decriminalize Section …………….. of the Indian Penal Code. 

A. 20th Law Commission, Justice A.P. Shah & 309

B. 20th Law Commission, Justice M.P. Shah & 307

C. Supreme Court, Justice H.L. Dattu & Section 309

D. Planning Commission, Law Minister, section 309

 

Q. 142 A bench headed by ……………… quashed allocation of 214 ……………….. as ………………..

A. Justice H L Dattu, coal block, illegal and arbitrary

B. Justice R. M. Lodha, coal blocks, illegal and arbitrary

C. Justice T. S. Thakur, licenses, illegal and arbitrary

D. None of the above

 

Q. 143 Choose the best option for the following statement:

No one can be compelled to sing the National Anthem since:

1. It will be violative of the right to freedom of speech and expression.

2. It will be violative of the right to freedom of conscience and practice & propagation of religion.

3. There is no legal provision obliging anyone to sing the National Anthem.

A. 1 and 2 are correct

B. 2 and 3 are correct

C. 1, 2 and 3 are correct

D. None is correct

 

Q. 144 Five years’ experience is a must to be able to practice as an advocate in the Supreme Court of India. This rule was prescribed by the….

A. Bar Council of India

B. Supreme Court of India

C. High Court of Delhi

D. Ministry of Law and Justice, Government of India

 

Q. 145 Union Government recently approved 33% Reservation for Women in:

A. Horizontally and each category (OBC, SC, ST, and others) in direct recruitment in all non-gazetted Police Posts in all Union Territories including Delhi

B. Horizontally and each category (OBC, SC, ST, and others) in direct recruitment in all gazetted Police Posts in all Union Territories including Delhi

C. Horizontally and each category (OBC, SC, ST, and others) in direct recruitment in all gazetted and non-gazetted Police Posts in all Union Territories including Delhi

D. Horizontally and each category (OBC, SC, ST, and others) in direct recruitment in all gazetted and non-gazetted Posts in all Union Territories including Delhi

 

Q. 146 As per Indian Protocol, who among the followings ranks highest in the order of precedence?

A. Deputy Prime Minister

B. Former President

C. Governor of a State within his State

D. Speaker of Lok Sabha

 

Q. 147 Consider the following statements and choose the best option:

1. The Chairman of the National Legal Services Authority (NALSA) is the Chief Justice of India.

2. Chief Justice Mr. Justice H. L. Dattu is the present Chairman of NALSA.

3. The Chairman of the National Legal Services Authority (NALSA) is the senior most judge (after CJI) of the Supreme Court of India.

4. Hon’ble Mr. Justice T. S. Thakur is the present Chairman of NALSA.

A. 1 and 2 are correct

B. 2 and 3 are correct

C. 3 and 4 are correct

D. None is correct

 

Q. 148 India and Britain recently signed an “extradition treaty”. Extradition means –

A. Exports without double taxation

B. Order of Indian courts will apply to Indians living in the U.K.

C. India and the U.K. will deport criminals on reciprocal basis to each other

D. None of the above

 

Q. 149 What is a ‘moot’?

A. A basic point of law

B. A basic fact of case

C. Mock court for practice by students in general

D. Another name for magistrate’s court

 

Q. 150 The temporary release of a convicted prisoner from jail for a fixed period is called –

A. Bail

B. Parole

C. Acquittal

D. Discharge

 

Q. 151 The Railway authorities allowed a train to be over-crowded. In consequence, a legitimate passenger, Mr. X got his pocket picked. Choose appropriate answer-

A. Mr. X can sue the railway authorities for the loss suffered.

B. Mr. X cannot sue because he had given his consent to travel in an over-crowded train.

C. Mr. X cannot sue the railway authorities because there was no infringement of legal right and mere fact that the loss was caused does not give rise to a cause of action

D. None of the above

 

Q. 152 Choose the best option for the following statement:

The distinction between fraud and misrepresentation:

1. Fraud is more or less intentional wrong, whereas misrepresentation may be quite innocent.

2. In addition to rendering the contract voidable, is a cause of action in tort for damages.

Simple misrepresentation is not a tort but a person who rightfully rescinds a contract is

entitled to compensation for any damages which he has sustained through the nonfulfilment of the contract.

3. A person complaining of misrepresentation can be met with the defence that he has “the means of discovering the truth with ordinary diligence”. But expecting fraud by silence in other cases of fraud it is no defence that “the plaintiff had the means of discovering the truth by ordinary diligence”.

4. None of the above

 

A. 1 is correct

B. 1 & 2 are correct

C. 1, 2 & 3 are correct

D. Only 4 is correct

 

Q. 153 In a recent case a Supreme Court bench comprising of Justice Dipak Misra and Justice Prafulla C Pant held that the amount of maintenance to be awarded under Section 125 of CrPC cannot be restricted for the iddat period (three months) only as the inherent and fundamental principle behind Section 125. Also, it said that an order under Section 125 CrPC can be passed if a person, despite having sufficient means, neglects or refuses to maintain the wife.

A. Shamima Farooqui v. Shahid Khan

B. Mohd. Ahmad Khan v. Shah Bano Begum

C. Hamida Bano v. Abdul Rasheed

D. Abdul kadir v. Salima

 

Q. 154 Select the correct statements on Social Justice Bench constituted on social issue:

1. Constituted by Supreme Court on 3 December 2014

2. Started operation on 12 December 2014

3. The brainchild of Chief Justice of India H L Dattu

4. Two-judge bench to be headed by Justice Madan B Lokur

5. The other member is Justice U U Lalit

 

A. 1, 2 & 5 are correct

B. 1, 2 & 3 are correct

C. 1, 3 & 4 are correct

D. All are correct

 

Q. 155 Select the correct statements about 14th Finance Commission which submitted its report to the President:

1. It covers the period between 1 April 2015 and 31 March 2020.

2. The Commission headed by former RBI Governor Y V Reddy.

3. Article 280 of Constitution provides for appointment of Finance Commission

4. 1st and 13th Finance Commission was headed by K C Neogy & Dr Vijay Kelkar

respectively.

 

A. 1, 3 & 5 are correct

B. 1, 2 & 3 are correct

C. 1, 3 & 4 are correct

D. All are correct

 

Q. 156 Who administers oath of office to the Governor of a State?

A. President of India

B. Chief Justice of High Court of the respective state

C. Chief Justice of India

D. Speaker of State Assembly

 

Q. 157 Governor of a State can make Laws during recess of State Legislative Assembly through….

A. Act

B. Bills

C. Notification

D. Ordinance

 

Q. 158 Who called Indian Constitution as Quasi-Federal?

A. Austin

B. K. C. Wheare

C. H. M. Servai

D. Jennings

 

Q. 159 President of India exercises his powers…

A. EIther directly or through officers subordinate to him

B. Through ministers

C. Through Prime Minister

D. Through cabinet

 

Q. 160 Vote on accounts is meant for ….

A. Vote on the report of CAG

B. To meet unforeseen expenditure

C. Appropriating funds pending passing of budget

D. Budget

 

Questions: 161 – 168

W, X, Y and Z are four friends, who do not mind exchanging items. X has two chessboards each costing Rs. 500, and a record player. Z originally had a cycle and a walkman. Each cricket bat costs Rs. 700. Both W and Z got a cricket bat from Y. X gave his record player costing Rs. 2000 to Y. Z get a camera costing Rs. 1500 from W. The cycle of Z costs Rs 1000 and the walkman is for Rs. 700. Y had three cricket bats at the beginning and W had two cameras the total cost of which is Rs. 5000. X gave one of his chessboards to Z and took Z’s cycle. Z gave his walkman to W.

 

Q. 161 Total cost of materials Z had at the beginning was:

A. Rs. 1500

B. Rs. 1700

C. Rs. 1000

D. Rs. 2000

 

Q. 162 At the beginning who had the coolest items:

A. W

B. X

C. Y

D. Z

 

Q. 163 Who did not have a cricket bat after the exchange of items was over?

A. W

B. X

C. Y

D. Z

 

Q. 164 Who became the gainer be highest amount through exchange?

A. W

B. X

C. Y

D. Z

 

Q. 165 The person incurring the highest amount of financial loss through exchange lost an amount of:

A. Rs. 600

B. Rs. 1000

C. None

D. Rs. 500

 

Q. 166 The amount of price of all the things remaining with four persons lie between:

A. Rs. 800-Rs. 900

B. Rs. 10000-12000

C. Rs. 9000-Rs. 10000

D. Rs. 10000-Rs. 11000

 

Q. 167 Even after exchanges, an item of highest value remained in possession of:

A. W

B. X

C. Y

D. Z

 

Q. 168 Among the things exchanged, which one faced the highest exchange value in percentage term.

A. Cricket Bat

B. Record Player

C. Camera

D. Cycle

 

Questions: 169 – 173

Recently, the answers of a test held nationwide were leaked to a group of unscrupulous people. The investigative agency has arrested the mastermind and nine other people A, B, C, D, E, F, G, H and I in this matter. Interrogating them, the following facts have been obtained regarding their operation. Initially, the mastermind obtains the correct answer-key. All the others create their answerkey from one or two people who already possess the same. These people are called his/her sources. If the key to a question from both sources is identical, it is copied, otherwise left blank. If the person has only one source, he/she copies the source’s answer into his/her copy. Finally each person compulsorily replaces one of the answers (not a blank one) a wrong answer in his/her answer key. The paper contained 200 questions, so the investigative agency has ruled out the possibility of two or more of them introducing wrong answers to the same question. The investigative agency has a copy of the correct answer key and has tabulated the following data. This data represents question numbers

 

Q. 169 Which one of the following must have two sources?

A. A

B. B

C. C

D. D

 

Q. 170 How many people (excluding the mastermind) needed to make answer keys before C could make his answer key?

A. 2

B. 3

C. 4

D. 5

 

Q. 171 Both G and H were sources to ….

A. F

B. B

C. A

D. None of these

 

Q. 172 Which of the following statement is true?

A. A introduced the wrong answer to question 27

B. E introduced the wrong answer to question 46

C. F introduced the wrong answer to question 14

D. H introduced the wrong answer to question 46

 

Q. 173 Which of the following two groups of people has identical sources?

(I) A, D and G

(II) E and H

A. Only (I)

B. Only (II)

C. Neither (I) nor (II)

D. Both (I) and (II)

 

Questions: 174 – 178

In the following question, a group of numerals is given followed by four groups of symbol/letter combinations lettered (A), (B), (C), and (D). Numerals are to be coded as per the codes and conditions. You have to find out which of the combinations (A), (B), (C), and (D) is correct and indicate your answer accordingly.

Following conditions apply:

1. If the first digit as well as the last digit is odd, both are to be coded as ‘X’.

2. If the first digit as well as the last digit is even, both are to be coded as ‘$’.

3. If the last digit is ‘O’, it is to be coded as #.

 

Q. 174 Find out the combination for: 487692

A. $KEFM@

B. AKEFM@

C. AKFEM@

D. $KEFM$

 

Q. 175 Find out the combination for: 713540

A. X%BA#

B. E%BA#

C. E%BAR

D. None of the above

 

Q. 176 Find out the combination for: 765082

A. EFB#K@

B. XFBRK@

C. EFBRK@

D. None of the above

 

Q. 177 Find out the combination: 364819

A. *FAK%X

B. XFAK&M

C. *FAK%M

D. None of the above

 

Q. 178 Find out the combination for: 546839

A. XAFK*X

B. XAFK*M

C. BAFK*X

D. None of the above

 

Questions: 179 – 181

From the alternatives given below, choose the best option that correctly classifies the four sentences as a: 

F: Fact: If it relates to a known matter of direct observation, or an existing reality or something known to be true.

J: Judgement: If it is an opinion or estimate or anticipation of common sense or intention.

I: Inference: If it is logical conclusion or deduction about something based on the knowledge of facts.

Q. 179 A. Everyday social life is impossible without interpersonal relationships.

B. The root of many misunderstandings has been cited poor relations among individuals.

C. Assuming the above to be true, social life will be much better if people understand the importance of good interpersonal relations.

D. A study reveals that interpersonal relations and hence life in general can be improved with a little effort on the part of individuals.

A. FJIJ

B. JFIF

C. FIFJ

D. IFFJ

 

Q. 180 A. The cabinet minister definitely took the wrong step in giving the government contract. 

B. Under the circumstances, he had many other alternatives.

C. The Prime Minister is embarrassed due to the minister’s decision.

D. If he has put the government in jeopardy, the minister must resign.

A. JFFI

B. IFJI

C. FFJI

D. IFIJ

 

Q. 181 A. If democracy is to survive, the people must develop a sense of consumerism.

B. Consumerism has helped improve the quality of goods in certain countries.

C. The protected environment in our country is helping the local manufacturers.

D. The quality of goods suffers if the manufacturers take undue advantage of this.

A. IJFI

B. JFJI

C. IJJF

D. IFJI

 

Questions: 182 – 184

Question consists of five statements followed by options consisting of three statements put together in a specific order. Choose the best option which indicates a valid argument, that is, where the third statement is a conclusion drawn from the preceding two statements.

Q. 182 A. Traffic congestion increases carbon monoxide in the environment.

B. Increase in carbon monoxide is hazardous to wealth.

C. Traffic congestion is hazardous to health.

D. Some traffic congestion does not cause increase carbon monoxide.

E. Some traffic congestion is not hazardous to health.

A. CBA

B. BDE

C. CDE

D. BAC

 

Q. 183 A. MBAs are in great demand.

B. Samrat and Akshita are in great demand

C. Samrat is in great demand

D. Akshita is in great demand.

E. Samrat and Akshita are MBAs

A. ABE

B. ECD

C. AEB

D. EBA

 

Q. 184 A. All software companies employ knowledge workers.

B. Infotech employees are knowledge workers.

C. Infotech is a software company.

D. Some software companies employ knowledge workers.

E. Infotech employs only knowledge workers

A. ABC

B. ACB

C. CDB

D. ACE

 

Questions: 185 – 186

Read the following information carefully to choose best option for the question:

A. ‘L%M’ means that M is brother of L.

B. ‘LxM’ means that L is mother of M.

C. ‘L÷M’ means that L is the sister of M.

D. ‘L=M’ means that M is father of L.

 

Q. 185 Which of the following mean “I is the nephew of Q?”

1. Q%J=I

2. Q÷MxB%J

3. C÷I=B%Q

A. Only 3

B. Only 1

C. Only 2

D. None of the above

 

Q. 186 If ‘A$B’ means that A is the father of B, ‘A*B’ means that A is the mother of B, ‘A@B’ means that A is the wife of B, then which of the following means that M is the grand-mother of N? 

A. M*R$T@N

B. M*R@T@N

C. M*T$N@R

D. M*T@N$R

 

Questions: 187 – 189

The question contains two statements numbered I and II.

You have to decide whether the information provided in the statements are sufficient to answer the question. Read both the statement and give your answer as:

Answer (1) if the information in the statement I alone are sufficient to answer the questions.

Answer (2) if the information in the statement II alone are sufficient to answer the questions.

Answer (3) if the information either in the statement I alone or in statement II alone are sufficient to answer the questions.

Answer (4) if the information even in both I and II statements are not sufficient to answer the questions.

 

Q. 187 Can a democratic system operate without an effective opposition?

I. The opposition is indispensible.

II. A good politician always learns more from his opponents than from his frevent supporters.

A. 1

B. 2

C. 3

D. 4

 

Q. 188 Do habits make men’s life rigid?

I. It is out of habit that people envy others.

II. Men become slave of habits.

A. 1

B. 2

C. 3

D. 4

 

Q. 189 Does intelligence predict the child’s ability to learn?

I. Intelligence is unaffected by bad teaching or dull home environment.

II. Children from poor home backgrounds do not well in their school-work.

A. 1

B. 2

C. 3

D. 4

 

Questions: 190 – 192

In the following question some capital alphabets are written in a row, below them their coding has been given. In the question, a particular word has been coded in a particular manner using codes given below the capital letters. You have to understand the pattern of coding and have to answer the question asked subsequently.

 

Q. 190 If LONDON is 5c62z5 then EUROPE is?

A. wh7cdw

B. wh7z6v

C. bv76cb

D. wh76cb

 

Q. 191 If DASH is 2a84, then SMASH is?

A. 75U7t

B. eya84

C. 8zqe3

D. 8zqe4

 

Q. 192 If FASHION is z64t7w, then POSITION is?

A. z64e476c

B. z64e47c6

C. c674e46z

D. c674e4z6

 

Questions: 193 – 194

The question contains two statements and two conclusions numbered I and II. You have to take the two given statements to be true even if they seem to be at variance from commonly known facts and decide which of the given conclusion(s) logically follow(s) from the two given statements.

Answer (1) If only conclusion I follows.

Answer (2) If only conclusion II follows.

Answer (3) If neither I nor II follows.

Answer (4) If both I and II follows.

 

Q. 193 Statements:

I. Some players are singers

II. All singers all tall

Conclusions:

I. Some players are tall

II. All players are tall

A. 1

B. 2

C. 3

D. 4

 

Q. 194 Statements:

I. Some vegetables are fruits.

II. No fruit is black

Conclusions:

I. Some fruits are vegetables

II. No vegetable is black

A. 1

B. 2

C. 3

D. 4

 

Q. 195 Amit first goes in South direction, then he turns towards left and travels for some distance. After that he turns right moves certain distance. At last he turns left and travels again for some distance. Now, in which direction is he moving?

A. South

B. West

C. East

D. None of the above

 

Q. 196 There are six houses in a row. Mr Aalekh has Mr. Mishra and Mr. Iliyas as neighbours. Mr. Mritynjay has Mr. Sandeep and Mr. Nayak as neighbours. Mr Sandeep’s house is not next to Mr. Mishra or Mr. Iliyas and Mr. Nayak does not live next to Mr. Iliyas. Who are Mr. Mishra’s next door neighbours?

A. Mr. Aalekh and Mr. Mishra

B. Mr. Aalekh and Mr. Iliyas

C. Mr. Nayak and Mr. Aalekh

D. None of the above

 

Q. 197 From the word ‘LAPAROSCOPY’, how many independent meaningful English words can be made without changing the order of the letters and using each letter only once?

A. 1

B. 2

C. 3

D. 4

 

Q. 198 If Monday falls on 1st of October, which day will fall three days after the 20th in that month?

A. Monday

B. Tuesday

C. Saturday

D. Wednesday

 

Q. 199 In the word GRAPHOLOGIST, if 1st and 7th letters, 2nd and 9th letters, 3rd and 11th letters, 4th and 8th letters and 5th and 12th letters are mutually interchanged then which letter will be 6th letter from the left of 10th letter from the left side?

A. S

B. T

C. G

D. None of these

 

Q. 200 The son of M is the father of N and grandfather (Mother’s father) of R. S is the daughter of N and sister of B. On the basis of this information, how is M related to B?

A. Grandfather

B. Grandmother

C. Grandmother’s mother

D. None of the above

 

Answer Sheet 
Question 1 2 3 4 5 6 7 8 9 10
Answer B A D B B D B A D C
Question 11 12 13 14 15 16 17 18 19 20
Answer B D A D C D A D A D
Question 21 22 23 24 25 26 27 28 29 30
Answer A A A A B A C A C C
Question 31 32 33 34 35 36 37 38 39 40
Answer B B D A A B A A B D
Question 41 42 43 44 45 46 47 48 49 50
Answer B B A A C A A C C A
Question 51 52 53 54 55 56 57 58 59 60
Answer A B B B B A A A A D
Question 61 62 63 64 65 66 67 68 69 70
Answer D B C C A B C C B B
Question 71 72 73 74 75 76 77 78 79 80
Answer B B B A C D D C D B
Question 81 82 83 84 85 86 87 88 89 90
Answer B C C A B A A D B A
Question 91 92 93 94 95 96 97 98 99 100
Answer C D D A B A C B A B
Question  101 102 103 104 105 106 107 108 109 110
Answer D D B B C C D C C A
Question  111 112 113 114 115 116 117 118 119 120
Answer A B C B A A C C B B
Question 121 122 123 124 125 126 127 128 129 130
Answer B C A D B D C B A A
Question 131 132 133 134 135 136 137 138 139 140
Answer D D D A A A A A A C
Question 141 142 143 144 145 146 147 148 149 150
Answer A B C A A C C C C B
Question 151 152 153 154 155 156 157 158 159 160
Answer C C A D D B D B A C
Question 161 162 163 164 165 166 167 168 169 170
Answer B A B D C D A D B C
Question 171 172 173 174 175 176 177 178 179 180
Answer D C D D B C D A B A
Question 181 182 183 184 185 186 187 188 189 190
Answer B A C B D C C B D B
Question 191 192 193 194 195 196 197 198 199 200
Answer C A A A C C B B D D

 

CLAT 2014 Previous Year Paper

CLAT 2014

Q. 1  Fill in the blank by choosing the most appropriate option.

A vote of …………………… proposed at the end of the meeting.

A. thanks were

B. thank was

C. thanks had been

D. thanks was

 

Q. 2 Fill in the blank by choosing the most appropriate option.

During the recession many companies will …………………… lay off workers.

A. be forced to

B. have the force to

C. forcefully

D. be forced into

 

Q. 3 Fill in the blank by choosing the most appropriate option.

She has good ……………….. over the famous foreign languages.

A. expertise

B. command

C. control

D. authority

 

Q. 4 Fill in the blank by choosing the most appropriate option.

The Chairman pointed out in favour of the manager that the profitability of the industrial

plant had ………………………… since he took over the administration.

A. arisen

B. declined

C. added

D. increased

 

Q. 5 Fill in the blank by choosing the most appropriate option.

When the examinations were over, ………………………. went to Paris.

A. me and Rohan

B. I and Rohan

C. Rohan and me

D. Rohan and I

 

Q. 6 Fill in the blank by choosing the most appropriate option.

Let’s go for a walk, …………………………. ?

A. can we

B. shall we

C. can’t we

D. shouldn’t we

 

Q. 7 Fill in the blank by choosing the most appropriate option.

Had Anil been on time, he ………………………….. missed the train.

A. would not have been

B. had not

C. will not have

D. would not have

 

Q. 8 Fill in the blank by choosing the most appropriate option.

The most alarming fact is that infection is spreading …………………… the state and reaching villages and small towns.

A. over

B. across

C. far

D. from

 

Q. 9 Fill in the blank by choosing the most appropriate option.

In big cities people are cut …………………… from nature.

A. off

B. down

C. away

D. out

 

Q. 10 Fill in the blank by choosing the most appropriate option.

The dissidents ……………………. a great problem in every political party.

A. give

B. cause

C. pose

D. hold

 

Q. 11 The sentences given in each question, when properly sequenced, form a coherent paragraph. Each sentence is labelled with a letter. Choose the most logical order of sentences from among the given choices to construct a coherent paragraph.

(a) Payment for imports and exports is made through a system called foreign exchange. The value of the money of one country in relation to the money of other countries is agreed upon.

(b) The rates of exchange vary from time to time.

(c) For example, an American dollar or a British pound sterling is worth certain amounts in the money of other countries.

(d) Sometimes a US dollar is worth 60 rupees in India.

A. abcd

B. bacd

C. acbd

D. cabd

 

Q. 12 The sentences given in each question, when properly sequenced, form a coherent paragraph. Each sentence is labelled with a letter. Choose the most logical order of sentences from among the given choices to construct a coherent paragraph.

(a) When a dictionary is being edited, a lexicographer collects all the alphabetically arranged citation slips for a particular word.

(b) The moment a new word is coined, it usually enters the spoken language.

(c) The dictionary takes note of it and makes a note of it on a citation slip.

(d) The word then passes from the realm of hearing to the realm of writing.

 

A. abcd

B. acbd

C. bacd

D. bcad

 

Q. 13 The sentences given in each question, when properly sequenced, form a coherent

paragraph. Each sentence is labelled with a letter. Choose the most logical order of sentences from among the given choices to construct a coherent paragraph.

(a) The impression that corruption is a universal phenomenon persists and the people do not co-operate in checking this evil.

(b) So there is hardly anything that the government can do about it now.

(c) It is regrettable that there is a widespread corruption in the country at all levels.

(d) Recently several offenders were brought to book, but there were not given deterrent punishment.

A. cdab

B. adbc

C. adcb

D. cbad

 

Q. 14 The sentences given in each question, when properly sequenced, form a coherent

paragraph. Each sentence is labelled with a letter. Choose the most logical order of sentences from among the given choices to construct a coherent paragraph.

(a) In all social affairs convention prescribes more or less generally accepted rules of behaviour.

(b) Of course, there is nothing absolute about conventions.

(c) They vary from country to country, from age to age.

(d) Convention has a necessary part to play in the life of everyone.

A. abcd

B. adbc

C. dacb

D. dabc

 

Q. 15 The sentences given in each question, when properly sequenced, form a coherent

paragraph. Each sentence is labelled with a letter. Choose the most logical order of sentences from among the given choices to construct a coherent paragraph.

(a) In fact, only recently there have been serious studies to find out how many of us actually have nightmares.

(b) Now that is changing.

(c) The study of nightmares has been curiously neglected.

(d) While results so far are inconclusive, it seems fair to say that at least half the population has occasional nightmares.

A. cadb

B. abdc

C. adcb

D. cbad

 

Q. 16 Given below are a few foreign language phrases which are commonly used. Choose the correct meaning for each of the phrases.

Ex officio

A. By virtue of previously held position

B. Former official

C. By virtue of office

D. Outside the office

 

Q. 17 Given below are a few foreign language phrases which are commonly used. Choose the correct meaning for each of the phrases.

Ultra Vires

A. Within powers

B. Full powers

C. Near powers

D. Beyond powers

 

Q. 18 Given below are a few foreign language phrases which are commonly used. Choose the correct meaning for each of the phrases.

Quid pro quo

A. Something for nothing

B. Something for something

C. Everything for something

D. Something for everything

 

Q. 19 Given below are a few foreign language phrases which are commonly used. Choose the correct meaning for each of the phrases.

Inter vivos

A. between the living

B. among the living and the dead

C. between the dead

D. among the dead and the living

 

Q. 20 Given below are a few foreign language phrases which are commonly used. Choose the correct meaning for each of the phrases.

Corpus juris

A. body of judges

B. group of jurists

C. body of law

D. knowledge of law

 

Q. 21 Which of the following spellings is correct?

A. Concensus

B. Consencus

C. Consenssus

D. Consensus

 

Q. 22 Which of the following spellings is correct?

A. Procede

B. Proceed

C. Proceede

D. Proced

 

Q. 23 Which of the following spellings is correct?

A. Accommodate

B. Acommodate

C. Accomodate

D. Acomodate

 

Q. 24 Which of the following spellings is correct ‘for a page at the booking of a book’?

A. Foreward

B. Forword

C. Forworde

D. Foreword

 

Q. 25 Which of the following spellings is correct?

A. Arguement

B. Argument

C. Arguemant

D. Arguemint

 

Q. 26 Choose the explanation that best reflects the spirit of the idiom/proverb/phrase given in each question.

To make clean breast of:

A. To tell the truth about something.

B. To gain prominence.

C. To destroy before it blooms.

D. To praise oneself.

 

Q. 27 Choose the explanation that best reflects the spirit of the idiom/proverb/phrase given in each question.

A man of straw:

A. A creditable man.

B. A very active man.

C. A man of no or little substance.

D. An unreasonable man.

 

Q. 28 Choose the explanation that best reflects the spirit of the idiom/proverb/phrase given in each question.

A wild-goose chase:

A. A wise search.

B. A fruitful search.

C. A worthwhile hunt.

D. A futile pursuit.

 

Q. 29 Choose the explanation that best reflects the spirit of the idiom/proverb/phrase given in each question.

Put on the market:

A. To offer for sale.

B. Alongside the market.

C. Already purchased.

D. None of the above.

 

Q. 30 Choose the explanation that best reflects the spirit of the idiom/proverb/phrase given in each question.

To meet someone halfway:

A. To show that you are prepared to strain your relationship with someone.

B. To compromise with someone.

C. Confrontation.

D. Incongruity

 

Questions: 31 – 40

Passage-1:

The spread of education in society is at the foundation of success in countries that are latecomers to development. In the quest for development, primary education is absolutely essential because it creates the base. But higher education is just as important, for it provides the cutting edge. And universities are the life-blood of higher education. Islands of excellence in professional education, such as Indian Institutes of Technology (IITs) and Indian Institutes of Management (IIMs), are valuable complements but cannot be substitutes for universities which provide educational opportunities for people at large. There can be no doubt that higher education made a significant contribution to economic development, social progress and political democracy in independent India. It is a source of dynamism for the economy. It has created social opportunities for people. It has fostered the vibrant democracy in our polity. It has provided a beginning for the creation of a knowledge society. But it would be a mistake to focus on its strengths alone. It has weaknesses that are a cause for serious concern. There is, in fact, a quite crisis in higher education in India that runs deep. It is not yet discernible simply because there are pockets of excellence, an enormous reservoir of talented young people and an intense competition in the admissions process. And, in some important spheres, we continue to reap the benefits of what was shown in higher education 50 years ago by the founding fathers of the Republic. The reality is that we have miles to go. The proportion of our population, in the age group 18-24, that enters the world of higher education is 7 per cent, which is only one-half the average of Asia. The opportunities for higher education, in terms of the number of places in universities, are simply not enough in relation to our needs. What is more, the quality of higher education in most of our universities require substantial improvement. It is clear that the system of higher education in India faces serious challenges. It needs a systematic overhaul, so that we can educate much larger numbers without diluting academic standards.This is imperative because the transformation of economy and society in the 21st century would depend, in significant part, on the spread and quality of education. It is only an inclusive society that can provide the foundations for a knowledge society. The challenges that confront higher education in India are clear. It needs a massive expansion of opportunities for higher education, to 1500 universities nationwide, that would India to attain a gross enrollment ratio of at least 15 per cent by 2015. It is just as important to raise the average quality of higher education in every sphere. At the same time, it is essential to create institutions that are exemplars of excellence at par with the best in the world. In the pursuit of these objectives, providing people with access to higher education inn a socially inclusive manner is imperative. The realisation of these objectives, combined with access, would not only develop the skills and capabilities we need for the economy but would also help transform India into a knowledge economy and society.

 

Q. 31 The principal focus of the passage is:

A. Primary education.

B. Intermediate education.

C. Higher education.

D. Entire education system.

 

Q. 32 The style of the passage can be best described as:

A. Academic

B. Critical and analytical

C. Comparative

D. None of the above

 

Q. 33 What kind of society can provide the foundation for a knowledge society?

A. Elite society

B. Contracted society

C. Exclusive society

D. Inclusive society

 

Q. 34 According to the passage, which one of the following is INCORRECT?

A. There are no quality institutes providing excellent professional education in India

B. Not many people go for higher education in India

C. Education is the basis of success

D. All the above options are correct

 

Q. 35 According to the passage, the current state of affairs of higher education in India is:

A. Satisfactory

B. Excellent, and there is no need of any expansion of opportunities for higher education

C. Not good enough, and there is a need of expansion of opportunities for higher education, besides creating institutions and universities that are models of excellence

D. Not explained in the passage

 

Q. 36 According to the passage, which of the following is not a challenge that confronts higher education in India?

A. Expanding opportunities for higher education

B. Creating institutions and universities that are exemplars of excellence.

C. Substantial improvement in the quality of higher education in most of our universities.

D. Getting into World University Rankings

 

Q. 37  According to the passage, which of the following is CORRECT?

A. Primary education is very important.

B. Universities are the life-blood of higher education.

C. Transformation of economy and society in the 21st century would depend, in significant part, on the spread and the quality of education among our people, particularly in the sphere of higher education.

D. All the above propositions are correct

 

Q. 38 Should the entire university system in India be modeled on premier institutes, such as IITs and IIMs, providing professional education?

A. Yes

B. For sure

C. No

D. The passage is silent on the question

 

Q. 39 What is the antonym of the expression ‘cutting edge’ ?

A. Conventional

B. Avant-garde

C. Advanced

D. Contemporary

 

Q. 40 What is the meaning of the word ‘discernible’?

A. Unobtrusive

B. Noticeable

C. Unremarkable

D. Inconspicious

 

Q. 41 The next number in the sequence is: 19, 29, 37, 43, ………………….

A. 45

B. 47

C. 50

D. 53

 

Q. 42 An unknown man is found murdered. The corpse has one gold plated tooth, right ear is pierced and a healed fracture of left hand thumb. A man with these characteristics is reported missing. What are the chances (probability) of the corpse being the missing man? (Given the occurrence of the gold-plated teeth in the area, 1 in 5000, left hand thumb fractures 1 in 20000 and of right ear pierces 1 in 100).

A. 1 in 1,000

B. 1 in 1,000,000

C. 1 in 1,000,000,000

D. 1 in 10,000,000,000

 

Q. 43 Value of “A” in the expression, 5 = 12 x 10 ÷ 120/240 = A x 10, is:

A. 11

B. 24.5

C. 34

D. 6.5

 

Q. 44 The Least Common Multiple (L.C.M.) of 0.12, 9.60 and 0.60 is:

A. 9.60

B. 0.12

C. 0.6

D. None of these

 

Q. 45 There are 30 boys and 40 girls in a class. If the average age of boys is 10 yr. and average age of girls is 8 yr., then the average age of the whole class is:

A. 8 yr.

B. 8.86 yr.

C. 8.2 yr.

D. 9 yr.

 

Q. 46 A person spends 1/3 part of his income on food, 1/4 part on house rent and remaining Rs. 630 on other items. The house rent is:

A. Rs. 504

B. Rs. 1512

C. Rs. 378

D. None of these

 

Q. 47 A person covers a certain distance by car at a speed of 30 km/h and comes back at a speed of 40 km/h. The average speed during the travel is:

A. 34.3 km/h

B. 35 km/h

C. 37.5 km/h

D. 32.8 km/h

 

Q. 48 An employer reduces the number of employees in the ratio 8 : 5 and increases wages in the ratio 7 : 9. Therefore, the overall wages bill is:

A. Increased in the ratio 45 : 56

B. Decreased in the ratio 56 : 45

C. Increased in the ratio 13 : 17

D. Decreased in the ratio 72 : 35

 

Q. 49 Father is 3 yr. older than the mother and the mother’s age is now twice the daughter’s age. If the daughter is 20 yr. old now, then the father’s age when the daughter was born in: 

A. 20 yr.

B. 40 yr.

C. 43 yr.

D. 23 yr.

 

Q. 50 If 80% of A = 20% of B and B = 5x% of A, then the value of x is:

A. 75

B. 80

C. 90

D. 85

 

Q. 51 A mixture of 40 L of alcohol and water contains 10% water. How much water should be added to this mixture, so that the new mixture contains 20% water?

A. 9L

B. 5L

C. 7L

D. 6L

 

Q. 52 ‘A’ can do a piece of work in 20 days and ‘B’ can do the same work in 15 days. How long will they take to finish the work, if both work together?

A. 13 1/2 days

B. 10 days

C. 8 4/7 days

D. 17 1/2 days

 

Q. 53 A man can row 5 km/h in still water. If the speed of the current is 1 km/h, it takes 3 h more in upstream than in the downstream for the same distance. The distance is:

A. 36 km

B. 24 km

C. 20 km

D. 32 km

 

Q. 54 A starts a business with Rs. 5000 and B joins the business 5 months later with an

investment of Rs. 6000. After a year, they earn a profit of Rs. 34000. Find the shares of A and B in the profit amount depending on their individual investment.

A. Rs. 20000, Rs. 14000

B. Rs. 16000, Rs. 16000

C. Rs. 14000, Rs. 20000

D. None of these

 

Q. 55 A farmer has some hens and some goats. If the total number of animal heads is 80 and the total number of animal feet is 200, what is the total number of goats?

A. 40

B. 60

C. 20

D. Cannot be determined

 

Q. 56 A square field has its area equal to 324 m^2. The perimeter of the field is:

A. 36m

B. 72m

C. 18m

D. 6561m

 

Q. 57 A closed metal box measure 30 cm x 20 cm x 10 cm. Thickness of the metal is 1 cm. The volume of the metal required to make the box is:

A. 1041 cm^3

B. 6000 cm^3

C. 4536 cm^3

D. 1968 cm^3

 

Q. 58 The difference between the simple interest and the compound interest (compounded annually) on Rs. 2000 for 2 yrs. at 8% per annum will be:

A. Rs. 10

B. Rs. 20

C. Rs. 13

D. Rs. 25

 

Q. 59 A dealer marked his goods 20% above the cost price and allows a discount of 10%. Then the gain percent is:

A. 2%

B. 4%

C. 6%

D. 8%

 

Q. 60 A man went to the Reserve Bank of India with Rs. 2000. He asked the cashier to give him Rs. 10 and Rs. 20 notes only in return. The man got 150 notes in all. How many notes of Rs. 10 did he receive?

A. 100

B. 150

C. 50

D. 70

 

Q. 61 Which of the following Judges of the Supreme Court of India is famously known as the ‘Green Judge’?

A. Justice VR Krishna Iyyar

B. Justice PN Bhagwati

C. Justice Kuldeep Singh

D. Justice BN Kirpal

 

Q. 62 Law Day is observed on:

A. 26th January

B. 26th May

C. 15th August

D. 26th November

 

Q. 63 In which year, Defence Research & Development Organisation (DRDO) was formed by the amalgamation of the Technical Development Establishment (TDE) of the Indian Army and the Directorate of Technical Development & Production (DTDP) with the Defence Science Organisation (DSO):

A. 1955

B. 1958

C. 1959

D. 1963

 

Q. 64 The Reserve Bank of India was established in the year:

A. 1858

B. 1935

C. 1947

D. 1950

 

Q. 65 Lord Buddha’s image is sometimes shown with the hand gesture, called ‘Bhumisparsha Mudra’. It signifies:

A. Buddha’s calling of the Earth to watch over Mara and to prevent Mara from disturbing his meditation.

B. Buddha’s calling of the Earth to witness his purity and chastity despite the temptations of Mara.

C. The gesture of debate or discussion/argument.

D. Both (A) and (B) are correct.

 

Q. 66 Which of the following statements is INCORRECT about the Fundamental Duties under the Constitution of India? It shall be the duty of every citizen of India –

A. To uphold and protect the sovereignty, unity and integrity of India.

B. To strive towards excellence in all spheres of individual and collective activity so that the nation constantly rises to higher levels of endeavour and achievements.

C. Who is a parent or guardian to provide opportunities for education to his child or, as the case may be, ward between the age of six and fourteen years.

D. To vote in public elections.

 

Q. 67 Which one of the following scripts of ancient India was mostly written from right to left?

A. Nandnagari

B. Brahmi

C. Kharoshti

D. Sharada

 

Q. 68 During the time of which Mughal Emperor did the East India Company establish its first factory in India?

A. Akbar

B. Jahangir

C. Shahjahan

D. Aurangzeb

 

Q. 69 Who, among the following, first translated the Bhagawat Gita into English?

A. Charles Wilkins

B. Alexander Cunningham

C. William Jones

D. James Prinsep

 

Q. 70 Match List I with List II and select the correct answer using the codes given below the lists: 

A. a-4; b-3; c-1

B. a-1; b-4; c-3

C. a-2; b-1; c-3

D. a-1; b-4; c-2

 

Q. 71 Match List I with List II and select the correct answer using the codes given below the lists:

A. a-2; b-3; c-1; d-4

B. a-4; b-3; c-2; d-1

C. a-1; b-2; c-4; d-3

D. a-2; b-3; c-4; d-1

 

Q. 72 Who was not a Chief Justice of India?

A. Justice M Patanjali Sastri

B. Justice KN Wanchoo

C. Justice HR Khanna

D. Justice MN Venkatachaliah

 

Q. 73 Who was the first Attorney-General for India?

A. CK Daphtary

B. MC Setalvad

C. Niren De

D. LN Sinha

 

Q. 74 Under the leadership of Mahatma Gandhi, the Civil Disobedience Movement, launched in 1930, started from? 

A. Sabarmati

B. Dandi

C. Sevagram

D. Champaran

 

Q. 75 Match List I with List II and select the correct answer using the codes given below the lists: 

A. a-1; b-2; c-3; d-4

B. a-4; b-3; c-2; d-1

C. a-2; b-4; c-3; d-1

D. a-1; b-4; c-3; d-2

 

Q. 76 Which of the following planets has the maximum number of natural satellites?

A. Earth

B. Mars

C. Jupiter

D. Saturn

 

Q. 77 In India, the interest rate on savings accounts in all the nationalised commercial banks is fixed by?

A. Union Ministry of Finance

B. Union Finance Commission

C. Indian Banks’ Association

D. None of the above

 

Q. 78 Solvents are the substances used to dissolve other substances. Consider the following substances : (1) Water (2) Ether (3) Toluene (4) Chloroform (5) Ethanol

Which of the above can be used as solvents?

A. 1 and 2

B. 2 and 3

C. 3, 4 and 5

D. All of these

 

Q. 79 Viruses are parasitic, having DNA/RNA ; but, they can be crystallised and lack respiration. Therefore, they are treated as:

A. Living beings

B. Non-living beings

C. Both living and non-living beings

D. None of the above

 

Q. 80 Who is the winner of the coveted Dadasaheb Phalke Award for the year 2013?

A. Javed Akhtar

B. Gulzar

C. AR Rahman

D. Indeevar

 

Q. 81 Who among the following has been recently appointed as brand ambassador for Central Reserve Police Force (CRPF):

A. Aamir Khan

B. Mahendra Singh Dhoni

C. MC Mary Kom

D. Sania Mirza

 

Q. 82 Till date (2014), how many people have been awarded the Bharat Ratna Award?

A. 40

B. 41

C. 42

D. 43

 

Q. 83 Which country Malala Yousafzai belongs to?

A. Pakistan

B. Afghanistan

C. The USA

D. England

 

Q. 84 Who recently became the first woman chief of the State Bank of India?

A. Arundhati Bhattacharya

B. Shubhalakshmi Panse

C. Vijaylakshmi Iyer

D. Chanda Kochhar

 

Q. 85 Who is the Chairman of the 14th Finance Commission?

A. Dr. M Govinda Rao

B. Dr. Vijay Kelkar

C. Dr. Y V Reddy

D. Dr. Raghuram Rajan

 

Q. 86 The Chairperson of the Seventh Pay Commission is:

A. Justice BN Srikrishna

B. Justice DK Jain

C. Justice AP Shah

D. Justice AK Mathur

 

Q. 87 Which one of the following is not correctly matched?

United Nations Specialised Agency and their headquarters:

A. International Civil Aviation Organisation – Montreal

B. World Trade Organisation – Geneva

C. United Nations Industrial Development Organisation – Brussels

D. International Fund for Agricultural Development – Rome

 

Q. 88 The First Five Year Plan of India was based on:

A. John W Miller Model

B. PC Mahalanobis Model

C. Gadgil Yojana

D. Herrod-Domar Model

 

Q. 89 Which one of the following measures is NOT likely to aid in improving India’s balance of payment position?

A. Promotion of import substitution policy

B. Devaluation of rupee

C. Imposition of higher tariff on imports

D. Levying the higher duty on exports

 

Q. 90 Find out the correct chronological sequence of the following persons’ visit to India at one time or another:

(1) Fa-Hien (2) I-Tsing (Yijing) (3) Megasthanese (4) Hiuen-Tsang

A. 1, 3, 2, 4

B. 1, 3, 4, 2

C. 3, 1, 4, 2

D. 3, 1, 2, 4

 

Q. 91 Which was the first newspaper to be published in India?

A. Bombay Samachar

B. The Hindu

C. Bengal Chronicle

D. Bengal Gazette

 

Q. 92 The change in the colour of stars is linked to:

A. Variation in their surface temperature

B. Variation in their distance from the earth

C. Fluctuations in their composition and size

D. Irregular absorption or scattering in earth’s atmosphere

 

Q. 93 Which one of the following does NOT remain to be a planet now?

A. Neptune

B. Uranus

C. Pluto

D. Venus

 

Q. 94 Who was appointed as the 23rd Governor of the Reserve Bank of India?

A. KC Chakrabarty

B. Urijit Patel

C. Raghuram Rajan

D. D Subbarao

 

Q. 95 Which one of following offices is held during the pleasure of the President of India?

A. Vice-President

B. Governor of a State

C. Chief Justice of India

D. Comptroller and Auditor General of India

 

Q. 96 Who was the first winner of the prestigious Jnanpith Award?

A. Tarasankar Bandyopadhyay

B. Kuppali Venkatappagowda Puttappa

C. G Sankara Kurup

D. Umashankar Joshi

 

Q. 97 Match List I with List II and choose the correct answer:

List I

(a) Visakhadatta (b) Varahamihira (c) Sushruta (d) Brahmagupta

List II

(1) Surgery (2) Drama (3) Astronomy (4) Mathematics

A. a-1; b-3; c-4; d-2

B. a-2; b-1; c-3; d-4

C. a-2; b-3; c-1; d-4

D. a-3; b-4; c-1; d-2

 

Q. 98 Who was among the following honoured with Arjun Award in Chess for the year 2013? 

A. Kavita Chahal

B. Parimarjan Negi

C. Ronjan Sodhi

D. Abhijeet Gupta

 

Q. 99 Economic growth rate projected by the IMF for India in the fiscal year 2014-15 is:

A. 5.4 per cent

B. 5.5 per cent

C. 5.6 per cent

D. 5.7 per cent

 

Q. 100 Who was sworn in as the Prime Minister of Italy on 22nd February 2014?

A. Oleksandr Turchynov

B. Matteo Renzi

C. Enrico Letta

D. Giorgio Napolitano

 

Q. 101 Which one of the following is the highest peacetime gallantry award of India?

A. Param Vir Chakra

B. Ashok Chakra

C. Maha Vir Chakra

D. Kirti Chakra

 

Q. 102 Which one of the following gases is lighter than air?

A. Carbon dioxide

B. Chlorine

C. Oxygen

D. Hydrogen

 

Q. 103  Name the scientist who is known as the father of modern genetics:

A. Jean-Baptiste Lamarck

B. Hugo de Vries

C. Gregor Johann Mendel

D. Charles Darwin

 

Q. 104 Which year was designated by the United Nations as International Women’s Year?

A. 1974

B. 1975

C. 1976

D. 1977

 

Q. 105 Mention the correct abbreviation for ATM:

A. Automated Teller Machine

B. Any Time Money

C. All Time Money

D. Auto-limited Teller Machine

 

Q. 106 General Election is being held in India from 7 April to 12 May 2014 to constitute:

A. 14th Lok Sabha

B. 15th Lok Sabha

C. 16th Lok Sabha

D. 17th Lok Sabha

 

Q. 107 Mention the name of the current Chief Justice of India:

A. Justice Altamas Kabir

B. Justice P Sathasivam

C. Justice RM Lodha

D. Justice HL Dattu

 

Q. 108 The 9th Ministerial Conference of the WTO, held during 3 December – 6 December 2013, was concluded at:

A. Geneva, SwitzerlandK

B. Kuala Lumpur, Malaysia

C. Warsaw, Poland

D. Bali, Indonesia

 

Q. 109 Nobel Prize in Literature for the year 2013 was awarded to:

A. Mo Yan

B. James E Rothman

C. Lars Peter Hansen

D. Alice Munro

 

Q. 110 Which one of the following satellites was successfully launched by the ISRO’s Polar Satellite launch Vehicle (PSLV-C24) on 4th April 2014?

A. IRNSS-1B

B. GSAT-14

C. INSAT-3D

D. SARAL

 

Questions: 111 – 113

Read the following information carefully and answer the questions given below: 

Five friends Satish, Rajesh, Rehman, Rakesh, and Vineet – each presents one paper to their class on Physics, Zoology, Botany, English, or Geology – one day a week; Monday through Friday. 

(i) Vineet does not present English and does not give his presentation on Tuesday.

(ii) Rajesh makes the Geology presentation, and does not do it on Monday or Friday.

(iii) The Physics presentation is made on Thursday.

(iv) Rehman makes his presentation, which is not on English, on Wednesday.

(v) The Botany presentation is on Friday, and not by Rakesh.

(vi) Satish makes his presentation on Monday.

 

Q. 111 What day is the English presentation made?

A. Friday

B. Monday

C. Tuesday

D. Wednesday

 

Q. 112 What presentation does Vineet do?

A. English

B. Geology

C. Physics

D. Botany

 

Q. 113 What day does Rakesh make his presentation on?

A. Monday

B. Tuesday

C. Wednesday

D. Thursday

 

Q. 114 Each question contains a statement on relationship and a question regarding relationship based on the statement. Choose the correct option.

Pointing to a photograph, a man said, “I have no brother or sister but that man’s father is

my father’s son”. Whose photograph was that?

A. His father’s

B. His own

C. His son’s

D. His nephew’s

 

Q. 115 Each question contains a statement on relationship and a question regarding relationship based on the statement. Choose the correct option.

Ranjan introduces Abhay as the son of the only brother of his father’s wife. How is Abhay related to Ranjan?

A. Son

B. Brother

C. Cousin

D. Uncle

 

Q. 116 Each question contains a statement on relationship and a question regarding relationship based on the statement. Choose the correct option.

Pointing to a lady on the stage, Bhumika said, “She is the sister of the son of the wife of my husband. How is the lady related to Bhumika?

A. Cousin

B. Sister-in-law

C. Sister

D. Daughter

 

Q. 117 Each question contains a statement on relationship and a question regarding relationship based on the statement. Choose the correct option.

Pinky, who is Victor’s daughter, says to Lucy, “Your mother Rosy is the younger sister of my father, who is the third child to Joseph”. How is Joseph related to Lucy?

A. Father-in-law

B. Father

C. Maternal Uncle

D. Grandfather

 

Q. 118 Each question contains a statement on relationship and a question regarding relationship based on the statement. Choose the correct option.

Pramod told Vinod, ‘Yesterday I defeated the only brother of the daughter of my grandmother”. Whom did Pramod defeat?

A. Father

B. Son

C. Father-in-law

D. Cousin

 

Questions: 119 – 123

Read the information given below to answer the questions.

(i) In a family of six persons, there are people from three generations. Each person has separate profession and also each one likes different colours. There are two couples in the family.

(ii) Charan is a CA and his wife neither is a doctor nor likes green colour.

(iii) Engineer likes red colour and his wife is a teacher.

(iv) Vanita is mother-in-law of Namita and she likes orange colour.

(v) Mohan is grandfather of Raman and Raman, who is a principal, likes black colour.

(vi) Sarita is granddaughter of Vanita and she likes blue colour, Sarita’s mother likes white colour.

 

Q. 119 Who is an Engineer?

A. Sarita

B. Vanita

C. Namita

D. Mohan

 

Q. 120 What is the profession of Namita?

A. Doctor

B. Engineer

C. Teacher

D. Cannot be determined

 

Q. 121 Which of the following is the correct pair of two couples?

A. Mohan – Vanita and Charan – Sarita

B. Vanita – Mohan and Charan – Namita

C. Charan – Namita and Raman – Sarita

D. Cannot be determined.

 

Q. 122 How many ladies are there in the family?

A. Two

B. Three

C. Four

D. None of these

 

Q. 123 Which colour is liked by CA?

A. White

B. Blue

C. Black

D. None of these

 

Q. 124 Two buses start from the opposite points of a main road, 150 kms apart. The first bus runs for 25 kms and takes a right turn and then runs for 15 kms. It then turns left and runs for another 25 kms and takes the direction back to reach the main road. In the meantime, due to a minor breakdown, the other bus has run only 35 kms along the main road. What would be the distance between the two buses at this point?

A. 65 kms

B. 75 kms

C. 80 kms

D. 85 kms

 

Q. 125 ‘G’, ‘H’, ‘I’, ‘J’, ‘K’, ‘L’, ‘M’, ‘N’ are sitting around a round table in the same order for group discussion at equal distances. Their positions are clock wise. If ‘M’ sits in the north, then what will be the position of ‘J’?

A. East

B. South-East

C. South

D. South-West

 

Q. 126 Roshan, Vaibhav, Vinay and Sumit are playing cards. Roshan and Vaibhav are partners. Sumit faces towards North. If Roshan faces towards West, then who faces towards South? 

A. Vinay

B. Vaibhav

C. Sumit

D. Data is inadequate

 

Q. 127 Five boys are standing in a row facing East. Pavan is to the left of Tvan, Vipin, Chavan. Tavan, Vipin, Chavan are to the left of Nakul. Chavan is between Tvan and Vipin. If Vipin is fourth from the left, then how far is Tavan from the right?

A. First

B. Second

C. Third

D. Fourth

 

Q. 128 One morning after sunrise, Suraj was standing facing a pole. The shadow of the pole fell exactly to his right. Which direction was Suraj facing?

A. West

B. South

C. East

D. Data is inadequate

 

Questions: 129 – 130

Read the information given below to answer the questions. Diana is three times older than Jackson; Edward is half the age of Stephen. Jackson is older than Edward.

 

Q. 129 Which one of the following can be inferred?

A. Jackson is older than Stephen.

B. Diana is older than Stephen.

C. Diana may be younger than Stephen.

D. None of the above.

 

Q. 130 Which one of the following information will be sufficient to estimate Diana’s age?

A. Edward is 10 year old.

B. Both Jackson and Stephen are older than Edward by the same number of years.

C. Both A and B above

D. None of the above

 

Q. 131 Study the sequence/pattern of letters or numbers carefully to work out the pattern on which it is based, and answer what the next item in the sequence must be. For example, the sequence ‘A, C, E, G, ?’ has odd numbered letters of alphabet; therefore, the next item must be ‘I’.

0, 3, 8, 15, ?

A. 24

B. 26

C. 35

D. None

 

Q. 132 Study the sequence/pattern of letters or numbers carefully to work out the pattern on which it is based, and answer what the next item in the sequence must be. For example, the sequence ‘A, C, E, G, ?’ has odd numbered letters of alphabet; therefore, the next item must be ‘I’.

8, 16, 28, 44, ?

A. 60

B. 64

C. 62

D. 66

 

Q. 133 Study the sequence/pattern of letters or numbers carefully to work out the pattern on which it is based, and answer what the next item in the sequence must be. For example, the sequence ‘A, C, E, G, ?’ has odd numbered letters of alphabet; therefore, the next item must be ‘I’.

0, 6, 24, 60, 120, 210, ?

A. 290

B. 240

C. 336

D. 504

 

Q. 134 POQ, SRT, VUW, ?

A. XYZ

B. XZY

C. YZY

D. YXZ

 

Q. 135 A1, C3, F6, J10, 015, ?

A. U21

B. V21

C. T20

D. U20

 

Q. 136 Two words, which have a certain relation, are paired. Select a correct option to substitute the question mark so as to make a similar relational pair with the word given after double colon (::).

Bow : Arrow :: Pistol : ?

A. Gun

B. Shoot

C. Rifle

D. Bullet

 

Q. 137 Two words, which have a certain relation, are paired. Select a correct option to substitute the question mark so as to make a similar relational pair with the word given after double colon (::).

Eye : Wink :: Heart : ?

A. Throb

B. Move

C. Pump

D. Respirate

 

Q. 138 Two words, which have a certain relation, are paired. Select a correct option to substitute the question mark so as to make a similar relational pair with the word given after double colon (::).

Ocean : Water :: Glacier : ?

A. Cooling

B. Cave

C. Ice

D. Mountain

 

Q. 139 Two words, which have a certain relation, are paired. Select a correct option to substitute the question mark so as to make a similar relational pair with the word given after double colon (::).

Prima Facie : On the first view :: In pari delicto : ?

A. Both parties equally at fault

B. While litigation is pending

C. A remedy for all disease

D. Beyond powers

 

Q. 140 Two words, which have a certain relation, are paired. Select a correct option to substitute the question mark so as to make a similar relational pair with the word given after double colon (::).

Delusion : hallucination :: Chagrin : ?

A. Illusion

B. Ordered

C. Cogent

D. Annoyance

 

Q. 141 Each question comprises two statements (numbered as I and II). You have to take the statements as true even if they seem to be at variance with commonly known facts. Read all the conclusions and then decide which of the given conclusions logically follow from the given statements, disregarding commonly known facts.

(I) All contracts are agreements.

(II) All agreements are accepted offers.

Which of the following derivations is correct?

A. All accepted offers are contracts

B. All agreements are contracts

C. All contracts are accepted offers

D. None of the above

 

Q. 142 Each question comprises two statements (numbered as I and II). You have to take the statements as true even if they seem to be at variance with commonly known facts. Read all the conclusions and then decide which of the given conclusions logically follow from the given statements, disregarding commonly known facts.

(I) Some beautiful women are actresses.

(II) All actresses are good dancers.

Which of the following derivations is correct?

A. Some beautiful women are good dancers.

B. All good dancers are actresses

C. Both (1) and (2)

D. None of the baove

 

Q. 143 Two statements are given below followed by two conclusions (I and II). You have to consider the two statements to be true even if they seem to be at variance with commonly known facts. You have to decide which of the conclusions, if any, follow from the given statements.

Statements:

Some books are magazine.

Some magazines are novels.

Conclusions:

(I) Some books are novels.

(II) Some novels are magazines.

Some

A. Only (I) follow

B. Only (II) follows

C. Both (I) and (II) follow

D. Neither (I) nor (II) follows

 

Q. 144 In each of the following questions, a related pair of words is followed bu four pairs of words or phrases. Select the pair that best expresses a relationship similar to the one expressed in the question pair.

Statements:

All students like excursions.

Some students go for higher education.

Conclusions:

(I) Students who go for higher education also like excursions.

(II) Some students do not go for higher education, but like excursions.

A. Only (I) follows

B. Only (II) follows

C. Both (I) and (II) follows

D. Neither (I) nor (II) follows

 

Q. 145 In each of the following questions, a related pair of words is followed bu four pairs of words or phrases. Select the pair that best expresses a relationship similar to the one expressed in the question pair.

Statements:

All good hockey players are in the Indian Hockey team.

‘X’ is not a good hockey player.

Conclusions:

(I) ‘X’ is not in the Indian Hockey team.

(II) ‘X’ wants to be in the Indian Hockey team.

A. Only (I) follows

B. Only (II) follows

C. Both (I) and (II) follows

D. Neither (I) nor (II) follows

 

Q. 146 In each of the following questions, a related pair of words is followed bu four pairs of words or phrases. Select the pair that best expresses a relationship similar to the one expressed in the question pair.

India : Tricolour

A. China : Sickle and Hammer

B. UK : Red Cross

C. USA : Stars and Stripes

D. None of the above

 

Q. 147 In each of the following questions, a related pair of words is followed bu four pairs of words or phrases. Select the pair that best expresses a relationship similar to the one expressed in the question pair.

Statute : Law

A. Proviso : Clause

B. Chapter : Exercise

C. University : School

D. Section : Illustration

 

Q. 148 In each of the following questions, a related pair of words is followed bu four pairs of words or phrases. Select the pair that best expresses a relationship similar to the one expressed in the question pair.

Buddhists : Pagoda

A. Parsis : Temple

B. Christians : Cross

C. Jains : Sun Temple

D. Jews : Synagogue

 

Q. 149 Each question consists of five statements (a-e) followed by options consisting of three statements put together in a specific order. Choose the option which indicates a valid argument; that is, where the third statement is a conclusion drawn from the preceding two statements.

a. Law graduates are in great demand.

b. Rajesh and Krishna are in great demand.

c. Rajesh is in great demand.

d. Krishna is in great demand.

e. Rajesh and Krishna are law graduates.

Choose the correct option:

A. abe

B. ecd

C. aeb

D. eba

 

Q. 150  Each question consists of five statements (a-e) followed by options consisting of three statements put together in a specific order. Choose the option which indicates a valid argument; that is, where the third statement is a conclusion drawn from the preceding two Statements. 

a. All captains are great players.

b. Some captains are successful sports administrators.

c. Ritwik is a great player.

d. Ritwik is a captain and successful sports administrator.

e. Some successful sports administrators are great players.

Choose the correct option:

A. acd

B. abe

C. dca

D. edc

 

Q. 151 Principle:

When one person signifies to another his willingness to do or to abstain from doing anything, with a view to obtaining the assent of that other to such act or abstinence, he is said to make a proposal. The expression of willingness/desire results in a valid proposal only when it is made/addressed to some person(s). 

Facts:

‘X’ makes the following statement in an uninhabited hall: ‘I wish to sell my mobile phone for Rs. 1,000.’

Which of the following derivations is CORRECT?

A. ‘X ‘made a statement that resulted in a promise

B. ‘X’ made a statement that resulted in a proposal

C. ‘X’ made a statement that did not result in any proposal

D. ‘X’ made a statement that resulted in an agreement

 

Q. 152 Principle: A proposal (offer) should be made with an intention that after its valid

acceptance, a legally binding promise or agreement will be created. The test for the determination of such intention is not subjective, rather it is objective. The intention of the parties is to be ascertained from the terms of the agreement and the surrounding circumstances under which such an agreement is entered into. As a general rule, in the case of arrangements regulating social relations, it follows as a matter of course that the parties do not intend legal consequences to follow. On the contrary, as a general rule, in the case of arrangements regulating business affairs, it follows as a matter of course that the parties intend legal consequences to follow. However, the above rules are just presumptive in nature, and hence, can be rebutted. 

Facts: One morning while having breakfast, ‘X’, the father, says to ‘Y’ (X’s son), in a casual manner, ‘I shall buy a motorbike for you if you get through the CLAT?

A. ‘X’ made a statement that resulted in an enforceable promise

B. ‘X’ made a statement that resulted in a valid proposal

C. ‘X’ made a statement that resulted in an enforceable agreement

D. ‘X’ made a statement that did not result in any enforceable agreement

 

Q. 153 Principle: Acceptance (of offer) must be communicated by the offeree to the offeror so as to give rise to a binding obligation. The expression ‘by the offeree to the offeror’ includes communication between their authorised agents.

Facts: ‘X’ made an offer to buy Y’s property for a stipulated price. ‘Y’ accepted it and communicated his acceptance to ‘Z’, a stranger.

Which of the following derivations is CORRECT?

A. Y’s acceptance resulted in an agreement

B. Y’s acceptance did not result in any agreement

C. Y’s acceptance resulted in a contract

D. Y’s acceptance resulted in a promise

 

Q. 154 Principle: Acceptance should be made while the offer is still subsisting. The offeror is free to retract his offer at any time before his offer gets accepted by the offeree. Once the offer is withdrawn or is lapsed, it is not open to be accepted so as to give rise to a contract.

Similarly, if a time is prescribed within which the offer is to be accepted, the, the offer must be accepted within the prescribed time. And, if no time is prescribed, then, the acceptance must be made within a reasonable time. ‘What is a reasonable time’, is a question of fact which is to be determined by taking into account all the relevant facts and surrounding circumstances.

Facts: ‘X’ makes an offer to ‘Y’ to sell his equipment for Rs. 1,000.00. No time is specified for the acceptance. ‘Y’ sends his reply two years after receiving the offer.

Which of the following derivations is CORRECT?

A. There arises a contract between ‘X’ and ‘Y’ to sell/buy the equipment in question for

Rs. 1,000.00

B. There does not arise any contract between ‘X’ and ‘Y’ to sell/buy the equipment in question for Rs. 1,000.00

C. ‘X’ is bound by his offer, and hence, cannot reject the acceptance made by ‘Y’

D. There arises a promise by ‘Y’ to buy the equipment

 

Q. 155 Principle: Minor’s agreement is void from the very beginning. It can never be validated. It cannot be enforced in the court of law.

Facts: ‘A’, a boy of 16 years of age, agrees to buy a camera from ‘B’, who is a girl of 21 years of age. Which of the following derivations is CORRECT?

A. There arises a contract between ‘A’ and ‘B’ to sell/buy the camera in question

B. There arises an enforceable agreement between ‘A’ and ‘B’ to sell/buy the camera in question

C. There does not arise any contract between ‘A’ and ‘B’ to sell/buy the camera in question

D. There arises a voidable contract between ‘A’ and ‘B’ to sell/buy the camera in question

 

Q. 156 Principle:

A contract which is duly supported by real and unlawful consideration is valid notwithstanding the fact that the consideration is inadequate. The quantum of consideration is for the parties to decide at the time of making a contract, and not for the courts (to decide) when the contract is sought to be enforced. An agreement to which the consent of the promisor is freely given is not void merely because the consideration is inadequate; but the inadequacy of the consideration may be taken into account by the Court in determining the question whether the consent of the promisor was freely given. Facts:

‘A’, agrees to sell his mobile phone worth Rs. 20,000/- for Rs 100/- only to ‘B’. A’s consent is freely given.

Which of the following derivations is CORRECT?

A. There is a contract between ‘A’ and ‘B’

B. There is no contract between ‘A’ and ‘B’ because consideration is not adequate

C. There is no contract between ‘A’ and ‘B’ because a mobile phone worth Rs. 20,000/- cannot be sold for just Rs. 100/-

D. None of the above

 

Q. 157 Principle: The consideration or object of an agreement is unlawful if it is forbidden by law. Every agreement of which the object or consideration is unlawful is void. 

Facts: ‘X’, promises to pay ‘Y’ Rs. 50,000, if he (‘Y’) commits a crime. ‘X’ further promises to indemnify him (‘Y’) against any liability arising thereof. ‘Y’ agrees to act as per X’s promise.

Which of the following derivations is CORRECT?

A. There is a contract between ‘X’ and ‘Y’

B. There is an agreement between ‘X’ and ‘Y’ which can be enforced by the court of law

C. There is an agreement between ‘X’ and ‘Y’ which can not be enforced by the court of law

D. There is a voidable contract between ‘X’ and ‘Y’

 

Q. 158 Principle: The consideration or object of an agreement is unlawful if the Court regards it as opposed to public policy. Every agreement of which the object or consideration is unlawful is void. 

Facts: ‘X’ promises to obtain for ‘Y’ an employment in the public service; and ‘Y’ promises to pay Rs. 5,00,000/- to ‘X’.

Which of the following derivations is CORRECT?

A. There is a contract between ‘X’ and ‘Y’

B. There is a voidable contract between ‘X’ and ‘Y’

C. There is an agreement between ‘X’ and ‘Y’ which can be enforced by the Court of law

D. There is an agreement between ‘X’ and ‘Y’ which cannot be enforced by the Court of law

 

Q. 159 Principle: Two or more persons are said to consent if they agree upon the same thing in the same sense. Consent is said to be free when it is not caused by coercion, or undue influence, or fraud, or misrepresentation, or mistake. When consent to an agreement is caused by coercion, undue influence, fraud or misrepresentation, the agreement is a contract voidable (rescindable or terminable) at the option of the party whose consent was so caused. However, when consent to an agreement is caused by mistake as to a matter of fact essential to the agreement, the agreement is void.

Facts: ‘X’ threatens to gun down ‘Y’, if he (Y) does not sell his property worth Rs. 20,00,000/- for Rs. 1,00,000/- only. As a consequence, ‘Y’ agrees to sell it as demanded by ‘X’.

Which of the following derivations is CORRECT?

A. There is a contract between ‘X’ and ‘Y’

B. There is an agreement between ‘X’ and ‘Y’ which can be enforced by the court of law.

C. There is an agreement between ‘X’ and ‘Y’ which cannot be enforced by the court of law.

D. There is no contract between ‘X’ and ‘Y’ which voidable at the option of ‘Y’

 

Q. 160 Principle: Agreements in restraint of marriage are void.

Facts: ‘X’ enters into an agreement with ‘Y’ whereunder he agrees not to marry anybody else other than a person whose name starts with the letter ‘A’, and promises to pay Rs. 1,00,000/- to ‘Y’ if he (‘X’) breaks this agreement 

A. There is a contract between ‘X’ and ‘Y’

B. There is an agreement between ‘X’ and ‘Y’ which can be enforced by the Court of law

C. There is an agreement between ‘X’ and ‘Y’ which cannot be enforced by the Court of law

D. There is a voidable contract between ‘X’ and ‘Y’

 

Q. 161 Principle: Vicarious liability is the liability of the Master of Principal for the tort committed by his servant or agent, provided the tort is committed in the course of employment. The Master or Principal is not liable for private wrongs of the servant/agent.

Facts: ‘X’ hands over some cash money at his house to ‘Y’, who is his (X’s) neighbour and is also cashier in a bank, to be deposited in A’s account in the bank. Instead of depositing the money, ‘Y’ misappropriates it.

Which of the following statements depicts correct legal position in this given situation?

A. The bank would not be liable because ‘Y’ did not do any wrong in the course of his employment

B. The bank would be vicariously liable because ‘Y’ was the employee of the bank

C. The bank would not be liable because ‘Y’ did not do any wrong

D. The bank would be liable because ‘Y’ acted as bank’s agent

 

Q. 162 Principle: A person has no legal remedy for an injury caused by an act to which he has consented.

Facts:  ‘R’, a cricket enthusiast, purchased a ticket to watch at20 match organised by the Indian Premier League (IPL). During the match, a ball struck for six hits ‘R’ on his body and injures him. He sues IPL for compensation for the medical expenses.

Which of the following derivations is CORRECT?

A. ‘R’ should be compensated as he purchased the ticket to get entertainment and not to get injured

B. ‘R’ would fail in his action, as he voluntarily exposed himself to the risk

C. IPL would be liable as it did not ensure that the spectators were protected from the risk of such injuries

D. None of the above

 

Q. 163 Principle: Ignorance of his excuses no one.

Facts: ‘X’ fails to file his income tax returns for a considerable number of years. The Income Tax department serves upon him a ‘show-cause notice’ as to why proceedings should not be initiated against him for the recovery of the income tax due from him with interest and penalty.

Which of the following derivations is CORRECT?

A. ‘X’ may defend himself by taking the plea that his legal advisor had not advised him to file the return

B. ‘X’ would have to pay the due, as ignorance of law and failure to comply with law is no legal ground of defence

C. ‘X’ may defend himself successfully by taking the plea that he was unaware of any such law being in force

D. None of the above

 

Q. 164 Principle: Damage without the violation of a legal right is not actionable in a court of law. If the interference with the rights of another person is not unlawful or unauthorised , but a necessary consequence of exercise of defendant’s own lawful rights, no action should lie.

Facts: There was as established school (ES) in a particular locality. Subsequently, a new school (NS) was set up in the same locality, which charged lower fees, on account of which people started patronising the new school. Because of the competition, ES had to reduce its fees. ES filled a case against NS saying that NS had caused it (ES) financial loss and, thus, claimed compensation.

Which of the following derivations is CORRECT?

A. Since no legal right of ES had been violated, therefore, as such, no compensation could be granted

B. Since damage is caused to ES, therefore, it should be awarded compensation

C. ES should be awarded compensation, as opening of school in competition is not good

D. No compensation could be granted, as reduction in fees is good for the public

 

Q. 165 Principle: Whenever there is an invasion of a legal right, the person in whom the right is vested, is entitled to bring an action though he has suffered no actual loss or harm, and may recover damages (compensation).

Facts: ‘A’ was a qualified voter for the Lok Sabha election. However, a returning officer wrongfully refused to take A’s vote. In spite of such a wrongful refusal, the candidate , for whom ‘A’ wanted to vote, won the election. But, ‘A’ brought an action for damages.

Which of the following derivations is CORRECT? 

A. Since no legal right of ‘A’ had been violated, therefore, as such, no compensation could be granted

B. Since legal right of ‘A’ had been violated, therefore, compensation should be granted

C. No compensation could be granted, as ‘A’ had suffered no loss as his candidate won the election

D. Such no fundamental right of ‘A’ had been violated, therefore, as such, no compensation could be granted

 

Q. 166 Principle: In a civil action for defamation, truth of the defamatory matter is an absolute defence. However, the burden of proving truth is on the defendant; and he is liable if he does not successfully discharge this burden. 

Facts: ‘D’, who was the editor of a local weekly, published a series of articles mentioning that ‘P’, who was a government servant, issued false certificates, accepted bribe, adopted corrupt and illegal means to mint money and was a ‘mischief monger’. ‘P’ brought a civil action against ‘D’, who could not prove the facts published by him.

Under the circumstances, which of the following derivations is CORRECT?

A. ‘D’ would be liable, since he could not prove the facts published by him

B. ‘D’ would not be liable, as such an action could curtail the right of expression and speech of press

C. ‘D’ would not be able, as media could publish anything

D. None of the above

 

Q. 167 Principle: A gift comprising both existing and future property is void as to the latter.

Facts: ‘X’ has a house which is owned by him. He contracted to purchase a plot of land adjacent to the said house, but the sale (of the plot of land) in his favour is yet to be completed. He makes a gift of both the properties (house and land) to ‘Y’.

A. Gift of both the properties is valid.

B. Gift of both the properties is void.

C. Gift of house is void, but the gift of the plot of land is valid.

D. Gift of house is valid, but the gift of the plot of land is void.

 

Q. 168 Principle: Caveat emptor, i.e., ‘let the buyer beware’ stands for the practical skill and judgement of the buyer in his choice of goods for purchase. It is the business of the buyer to judge for himself that what he buys has its use and worth for him. Once nought, and if the buy is not up to his expectations, then he alone is to blame and no one else.

Facts: For the purpose of making uniform for the employees, ‘A’ bought dark blue coloured cloth from ‘B’, but did not disclose to the seller (B) the specific purpose of the said purchase. When uniforms were prepared and used by the employees, the cloth was found unfit. However, the cloth was fit for a variety of other purposes (such as, making caps, boots and carriage lining, etc).

Applying the fore-stated principle, which of the following derivations is CORRECT as regards remedy available to ‘A’ in the given situation?

A. ‘A’ (the buyer) would succeed in getting some remedy from ‘B’ (the seller)

B. ‘A’ (the buyer) would not succeed in getting any remedy from ‘B’ (the seller)

C. ‘A’ (the buyer) would succeed in getting refund from ‘B’ (the seller)

D. ‘A’ (the buyer) would succeed in getting a different variety of cloth from ‘B’ (the seller), but not the refund

 

Q. 169 Principle: The transferor of goods cannot pass a better title than what he himself possesses.

Facts: ‘X’ sells a stolen bike to ‘Y’. ‘Y’ buys it in good faith.

As regards the title to bike, which of the following derivations is CORRECT?

A. The real owner cannot get back the bike from ‘Y’

B. ‘Y’ will get no title, as transferor’s (X’s) title was defective.

C. ‘Y’ will get good title, as he is a bonafide buyer

D. ‘Y’ will get good title, as has not committed any wrong (stolen the bike)

 

Q. 170 Principle: Negligence is a breach of duty or a failure of one party to exercise the standard of cure required by law, resulting in damage to the party to whom the duty was owed. A plaintiff can take civil action against the respondent, if the respondent’s negligence causes the plaintiff injury or loss of property.

Facts: ‘D’ went to a cafe and ordered and paid for a tin/can of soft drink. The tin was opaque, and, therefore, the contents could not be seen from outside. She (D) consumed some of the contents and then lifted the tin to pour the remainder of the content into a tumbler. The remains of a snail in decomposed state dropped out of the tin into the tumbler. ‘D’ later complained of a stomach pain and her doctor diagnosed her as having gastroenteritis and being in a state of severe shock. She sued the manufacturer of the drink for negligence. Applying the afore-stated principle, which of the following derivations is CORRECT as regards liability of the manufacturer in the given situation?

A. The manufacturer is liable for negligence, as it owed a duty (to consumers) to take reasonable care to ensure that its products are safe for consumptions

B. The manufacturer is not liable for negligence, as there is no direct contract between ‘D’ and the manufacturer. No duty is owed by the manufacturer towards a particular consumer (D)

C. The manufacturer is not liable for negligence because it would otherwise become very difficult for the manufacturer to do business

D. The manufacturer could be made liable under criminal law, but not for tort of negligence

 

Q. 171 Principle: Master is liable for the wrongful acts committed by his servant; provided the acts are committed during the course of employment. However, the master is not liable if the wrongful act committed by his servant has no connection, whatsoever, with the servant’s contract of employment.

Facts: ‘D’ is a driver employed by ‘M’, who is the owner of a company. During the lunch time, ‘D’ goes to a closeby tea shop to have a cup of tea. There he (D) picks up fight with the tea shop owner (T), which resulted in some damage to his shop. ‘T’ wants to sue ‘M’ for claiming comprehension for the damage caused by the fight. Which of the following derivations is CORRECT?

A. ‘M’ will be liable because ‘D’ is his servant

B. Both ‘M’ and ‘D’ will be liable

C. ‘M’ will not be liable because the wrongful act (picking up fight) was not committed in the course of D’s employment

D. ‘M’ will be liable albeit the wrongful act (picking up fight) was not committed in the course of D’s employment

 

Q. 172 Principle: The Constitution of India guarantees the ‘right of life’, which means ‘right to live with human dignity’. The right to life under the Constitution, however, does not include the right to die. 

Facts: ‘M’, who is 90, lives all alone as he has no family or children or grandchildren. He suffers from physical and mental distress, as there is no one to look after him. He has little means to foot his medical expenses. Under these circumstances, he approaches the court with a prayer that he should be granted the right to die with dignity because he does not want to be a burden on the society. Further, as it is his life, he has a right to put an end to it. Which of the following derivations is CORRECT?

A. The prayer can be granted, as suicide is not an offence in India

B. The prayer can be granted, as the right to life under the Constitution includes the right to die.

C. The prayer can be granted, as a person cannot be forced to enjoy right to life to his detriment, disadvantage and disliking.

D. The prayer cannot be granted, as the right to life under the Constitution does not include the right to die

 

Q. 173 Principle: Trespass to land means direct interference with the possession of land without lawful justification. Trespass could be committed either by a person himself entering the land of another person or doing the same through some tangible object(s). 

Facts: ‘A’ throws some stones upon his neighbour’s (B’s) premises.

A. ‘A’ has committed trespass

B. ‘A’ has not committed trespass, as he has not entered B’s premises

C. ‘A’ has committed nuisance

D. None of the above

 

Q. 174 Nuisance is an unlawful interference with a person’s use or enjoyment of land or some right over or in connection with it. If the interference is ‘direct’, the wrong is trespass; whereas, if the interference is ‘consequential’, it amounts to nuisance

Facts: ‘A’ plants a tree on his land. However, he allows its branches to project over the land of ‘B’. Which of the following derivations is CORRECT?

A. ‘A’ has committed trespass

B. ‘A’ has committed nuisance

C. ‘A’ has not committed trespass

D. None of the above

 

Q. 175 Principle: Interference with another’s goods in such a way as to deny the latter’s title to the goods amounts to conversion, and thus it is a civil wrong. It is an act intentionally done inconsistent with the owner’s right, though the doer may not know of, or intend to challenge, the property or possession of the true owner.

Facts: ‘R’ went to a cycle-stand to park his bicycle. Seeing the stand fully occupied, he removed a few bicycles in order to rearrange a portion of the stand and make some space for his bicycle. He parked his bicycle properly, and put back all the bicycles except the one belonging to ‘S’. In fact, ‘R’ was in a hurry, and therefore, he could not put back S’s bicycle. Somebody came on the way and took away S’s bicycle. The watchman of the stand did not take care of it assuming that the bicycle was not parked inside the stand. ‘S’ filed a suit against ‘R’ for conversion.

Which of the following derivations is CORRECT?

A. ‘R’ could not be held liable for the negligence of the watchman

B. ‘S’ would succeed because R’s act led to the stealing of his bicycle

C. ‘S’ would not succeed because ‘R’ did not take away the bicycle himself

D. ‘S’ would not succeed because R’s intention was not bad

 

Q. 176 Principle: Nothing is an offence which is done by a person who is bound by law to do it.

Facts: ‘A’, a police officer, without warrant, apprehends ‘Z’, who has committed murder.

A. ‘A’ is guilty of the offence of wrongful confinement

B. ‘A’ is not guilty of the offence of wrongful confinement

C. ‘A’ is guilty of the offence of wrongful restraint

D. ‘A’ cannot apprehend ‘Z’ without a warrant issued by a court of law

 

Q. 177 Principle: When a criminal act is done by several persons in furtherance of the common intention of all, each of such persons is liable for that act in the same manner as if it were done by him alone.

Facts: Roshan along with two of his friends, Tushar and Tarang proceeded to the house of Darshan in order to avenge an insult made by the brother of Darshan. They opened fire on the members of Darhsan’s family. It was found that the shots of Roshan did nit hit anyone, but the shots of Tushar and Tarang succeeded in killing Darshan.

A. Roshan was not liable for the offence of murder of Darshan, as Rohan’s shots did not hit Darshan

B. Only Tushar and Tarang were liable for the offence of murder of Darshan, as their shots hit Darshan

C. Roshan along with Tushar and Tarang was liable for the offence of murder of Darshan

D. Roshan was liable to a lesser extent comparing to his friends for the offence of murder of Darshan, as Roshan’s shots did not hit Darshan

 

Q. 178 Principle: No communication made in good faith is an offence by reason of any harm to the person to whom it is made, if it is made for the benefit of that person.

Facts: ‘A’, a surgeon, in good faith, communicates to a patient his opinion that he cannot live. The patient dies in consequence of the shock.

A. ‘A’ has committed the offence of causing death of his patient.

B. ‘A’ has not committed the offence of causing death of his patient.

C. ‘A’ has partially committed the offence of causing death of his patient.

D. None of the above

 

Q. 179 Principle: Whoever, being legally bound to furnish information on any subject to any public servant, as such, furnishes, as true, information on the subject which he knows or has reason to believe to be false, has committed a punishable offence of furnishing false information. 

Facts: Sawant, a landholder, knowing of the commission of a murder within the limits of his estate, willfully misinforms the Magistrate of the district that the death has occurred by accident in consequence of the bite of a snake.

A. Sawant is not guilty of the offence of furnishing false information to the Magistrate

B. Sawant is guilty of the offence of furnishing false information to the Magistrate

C. Sawant is not legally bound to furnish true information to the Magistrate

D. Sawant has the discretion to furnish true information to the Magistrate, as the murder was committed within the limits of his estate

 

Q. 180 Principle: Whoever unlawfully or negligently does any act which is, and which he knows or has reason to believe to be, likely to spread the infection of any disease dangerous to life, shall be guilty of a negligent act likely to spread infection of disease dangerous to life.

Facts: ‘K’, a person, knowing that he is suffering from Cholera, travels by a train without informing the railway officers of his condition.

A. ‘K’ has committed an unlawful and negligent act, which is likely to spread infection of Cholera disease dangerous to the life of fellow-passengers

B. Railway officers are guilty of an unlawful and negligent act, as ‘K’ who is suffering from Cholera disease has travelled by the train

C. ‘K’ has not committed an unlawful and negligent act, which is likely to spread infection of Cholera disease dangerous to the life of fellow-passengers

D. Both ‘K’ and Railway officers are guilty of an unlawful and negligent act, which is likely to spread infection of Cholera disease dangerous to the life of fellow-passengers

 

Q. 181 Principle: Whoever drives any vehicle, or rides, on any public way in a manner so rash or negligent as to endanger human life, or to be likely to cause hurt or injury to any other person, has committed an offence, which shall be punished in accordance with the law.

Facts: ‘X’, a truck driver, driving his vehicle rashly and negligently at a high speed climbed the footpath and hit ‘Y’, a pedestrian, from behind causing his death.

A. ‘X’ is not guilty of rash an negligent driving

B. ‘X’ should have taken sufficient care on the footpath

C. ‘X’ is guilty of rash an negligent driving

D. ‘X’ is only in part guilty of rash an negligent driving

 

Q. 182 Principle: Whoever causes death by doing an act with the intention of causing death, or with the intention of causing such bodily injury as is likely to cause death, or with the knowledge that he is likely by such act to cause death, commits the offence of culpable homicide.

Facts: ‘A’ knows ‘Z’ to be behind a bush. ‘B’ does not know it. ‘A’, intending to cause, or knowing it to be likely to cause Z’s death, induces ‘B’ to fire at the bush. ‘B’ fires and kill ‘Z’.

A. ‘B’ has committed the offence of culpable homicide.

B. ‘A’ has committed the offence of culpable homicide.

C. Both ‘A’ and ‘B’ has committed the offence of culpable homicide.

D. None of them has committed the offence of culpable homicide.

 

Q. 183 Principle: Whoever, intending to take dishonestly any movable property out of the possession of any person without that person’s consent, moves that property in order to such taking, is said to commit theft.

Facts: ‘Z’, going on a journey, entrusts his plate to the possession of ‘A’, the keeper of a warehouse, till ‘Z’ shall return. Then, ‘A’ carries the plate to a goldsmith and sells it.

A. ‘A’ has committed theft

B. ‘A’ has not committed theft

C. ‘A’ lawfully sold the plate to the goldsmith

D. None of the above is true

 

Q. 184 Principle: Whoever makes any false document or part of a document with intent to cause damage or injury, to the public or to any person, or to support any claim or title, or to cause any person to part with property, or to enter into any express or implied contract, or with intent to commit fraud or that fraud may be committed, commits forgery.

Facts: ‘A’ without Z’s authority, affixes Z’s seal to a document purporting to be a conveyance of an estate from ‘Z’ to ‘A’, with the intention of selling the estate to ‘B’ and thereby of obtaining from ‘B’ the purchase-money.

A. ‘B’ has committed forgery

B. ‘Z’ has committed forgery

C. ‘A’ has committed forgery

D. ‘A’ and ‘B’ have committed forgery

 

Q. 185 Principle: Whoever intentionally uses force to any person, without that person’s consent, in order to the committing of any offence, or intending by the use of such force to cause, or knowing it to be likely that by the use of such force he will cause injury, fear or annoyance to the person to whom the force is used, is said to use criminal force to that other. 

Facts: ‘Z’ is riding a palanquin. ‘A’, intending to rob ‘Z’ , seized the pole and stops the palanquin. Her ‘A’ has caused cessation of motion to ‘Z’, and ‘A’ has done this by his own bodily power. 

A. ‘A’ has used criminal force to ‘Z’

B. ‘A’ has no intention to use criminal force to ‘Z’

C. ‘A’ has used force with the consent of ‘Z’

D. None of the above is correct

 

Q. 186 Principle: One of the essential conditions for a marriage between any two persons to be solemnized under the Special Marriage Act, 1954 is that at the time of the marriage the male has completed the age of twenty-one years and the female the age of eighteen years. If the said condition is not fulfilled such a marriage is null or void.

Facts: ‘A’, a male aged twenty-two years, proposes to marry ‘B’, a female aged sixteen years, at Delhi in the month of June 2014 under the Special Marriage Act, 1954

A. Marriage between ‘A’ and ‘B’ can be legally solemnized under the Special Marriage Act, 1954

B. Marriage between ‘A’ and ‘B’ cannot be legally solemnized under the Special Marriage Act, 1954

C. Marriage between ‘A’ and ‘B’ can remain valid for A under the Special Marriage Act, 1954

D. None of the above is correct

 

Q. 187 Principle: Under the Hindu Marriage Act, 1955 either the husband or the wife can move a petition for a decree of divorce on the ground of desertion. The term ‘desertion’ means desertion of the petitioner by the other party to the marriage for a continuous period of not less than two years immediately preceding the presentation of the petition, without reasonable cause and without the consent or against the wish of such party and includes the willful neglect of the petitioner by the other party to the marriage, and its grammatical variations and cognate expressions shall be construed accordingly. It is also said that desertion is withdrawal not from a place but from a state of things.

Facts: Rohan, a technocrat, went to US in January 2011 for pursuing his higher studies for a period of three years. In fact, Rohan went to US with the consent oh his wife Basanti, who stayed at her parents’ home , and with a promise of his return to India upon the completion of his studies. From US he has quite often been in touch with his wife. Subsequently, Rohan has got a job there in US and he wishes to take his wife. She refuses to go to US and, in the meanwhile, she files a petition for a decree of divorce on the ground of desertion by her husband.

A. Rohan’s three year stay in US in the above context can amount to a ground of desertion for divorce

B. Rohan’s three year stay in US in the above context cannot amount to a ground of desertion for divorce

C. Rohan’s continued stay after three years can amount to a ground of desertion for divorce

D. Basanti’s refusal can amount to a ground of desertion for divorce

 

Q. 188 Principle: Under the Hindu Adoptions and Maintenance Act, 1956, no person shall be capable of being taken in adoption unless he or she is a Hindu, he or she not already been adopted, he or she has not been married, unless there is a custom or usage applicable to the parties which permits persons who are married being taken in adoption,, and he or she has not completed the age of fifteen years, unless there is a custom or usage applicable to the parties which permits persons who have completed the gae of fifteen years being take in adoption. 

Facts: Vijay being natural father had given Tarun, a boy aged 10 years, in adoption to Manoj in March 2010 in accordance with the Hindu Adoptions and Maintenance Act, 1956. In May 2012 Manoj gave Tarun in adoption to Sanjay. Subsequently in December 2013, Sanjay gave Tarun in adoption to Vijay.

A. Adoption of Tarun by Sanjay is valid

B. Adoption of Tarun by Vijay is valid

C. Adoption of Tarun by Manoj is valid

D. None of the above adoptions is valid

 

Q. 189 Principle: Under copyright law copyright subsists in original literary works also. A literary work need not be of literary quality. Even so prosaic a work as an index of railway stations or a railway guide or a list of stock exchange quotations qualify as a literary work if sufficient work has been expended in compiling it to give it a new and original character.

Facts: Michael works hard enough, walking down the streets, taking down the names of people who live at houses and makes a street directory as a result of that labour. 

A. Michael’s exercise in making a street directory is sufficient to justify in making claim to copyright in that work which is ultimately produced

B. Michael’s exercise in making a street directory is not enough to justify in making claim to copyright in that work which is ultimately produced 

C. A street directory cannot be enough to be published as a literary work

D. None of the above statements is correct

 

Q. 190 Principle: Every person shall be liable to punishment under the Indian Penal Code and not otherwise for every act or omission contrary to the provisions of the Code of which he shall be guilty within the territory of India. In other words, the exercise of criminal jurisdiction depends upon the locality of the offence committed, and not upon the nationality or locality of the offender.

Facts: ‘X’, a Pakistani citizen, while staying at Karachi, made false representations to ‘Y’, the complainant, at Bombay through letters, telephone calls and telegrams and induced the complainant to part with money amounting to over rupees five lakh to the agents of ‘X’ at Bombay, so that rice could be shipped from Karachi to India as per agreement. But the rice was never supplied to the complainant.

A. The offence of cheating under section 420 of the Code was committed by ‘X’ within India, even though he was not physically present at the time and place of the crime

B. The offence of cheating as per section 420 of the Code was not committed by ‘X’ within India, as he was not physically present at the time and place of the crime

C. Only the agents of ‘X’ had committed the offence of cheating under section 420 of the Code within India, as they were physically present at the time and place of the crime

D. ‘Y’ was also liable for the offence of cheating under section 420 of the Code within India, as he was physically present in the time and place of the crime

 

Q. 191 Principle: When two or more persons agree to do, or cause to be done, (1) an illegal act, or (2) an act which is not illegal by illegal means, through such an agreement such persons are said to have been engaged in a criminal conspiracy to commit an offence. It is said that no consummation of the crime need to achieved or even attempted.

Facts: ‘X’, ‘Y’ and ‘Z’ plan to kill ‘D’. They agree that only one among them, that is ‘Z’, will execute the plan. In pursuance of it ‘Z’ buys a gun and loads it.

A. Only ‘Z’ can be charged with criminal conspiracy to kill ‘D’

B. All of them, i.e., ‘X’, ‘Y’, and ‘Z’, can be charged with criminal conspiracy to kill ‘D’

C. ‘X’ and ‘Y’ cannot be charged with criminal conspiracy to kill ‘D’

D. None of them can be charged with criminal conspiracy to kill ‘D’

 

Q. 192 Principle: ‘Wrongful gain’ is gain by unlawful means of property to which the person gaining is not legally entitled. ‘Wrongful loss’ is the loss by unlawful means of property to which the person losing it is legally entitled.

Facts: ‘X’ takes away Y’s watch out of Y’s possession, without Y’s consent and with the intention of keeping it.

A. ‘X’ causes ‘wrongful gain’ to ‘Y’

B. ‘Y’ causes ‘wrongful gain’ to ‘X’

C. ‘X’ causes ‘wrongful loss’ to ‘Y’

D. ‘Y’ causes ‘wrongful loss’ to ‘X’

 

Q. 193 Principle: Nothing is an offence by reason that it causes, or that it is intended to cause, or that it is known to be likely to cause, any harm, if that harm is so slight that no person of ordinary sense and temper would complain of such harm. 

Facts: ‘X’ takes a plain sheet of paper from Y’s drawer without Y’s consent to write a letter to his friend.

A. ‘X’ has committed an offence in the above context

B. ‘X’ has committed no offence in the above context

C. ‘Y’ can sue ‘X’ for an offence in the above context

D. None of the above is correct in the above context

 

Q. 194 Principle: When an act which would otherwise be a certain offence, is not that offence, by reason of the youth, the want of maturity of understanding, the soundness of mind or the intoxication of the person doing that act, or by reason of any misconception on the part of that person, every person has the same right of private defence against that act which he would have if the act were that offence.

Facts: ‘X’, under the influence of madness, attempts to kill ‘Y’.

A. ‘Y’ has the right of private defence against ‘X’

B. ‘Y’ does not have the right of private defence against ‘X’

C. ‘Y’ has the right of private defence against ‘X’, only if ‘X’ is not under the influence of madness

D. ‘X’ has the right of private defence against ‘Y’

 

Q. 195 Principle: When a person fraudulently or erroneously represents that he is authorized to transfer certain immovable property and professes to transfer such property for consideration, such transfer shall, at the option of the transferee, operate on any interest which the transferor may acquire in such property at any time during which the contract of transfer subsists.

Facts: ‘A’, a Hindu who has separated from his father ‘B’, sells to ‘C’ three fields, X, Y and Z, representing that ‘A’ is authorised to transfer the same. Of these fields Z does not belong to ‘A’, it having been retained by ‘B’ on the partition; but on B’s dying ‘A’ as successor obtains Z, and at that time ‘C’ had not cancelled the contract of sale. 

A. ‘A’ can sell Z to a third party

B. ‘A’ is not required to deliver Z to ‘C’

C. ‘A’ is required to deliver Z to ‘C’

D. None of the above statements is correct

 

Q. 196 Principle: Under the Transfer of Property Act, 1882 a property must be transferred by one living person to another living person. The Act deals only with transfer of property between living persons.

Facts: ‘X’ wants to transfer his property to the presiding deity in a temple situated within the estate of ‘A’.

A. Transfer of property by ‘X’ will be valid

B. Transfer of property by ‘X’ will be invalid

C. Transfer of property by ‘X’ to the presiding deity will become a valid transfer to ‘A’

D. None of the above is correct

 

Q. 197 Principle: When there is transfer of ownership of one thing for the ownership of some other thing it is called exchange; while transfer of ownership for consideration of money is called sale, whereas, without consideration it becomes gift.

Facts: ‘A’ transfers his house worth Rs. 50 Lakhs to ‘B’ for a shopping building worth the same amount, as consideration, from ‘B’.

A. The transaction is a gift

B. The transaction is a sale

C. The transaction is a exchange

D. The transaction is a mortgage

 

Q. 198 Principle: One of the principles of natural justice is Nemo judex in causa sua, which means that no one should be a judge in his own cause. In other words, no person can judge a case in which he has an interest.

Facts: ‘X’, a member of the selection board for a government service, was also a candidate for selection for the same service. ‘X’ did not take part in the deliberations of the board when his name was considered and approved.

A. Selection of ‘X’ is against the principle of natural justice.

B. Selection of ‘X’ is not against the principle of natural justice.

C. Non-selection of ‘X’ will be against the principle of natural justice.

D. Non-participation of ‘X’ in the board deliberations will render his selection valid.

 

Q. 199 Principle: Strike is a collective stoppage of work by workmen undertaken in order to bring pressure upon those who depend on the sale or use of the products or work; whereas, lock-out is a weapon in the hands of the employer, similar to that of strike in the armoury of workmen, used for compelling persons employed by him to accept his terms or conditions of or affecting employment. While in closure there is permanent closing down of a place of employment or part thereof, in lay-off an employer, who is willing to employ, fails or refuses or is unable to provide employment for reasons beyond his control. 

Facts: Workmen of a textile factory went on strike as per law, demanding the payment of bonus. Employer of the factory refused to pay any extra allowances, including bonus, and besides he closed down the factory till the strike was stopped.

A. Act of closing down the factory by the employer amounted to strike.

B. Act of closing down the factory by the employer amounted to lay-off.

C. Act of closing down the factory by the employer amounted to lock-out.

D. Act of closing down the factory by the employer amounted to closure.

 

Q. 200 Principle: Trade disputes means any dispute between employers and workmen or between workmen and workmen, or between employers and employers which is connected with the employment or non-employment, or the terms of employment or the conditions of labour, of any person. Disputes connected with the non-employment must be understood to include a dispute connected with a dismissal, discharge, removal or retrenchment of a workman.

Facts: ‘X’, an employee in a sugar factory, raised a dispute against ‘Y’, the employer, through trade union regarding certain matters connected with his suspension from the employment.

A. Matters connected with suspension can amount to a trade dispute

B. Matters connected with suspension cannot amount to a trade dispute

C. Only after dismissal, matters connected with suspension can amount to a trade dispute

D. None of the above is correct

 

Answer Sheet 
Question 1 2 3 4 5 6 7 8 9 10
Answer D A B D D B D B A B
Question 11 12 13 14 15 16 17 18 19 20
Answer C B A D D C D B A C
Question 21 22 23 24 25 26 27 28 29 30
Answer D B A D B A C D A B
Question 31 32 33 34 35 36 37 38 39 40
Answer C B D A C D D C A B
Question 41 42 43 44 45 46 47 48 49 50
Answer D D B A B C A B D B
Question 51 52 53 54 55 56 57 58 59 60
Answer B C A A C B D C D A
Question 61 62 63 64 65 66 67 68 69 70
Answer C D B B D D C B A D
Question 71 72 73 74 75 76 77 78 79 80
Answer D C B A C C D D C B
Question 81 82 83 84 85 86 87 88 89 90
Answer C D A A C D C D D C
Question 91 92 93 94 95 96 97 98 99 100
Answer D A C C B C C D A B
Question  101 102 103 104 105 106 107 108 109 110
Answer B D C B A C C D D A
Question  111 112 113 114 115 116 117 118 119 120
Answer B D D C C D D A D D
Question 121 122 123 124 125 126 127 128 129 130
Answer B B D A D A D B B C
Question 131 132 133 134 135 136 137 138 139 140
Answer A B C D A D A C A D
Question 141 142 143 144 145 146 147 148 149 150
Answer C A B C A C A D C B
Question 151 152 153 154 155 156 157 158 159 160
Answer C D B B C A C D D C
Question 161 162 163 164 165 166 167 168 169 170
Answer A B B A B A D B B A
Question 171 172 173 174 175 176 177 178 179 180
Answer C D A B B B C B B A
Question 181 182 183 184 185 186 187 188 189 190
Answer C B B C A B B C A A
Question 191 192 193 194 195 196 197 198 199 200
Answer B C B A C B C A C A

 

CLAT 2013 Previous Year Paper

CLAT 2013

Questions: 1 – 10

Direction for Questions 1 to 10 : Read the given passage carefully and attempt the questions that follow and shade the appropriate answer in the space provided for it on the OMR answer sheet.

It is an old saying that knowledge is power. Education is an instrument which imparts knowledge and therefore, indirectly controls power.Therefore, ever since the dawn of our civilisation, persons in power have always tried to supervise or control education. It has been handmaid of the ruling class. During the Christian era, the ecclesiastics controlled the institution of education and diffused among the people the gospel of the Bible and religious teachings. These gospels and teachings were no other than a philosophy for the maintenance of the existing society. It taught the poor man to be meek and to earn his bread with the sweat of his brow, while the priests and the landlords lived in luxury and fought duels for the slightest offence. During the Renaissance, education passed more from the clutches of the priest into the hands of the prince. In other words, it became more secular. Under the control of the monarch, education began to devise and preach the infallibility of its masters, the monarch or king.It also invented and supported fantastic theories like “The Divine right Theory” and that the king can do no wrong, etc. With the advent of the industrial revolution, education took a different turn and had to please the new masters. It now no longer remained the privilege of the baron class, but was thrown open to the new rich merchant class of the society. The philosophy which was in vogue during this period was that of “Laissez Faire” restricting the function of the state to a mere keeping of laws and order while on the other hand, in practice the law of the jungle prevailed in the form of free competition and the survival of the fittest.

 

Q. 1 What does the theory “Divine Right of King” stipulate?

A. The kings are God

B. That the right of governing is conferred upon the kings by God

C. They have the right to be worshipped like Gods by their subjects

D. That the right of kings are divine and therefore sacred

 

Q. 2 Who controlled education during the Renaissance?

A. The common people

B. The prince

C. The church and the priests

D. None of these

 

Q. 3 What did the ruling class in the Christian era think of the poor man?

A. That he is the beloved of God

B. That he deserves all sympathy of the rich

C. That he should be strong and lord over others

D. That he is meant for serving the rich

 

Q. 4 Who controlled the institution of education during the Christian Era?

A. The secular leaders of society

B. The church and the priests

C. The monarchs

D. The common people

 

Q. 5 What does the word “infallibility” mean?

A. That every man is open to error

B. Sensitivity

C. The virtue of not making mistake

D. That some divine power is responsible for determining the fate of the man

 

Q. 6 What do you mean by the “sweat of his brow”?

A. Very hard work

B. The tiny droplets of sweat on the forehead

C. The wrinkles visible on the forehead

D. The sign of innocence

 

Q. 7 What does the policy of “Laissez Faire” stand for?

A. Individual freedom in the economic field

B. State control over law and order in society

C. Joint control of the means of production by the state and private enterprise

D. Decontrol over law and order by the ruling class

 

Q. 8 Which of the following describes the writer?

A. Concerned

B. Unconcerned

C. Aggressive

D. Frustrated

 

Q. 9 Choose the correct synonym out of the four choices given : Gospels

A. Chit chat

B. A teaching or doctrine of a religious teacher

C. Rumour

D. Guidance

 

Q. 10 Choose the correct synonym out of the four choices given : Vogue

A. Uncertain

B. Out-dated

C. The prevailing fashion or style

D. Journey

 

Q. 11 Select the word that is spelt correctly.

A. paraphernalia

B. paraphrenalia

C. parapherenalia

D. paraphrennalia

 

Q. 12 Select the word that is spelt correctly.

A. enterprenuer

B. entrepreneur

C. entrepenur

D. enteruepeur

 

Q. 13 Select the word that is spelt correctly.

A. onomaetopoeia

B. onomoatopoeia

C. onomatopoeia

D. onomatopoeia

 

Q. 14 Select the word that is spelt correctly.

A. hemorhage

B. haemorhhage

C. haemorrhage

D. hemoorhage

 

Q. 15 Select the word that is spelt correctly.

A. dylexsia

B. dyslexia

C. dislexia

D. dislecsia

 

Questions: 16 – 25

Direction for questions 16 to 25 : Select the best option from the four alternatives given below and shade the appropriate answer in the space provided for it in the OMR answer sheet.

 

Q. 16 Unless he ______ this office, I will not say anything.

A. Left

B. Did not leave

C. Leaves

D. Had left

 

Q. 17 _________, I would help all the poor people.

A. If I am rich

B. If I was rich

C. If I were rich

D. In case I am rich

 

Q. 18 I _____ the news an hour ago.

A. Have heard

B. Heard

C. Was hearing

D. Have been hearing

 

Q. 19 He spoke _______ about his prospects.

A. Confidentially

B. Consciously

C. Confidently

D. Conscientiously

 

Q. 20 The boy is not interested in playing, ________?

A. Doesn’t he?

B. Isn’t he?

C. Didn’t he?

D. Is he?

 

Q. 21 He told us that we should never live beyond _______ means.

A. His

B. Their

C. Our

D. Her

 

Q. 22 May I request _________ you again to consider my case favourably.

A. To

B. Onto

C. Of

D. No proposition required

 

Q. 23 Known as devout and serious person, she also has ______ sense of humour.

A. Better

B. Quick

C. Good

D. Beautiful

 

Q. 24 Galileo said, “The Earth ______ around the sun”.

A. Revolved

B. Is revolving

C. Revolves

D. Is revolved

 

Q. 25 We ________ our work before the guests arrived at our house.

A. Shall finish

B. Have finished

C. Had finished

D. Shall have finished

 

Questions: 26 – 30

Direction for Questions 26 to 30: The sentences given in each question, when properly sequenced, form a coherent paragraph. Each sentenced is labelled with a letter. Choose the most logical order of sentences from among the given choices to construct a coherent paragraph. Shade the appropriate answer in the space provided for it in the OMR sheet.

 

Q. 26 a: People who start up their own business typically come from two extreme backgrounds: One is the business family background and the other is a steady professional family background.

b: Typically, people from different backgrounds face different kinds of basic problems.

c: The people from both the backgrounds find it very difficult to establish and manage an enterprise.

d: Starting up and managing a small business is no joke.

A. d b c a

B. b a c d

C. d a c b

D. c d a b

 

Q. 27 a: Venture capital is recommended as the ideal source of financing for a successfully small business.

b: Several companies including start ups have been funded by dedicated venture funds during this decade.

c: Despite this, an average Indian entrepreneur understands and appreciation of venture capital concept has been woefully inadequate.

d: In the Indian context, though venture capital has been a relatively late entrant, it has already made a reasonable impact.

A. a b c d

B. a d b c

C. a c b d

D. a d c b

 

Q. 28 a: Progress in diagnosis, in preventive medicine and in treatment, both medicinal and surgical, has been rapid and breathe taking.

b: Much in medicine which is not taken for granted was undreamt of even as recently as 20 years ago.

c: Presently small pox has been eradicated, poliomyelitis practically banished, tuberculosis has become curable and coronary artery disease surgically relievable.

d: The dramatic surge in the field of molecular biology and research by immunologists and geneticists has succeeded in controlling parasitic diseases like malaria and river blindness that affect millions of people round the world.

A. b d c a

B. b a c d

C. b c a d

D. b d a c

 

Q. 29 a: Instead, many deaths and injuries result from falling objects and the collapse of buildings, bridges and other structures.

b: Earthquakes almost never kill people directly.

c: Fire resulting from broken gas or power lines is another major danger during a quake.

d: Spills of hazardous chemicals are also a concern during an earthquake.

A. c a b d

B. d a c b

C. d c a b

D. b a c d

 

Q. 30 a: The Winchester or hard disk drives can store much more data than what can be stored on a floppy diskette.

b: Hard disks come sealed and they cannot be removed or changed like floppy diskettes.

c: Often floppy disk system is used in conjunction with the Winchester disk system.

d: This makes for an ideal system for secondary storage.

A. c a b d

B. c b d a

C. b a c d

D. a b c d

 

Q. 31 Given below is a foreign language phrase which are commonly used. Choose the correct meaning for the options

El Dorado

A. An imaginary place

B. High altitude

C. A literary man

D. A country full of gold and precious stones

 

Q. 32 Given below is a foreign language phrase which are commonly used. Choose the correct meaning for the options:

quantum ramifactus

A. The amount of damage suffered

B. The amount of damage caused

C. The amount of damage paid

D. The amount of damage received

 

Q. 33 Given below is a foreign language phrase which are commonly used. Choose the correct meaning for the options:

Corpus delicti

A. Fake evidence of an offence

B. Hearsay evidence of an offence

C. Lack of evidence of an offence

D. An evidence which constitute an offence

 

Q. 34 Given below is a foreign language phrase which are commonly used. Choose the correct meaning for the options:

Vis-a-vis

A. Direct

B. Opposite

C. Face to face

D. Agree

 

Q. 35 Given below is a foreign language phrase which are commonly used. Choose the correct meaning for the options:

Carte blanche

A. Complete authority

B. Issue the warrant

C. No authority

D. Lack of authority

 

Q. 36 Choose the explanation that catches the spirit of the idiom-

To blaze a trail

A. To set on fire

B. To blow the trumpet

C. To initiate work in a movement

D. To be hopeful

 

Q. 37 Choose the explanation that catches the spirit of the idiom-

A snake in the grass

A. A secret or hidden enemy

B. An unreliable person

C. Unforeseen happening

D. A dangerous enemy

 

Q. 38 Choose the explanation that catches the spirit of the idiom-

Have too many irons in the fire

A. Engaged in too many enterprises at the same time

B. Facing too many problems at the same time

C. Said or done too many things at the same time

D. To incite the feeling amongst the people

 

Q. 39 Choose the explanation that catches the spirit of the idiom-

A fair weather friend

A. A friend who is fair to us at all the times

B. A friend who deserts us in difficulties

C. A friend whom we love the most

D. A friend who loves us the most

 

Q. 40 Choose the explanation that catches the spirit of the idiom-

A Panacea

A. An injection that serves as a life line

B. A lecture full of precepts

C. A strong drug that induces sleep

D. A single cure for all diseases or troubles

 

Q. 41 2/3 is a rational number whereas √2/√3 is

A. Also a rational number

B. An irrational number

C. Not a number

D. A natural periodic number

 

Q. 42 Greatest number which divides 926 and 2313, leaving 2 and 3 remainders, respectively, is 

A. 52

B. 54

C. 152

D. 154

 

Q. 43 A single discount equivalent to a discount series 15% and 5% is

A. 32%

B. 20%

C. 10%

D. 8.5%

 

Q. 44 By selling a cycle for Rs 2345, a student loses 19%. His cost price is nearly

A. Rs 4000

B. Rs 5000

C. Rs 3000

D. Rs 3500

 

Q. 45 Diagonals of a rhombus are 1 meter and 1.5 meter in lengths. The area of the rhombus is 

A. 0.75 m2

B. 1.5 m

C. 1.5 m2

D. 0.375 m2

 

Q. 46 An angle in a semi circle is

A. π

B. π/4

C. π/2

D. 2π

 

Q. 47 In a school for midday meal food is sufficient for 250 students for 33 days, if each student is given 125 gm meals. 80 more students joined the school. If same amount of meal is given to each student, then the food will last for

A. 20 days

B. 40 days

C. 30 days

D. 25 days

 

Q. 48 In a school of 500 students, 102 students can read Hindi and Tamil both, 200 students can read only Hindi. The students who can read Tamil are

A. 98

B. 402

C. 302

D. 300

 

Q. 49 The value of k for which kx + 3y – k + 3 = 0 and 12x + ky = k, have infinite solutions, is 

A. 0

B. -6

C. 6

D. 1

 

Q. 50 Table shows the daily expenditure on food of 25 households in a locality: Rs 100-150 150- 200 200-250 250-300 300-350 Households 4 5 12 2 2

The mean daily expenditure on food is

A. Rs 111

B. Rs 161

C. Rs 211

D. Rs 261

 

Q. 51 A box contains 24 marbles, some are green and others are blue. If a marble is drawn at random from the jar, the probability that it is green is 2/3. The number of blue balls in the jar is

A. 13

B. 12

C. 16

D. 8

 

Q. 52 The population of a city is 250 thousand. It is increasing at the rate of 2% every year. The growth in the population after 2 years is

A. 2500

B. 10000

C. 252000

D. 10100

 

Q. 53 If a point (x, y) in a OXY plane is equidistant from (-1,1) and (4,3) then

A. 10x + 4y = 23

B. 6x + 4y = 23

C. –x + y = 7

D. 4x + 3y = 0

 

Q. 54 Sum of first 15 multiples of 8 is

A. 960

B. 660

C. 1200

D. 1060

 

Q. 55 A rod of 2 cm diameter and 30 cm length is converted into a wire of 3 meter length of uniform thickness. The diameter of the wire is

A. 2/10 cm

B. 2/√10 cm

C. 1/√10 cm

D. 1/10 cm

 

Q. 56 Two straight poles of unequal length stand upright on a ground. The length of the shorter pole is 10 meters. A pole joins the top of the two vertical poles. The distance between the two tops is 5 meters. The distance between the poles along the ground is 4 meter. The area thus formed by the three poles with the ground is

A. 52 m2

B. 46 m2

C. 20 m2

D. 50 m2

 

Q. 57 Pipe A can fill a tank in 10 hours and pipe B can fill the same tank in 12 hours. Both the pipes are opened to fill the tank and after 3 hours pipe A is closed. Pipe B will fill the remaining part of the tank in

A. 5 hours

B. 4 hours

C. 5 hours 24 minutes

D. 3 hours

 

Q. 58 A ground 100×80 meter2 has two cross roads in its middle. The road parallel to the length is 5 meter wide and the other road is 4 meter wide, both roads are perpendicular to each other. The cost of laying the bricks at the rate of Rs. 10 per m2, on the roads, will be

A. Rs 7000

B. Rs 8000

C. Rs 9000

D. Rs 10000

 

Q. 59 If selling price of 10 articles is equal to cost price of 11 articles, then gain is

A. 8%

B. 9%

C. 8.5%

D. 10%

 

Q. 60 Angles of a quadrilateral are in the ratio 3 : 4 : 5 : 8. The smallest angle is

A. 54°

B. 72°

C. 36°

D. 18°

 

Q. 61 The Headquarter of European Union is situated in

A. England

B. Germany

C. France

D. Belgium

 

Q. 62 India in 2008 successfully put CHANDRAYAAN-1 into its initial orbit by

A. PSLV- C12

B. PSLV- C11

C. PSLV-14

D. GSLV-D3

 

Q. 63 Vishwanathan Anand retained the World Chess Championship in 2012 by defeating Boris Gelfand. Mr. Gelfand belongs to

A. Israel

B. Russia

C. Poland

D. USA

 

Q. 64 Kapilvastu Relics (fragments of Lord Buddha’s bone), for the second time in 114 years, recently travelled from India to

A. China

B. Sri Lanka

C. Myanmar

D. Japan

 

Q. 65 Dr. Norman Borlaugh is famous as father of the Green Revolution in 1960s. His initial goal was to create varieties of wheat adapted to the climate of

A. Mexico

B. India

C. USA

D. China

 

Q. 66 A feature ‘Bluetooth’ now common in mobile phones, gets its name from a

A. Chinese 10th Century King

B. UK Software Company

C. Greek Goddess

D. Danish 10th Century King

 

Q. 67 Which pair of states does not touch each other

A. Meghalaya, Manipur

B. Chhattisgarh, U.P.

C. Rajasthan, Punjab

D. J.K., H.P.

 

Q. 68 Baglihar dam, is constructed on river

A. Ravi

B. Chenab

C. Indus

D. Sutlej

 

Q. 69 Navjivan Trust was instituted with the objectives of propagating peaceful means of attaining third Swaraj, by

A. Mohan Das Karam Chand Gandhi

B. Lala Lajpat Rai

C. Bal Gangadhar Tilak

D. Dr. Ambedkar

 

Q. 70 World Cup Football 2014 and Olympics 2016 will be held in

A. USA

B. Brazil

C. Russia

D. South Africa

 

Q. 71 In 2012-13, India’s target is to restrict the fiscal deficit to x % of the GDP, where x is

A. 10

B. 8.3

C. 15

D. 5.1

 

Q. 72 POSCO steel project to come up but being strongly protested by the people is located in 

A. Chhattisgarh

B. Jharkhand

C. Odisha

D. Andhra Pradesh

 

Q. 73 Bharat Ratna and Padma Awards in our country were instituted in the year

A. 1952

B. 1954

C. 1962

D. 1964

 

Q. 74 Who was crowned the Miss World 2012 on August 18, 2012?

A. Ms. Jessica Kahawaty

B. Ms. Wenxia Yu

C. Ms. Vanya Mishra

D. Ms. Sophie Moulds

 

Q. 75 Vishwaroopam is a 2013 Tamil spy thriller film written, directed and co-produced by _________ who also enacts the lead role.

A. Prakash Raj

B. Rajni Kant

C. Kamal Haasan

D. Chiranjeevi

 

Q. 76 Vijay Kumar, who clinched a silver medal in London Olympics in 2012 is associated with 

A. Boxing

B. Shooting

C. Weight Lifting

D. Wrestling

 

Q. 77 Sushil Kumar who won a silver medal in London Olympics in 2012 is associated with

A. Shooting

B. Boxing

C. Wrestling

D. Weight lifting

 

Q. 78 How many medals did India win in London Olympics 2012?

A. 3

B. 4

C. 5

D. 6

 

Q. 79 The present Pope chosen in March 2013 hails from which country?

A. Brazil

B. Mexico

C. Argentina

D. Panama

 

Q. 80 The first person to set foot on the moon on July 20, 1969 and who died on Aug. 25, 2012 was

A. Nevil Armstrong

B. Neil Armstrong

C. Gagan Narang

D. Michael Phelps

 

Q. 81 In which place, on Feb 21, 2013, two powerful explosive devices planted on bicycles had exploded in Andhra Pradesh?

A. Dilrubnagar

B. Dilsukhnagar

C. Dilkushnagar

D. Dilshaknagar

 

Q. 82 On which date Maha Kumbh Mela started in Prayag this year?

A. 14-1-2013

B. 1-1-2013

C. 26-1-2013

D. 4-1-2013

 

Q. 83 On which date International Women’s Day is celebrated?

A. 18th March

B. 8th March

C. 28th March

D. 18th Feb

 

Q. 84 Duration of which Five Year Plan was 2007-2012

A. X

B. XII

C. IX

D. XI

 

Q. 85 As per census 2011, which State has the lowest sex ratio (877:1000)?

A. Punjab

B. Haryana

C. Uttar Pradesh

D. Bihar

 

Q. 86 Hugo Chavez who died on March 5, 2013 after losing his battle with cancer, was the President of which country?

A. Argentina

B. Cuba

C. Brazil

D. Venezuela

 

Q. 87 Carlos Slim, who tops the list of world’s wealthiest people, for the fourth year in a row, belongs to which country?

A. USA

B. England

C. Mexico

D. Germany

 

Q. 88 In the name P. Chidambaram, the present Union Finance Minister, what does ‘P’ stands for? 

A. Palghat

B. Pallakudi

C. Palaniappan

D. Perumal

 

Q. 89 The earlier name of which city was New Amsterdam?

A. Chicago

B. California

C. Washington D.C.

D. New York City

 

Q. 90 The grant of Diwani of Bengal, Bihar and Orissa is associated with

A. Nawab Shuja-ud-Daula

B. Nawab Asif-ud-Daula

C. Shah Alam I

D. Shah Alam II

 

Q. 91 Where did Kuchipudi, an eminent dance form, originate?

A. Kerala

B. Andhra Pradesh

C. Uttar Pradesh

D. Tamil Nadu

 

Q. 92 Light Year is the unit of

A. Time

B. Distance

C. Light

D. None of these

 

Q. 93 The leaning tower of Pisa does not fall because

A. It is tapered at the top

B. It covers a large base area

C. Its C.G. is inside the tower

D. The vertical line passing through the C.G. of the tower falls within its base

 

Q. 94 “Paradise Regained” was written by

A. John Milton

B. Michel Angelo

C. John Keats

D. Lord Byron

 

Q. 95 Which is the richest temple in India?

A. Balaji Temple of Tirupathi

B. Padmanabha Swamy Temple of Thiruvananthapuram

C. Shirdi Sai Baba Temple

D. Jagannatha Temple of Puri

 

Q. 96 Who founded the Red Cross?

A. Henry Dunant

B. Alexander

C. James Cook

D. Bismark

 

Q. 97 World Literacy Day is celebrated on

A. 5th September

B. 6th September

C. 8th September

D. None of these

 

Q. 98 South African Paralympics icon Oscar Pistorius has been accused of killing

A. Julia Kamp

B. Reeva Steenkamp

C. Pistorius Kamp

D. Shakeera Kamp

 

Q. 99 In the month of March, 2013 the Supreme Court of India issued a notice that the

ambassador of the following country shall not leave India without the permission of the Supreme Court

A. Germany

B. Maldives

C. Italy

D. Nepal

 

Q. 100 The Constitution (One Hundred Seventeenth Amendment) Bill, 2012 makes provisions regarding

A. Reservation in matters of promotions for Scheduled Castes and Scheduled Tribes

B. Reservation in matters of appointments for Scheduled Castes and Scheduled Tribes

C. Reservation in matters of appointments and promotions for Scheduled Castes and Scheduled Tribes

D. None of the above

 

Q. 101 The number of High Courts in India is

A. 18

B. 24

C. 21

D. 28

 

Q. 102 The last British Emperor of India was

A. King George I

B. King George III

C. King George V

D. King George VI

 

Q. 103 Palaeolithic period is also known as

A. Mesolithic Age

B. Late Stone Age

C. Old Stone Age

D. Neolithic Age

 

Q. 104 Capital of India was transferred from Calcutta to Delhi in the year

A. 1901

B. 1911

C. 1921

D. 1922

 

Q. 105 The chairman of Fundamental Rights Sub-Committee of the Constituent Assembly was 

A. Rajendra Prasad

B. B.R. Ambedkar

C. Jawaharlal Nehru

D. J.B. Kripalani

 

Q. 106 The Environment Protection Act was passed by the Parliament of India in the year

A. 1976

B. 1986

C. 1996

D. 2006

 

Q. 107 International Year of Biodiversity is/was/will be

A. 2010

B. 2011

C. 2012

D. 2014

 

Q. 108 The first Shaka king in India was

A. Rudradaman

B. Menadar

C. Maues

D. Damanrudra

 

Q. 109 Potential Energy is described by the expression:

A. PE= mgh

B. PE= ngh

C. PE= oph

D. PE= pph

 

Q. 110 Where was 16th NAM Summit held?

A. Tehran

B. Mehran

C. Turban

D. Bagdad

 

Questions: 111 – 113

Direction for Questions 111 to 113 : Answer the following questions based on the statements given below:

i. There are 3 poles on each side of the road.

ii. These six poles are labelled A, B, C, D, E and F.

iii. The poles are of different colours namely Golden, Silver, Metallic, Black, Bronze and White.

iv. The poles are of different heights.

v. E, the tallest pole, is exactly opposite to the Golden colours pole.

vi. The shortest pole is exactly opposite to the Metallic coloured pole.

vii. F, the Black coloured pole, is located between A and D.

viii. C, the Bronze coloured pole, is exactly opposite to A.

ix. B, the Metallic coloured pole, is exactly opposite to F.

x. A, the White coloured pole, is taller than C but shorter than D and B.

 

Q. 111 What is the colour of the pole diagonally opposite to the Bronze coloured pole?

A. White

B. Silver

C. Metallic

D. Golden

 

Q. 112 Which is the second tallest pole?

A. A

B. D

C. B

D. Cannot be determined

 

Q. 113 Which is the colour of the tallest pole?

A. Golden

B. Silver

C. Bronze

D. None of these

 

Questions: 114 – 115

Directions for questions 114 and 115 : Answer the questions on the basis of the information given below:

The head of a newly formed Government desires to appoint five of the six elected ministers P, Q, R, S, T and U to portfolios of Foreign, Industry and Commerce, Agriculture, Rural Development and Human Resources. U does not want any portfolio if S gets one of the five. R wants either Foreign or Human Resources or no portfolio. Q says that if S gets Industry and Commerce or Rural Development then she must get the other one. T insists on a portfolio if P gets one. 

 

Q. 114 Which of the following is a valid assignment

A. P- Foreign, Q- Industry and Commerce, R- Agriculture, S- Rural Development, THuman Resources

B. R- Foreign, S- Industry and Commerce, P- Agriculture, Q- Rural Development, THuman Resources

C. P- Foreign, Q- Industry and Commerce, T- Agriculture, S- Rural Development, UHuman Resources

D. Q- Foreign, U- Industry and Commerce, T- Agriculture, R- Rural Development, PHuman Resources

 

Q. 115 If P gets Foreign and R gets Human Resources, then which is NOT a valid assignment of Agriculture and Rural Development

A. S- Agriculture, Q- Rural Development

B. U- Agriculture, Q- Rural Development

C. Q- Agriculture, T- Rural Development

D. Q- Agriculture, S- Rural Development

 

Q. 116 A related pair of words is followed by four pairs of words or phrases. Select the pair that best expresses a relationship similar to the one expressed in the question pair.

Action : Reaction

A. Introvert : Extrovert

B. Assail : Defend

C. Diseased : Treatment

D. Death : Rebirth

 

Q. 117 A related pair of words is followed by four pairs of words or phrases. Select the pair that best expresses a relationship similar to the one expressed in the question pair.

Sorrow : Misery

A. Love : Obsession

B. Amity : Harmony

C. Happiness : Joy

D. Enemy : Hatred

 

Q. 118 A related pair of words is followed by four pairs of words or phrases. Select the pair that best expresses a relationship similar to the one expressed in the question pair.

Drama : Audience

A. Brawl : Vagabonds

B. Game : Spectators

C. Art : Critic

D. Movie : Actors

 

Q. 119 A related pair of words is followed by four pairs of words or phrases. Select the pair that best expresses a relationship similar to the one expressed in the question pair.

Nuts : Bolts

A. Nitty : Gritty

B. Bare : Feet

C. Naked : Clothes

D. Hard : Soft

 

Q. 120 A related pair of words is followed by four pairs of words or phrases. Select the pair that best expresses a relationship similar to the one expressed in the question pair.

Book : Author

A. Rain : Flood

B. Light : Switch

C. Symphony : Composer

D. Song : Music

 

Questions: 121 – 125

Directions for questions 121 to 125 : Each question contains a statement on relationship and a question regarding relationship based on the statement. Select the correct option.

 

Q. 121 Moni is daughter of Sheela. Sheela is wife of my wife’s brother. How Moni is realted to my wife?

A. Cousin

B. Niece

C. Sister

D. Sister-in-law

 

Q. 122 Annu is daughter of my mother’s brother Abahi. Pari is grand daughter of my mother. Pari should call Annu as

A. Maternal Aunt

B. Sister

C. Cousin

D. Niece

 

Q. 123 Markandey is Rajiv’s mother’s father. Markandey is three brothers. One of them has grandson Abahi. Rajan is son of Abahi. Rajan is related to Rajiv as

A. Brother

B. Nephew

C. Cousin

D. Uncle

 

Q. 124 Deepak said to Nitin, “That boy playing with the football is the younger of the two brothers of the daughter of my father’s wife”. How is the boy playing football related to Deepak?

A. Son

B. Brother

C. Cousin

D. Brother-in-law

 

Q. 125 Pointing to a woman in the photograph, Rajesh said, “The only daughter of her grandfather is my wife”. How is Rajesh related to that woman?

A. Uncle

B. Father

C. Maternal Uncle

D. Brother

 

Questions: 126 – 130

Direction for Questions 126 to 130 : Read the information given below to answer the questions.

i. Kareena’s dieting schedule consists of having only one fruit on a given day of the week.

ii. Dietician has prescribed banana, papaya, pomegranate, apple and grape from Sunday to Friday, one day being a fasting day. Kareena cannot eat any fruit on Saturday.

iii. Pomegranate day is neither on the first day nor on the last day but earlier than the papaya day.

iv. Apple day is on the immediate next day of papaya day.

v. Banana day is on the immediate previous day of the fasting day.

vi. Apple day and grape day must have a gap of two days between them.

vii. Grape day is the day immediately following the fasting day.

 

Q. 126 Which of the following is the fasting day?

A. Monday

B. Tuesday

C. Wednesday

D. Thursday

 

Q. 127 Banana day and apple day have a gap of how many days between them?

A. One

B. Two

C. Three

D. Four

 

Q. 128 Which day is grape day?

A. Monday

B. Tuesday

C. Thursday

D. Sunday

 

Q. 129 Which day is pomegranate day?

A. Sunday

B. Monday

C. Tuesday

D. Wednesday

 

Q. 130 Which of the following is the correct statement?

A. Apple day is after papaya day.

B. Banana day is on Wednesday.

C. Fasting day is on Tuesday.

D. Papaya day is earlier than banana day.

 

Q. 131 The question contains one statement and two courses of action I and II. Assuming the statements to be true, decide which of the two courses of action most logically follows. Statement: Indian children are very talented but are instead weak in science and mathematics.

I: Teaching and textbooks are not available in mother language.

II: Education based on experiments in both the subjects is lacking.

A. If only I follows.

B. If only II follows.

C. If either I or II follows.

D. If neither I nor II follows.

 

Q. 132 The question contains one statement and two courses of action I and II. Assuming the statements to be true, decide which of the two courses of action most logically follows.

Statement: Despite of child labour laws, children can be seen working in hotels, shops, houses, very frequently.

I: The government should not make such laws which cannot be enforced.

II: A proper education system for the primary level particularly for lower caste community may eradicate this problem.

A. If only I follows.

B. If only II follows.

C. If either I or II follows.

D. If neither I nor II follows.

 

Q. 133 The question contains one statement and two courses of action I and II. Assuming the statements to be true, decide which of the two courses of action most logically follows. 

Statement: Kyoto protocol on environment is signed by almost every country of the world. 

I: As a result air, water and soil pollution have come down.

II: Increasing production of automobiles, refrigerators and fertilisers do not affect our environment.

A. If only I follows.

B. If only II follows.

C. If either I or II follows.

D. If neither I nor II follows.

 

Q. 134 The question contains one statement and two courses of action I and II. Assuming the statements to be true, decide which of the two courses of action most logically follows.

Statement: School dropout rate is very high in the rural areas as children support their parents in income earning activities.

I: Public awareness programme on primary education should be expanded immediately to educate parents.

II: Compensation is not a remedy.

A. If only I follows.

B. If only II follows.

C. If either I or II follows.

D. If neither I nor II follows.

 

Q. 135 The question contains one statement and two courses of action I and II. Assuming the statements to be true, decide which of the two courses of action most logically follows.

Statement: Smoking is one of those human weaknesses which tend to test the will power of the smoker of the edge.

I: It is very difficult for the smoker to give up smoking even if they want to do so.

II: Human beings have other weaknesses as well

A. If only I follows.

B. If only II follows.

C. If either I or II follows.

D. If neither I nor II follows.

 

Q. 136 Direction for Questions 136 to 140 : Complete the series by choosing the correct option. 

0, 1, 1, 2, 3, 5, 8, 13, 21, ?

A. 34

B. 35

C. 33

D. 36

 

Q. 137 Complete the series by choosing the correct option.

A -10, E -15, I -20, M -25, …….

A. Q-5

B. Q-30

C. P-30

D. R-30

 

Q. 138 Direction for Questions 136 to 140 : Complete the series by choosing the correct option. 

17, 36, 74, 150, ? , 606

A. 250

B. 303

C. 300

D. 302

 

Q. 139 Complete the series by choosing the correct option.

2, 1, 4, 3, 6, 5, 8, ?

A. 9

B. 10

C. 7

D. 8

 

Q. 140 Complete the series by choosing the correct option.

1, 4, 27, 256. ?

A. 625

B. 3125

C. 3025

D. 1225

 

Questions: 141 – 145

Direction for Questions 141 to 145: In each question below are given two statements numbered I and II. You have to take the two given statements as true even if they seem to be at variance with commonly known facts. Read all the conclusions and then decide which of the given conclusions logically follow from the given statements, disregarding commonly known facts.

 

Q. 141 I: All vegetables have gravy.

II: All lunch has vegetable.

A. All lunch has gravy.

B. All gravy has lunch.

C. Both (A) and (B).

D. None of the above

 

Q. 142 I: Karan Johar is a good director.

II: Directors are intelligent.

A. All intelligent are directors.

B. Karan Johar is intelligent.

C. Both (A) and (B).

D. None of the above

 

Q. 143 I: Some blues are green.

II: Pink is green

A. Some blue is pink.

B. Some green is pink.

C. If either (A) or (B) follows.

D. Some pinks are blues.

 

Q. 144 I: All boys are tall.

II: All Punjabi are tall.

A. All boys are Punjabi.

B. Some boys are Punjabi.

C. Both of the above.

D. None of the above.

 

Q. 145 I: All girls go to the college.

II: Rina does not go to the college.

A. Rina is not a girl.

B. Going to college is not essential to be a girl.

C. Rina is a girl.

D. None of the above.

 

Questions: 146 – 150

Direction for Questions 146-150 : Read the information given below to answer the questions. A, B, C, D, E, F, G and H want to have a dinner on a round table and they have worked out the following seating arrangements.

i. A will sit beside C

ii. H will sit beside A

iii. C will sit beside E

iv. F will sit beside H

v. E will sit beside G

vi. D will sit beside F

vii. G will sit beside B

viii. B will sit beside D

 

Q. 146 Which of the following is wrong?

A. A will be to the immediate right of C

B. D will be to the immediate left of B

C. E will be to the immediate right of A

D. F will be to the immediate left of D

 

Q. 147 Which of the following is correct?

A. B will be to the immediate left of D

B. H will be to the immediate right of A

C. C will be to the immediate right of F

D. B will be to the immediate left of H

 

Q. 148 A and F will become neighbours if:

A. B agrees to change her sitting position

B. C agrees to change her sitting position

C. G agrees to change her sitting position

D. H agrees to change her sitting position

 

Q. 149 During sitting:

A. A will be directly facing C

B. B will be directly facing C

C. A will be directly facing B

D. B will be directly facing D

 

Q. 150 H will be sitting between:

A. C and B

B. A and F

C. D and G

D. E and G

 

Questions: 151 – 200

Each question in this section consists of legal propositions/principles (hereinafter referred to as ‘principle’) and facts. These principles have to be applied to the given facts to arrive at the most reasonable conclusion. Such principles may or may not be true in the real sense, yet you have to conclusively assume them to be true for the purposes of this section. In other words, in answering the following questions, you must not rely on any principles except the principles that are given herein. The objective of this section is to test your interest towards study of law, research aptitude and problem solving ability even if the “most reasonable conclusion” arrived at may be unacceptable for any other reason. It is not the objective of this section to test your knowledge of law. 

 

Q. 151 Principle: When an offer is accepted by a person to whom it is made, it becomes a promise. But this promise will become legally binding only when the acceptance of the offer is unconditional.

Facts: Ram makes an offer to sell his house to Shyam for Rs.50 lacs. Shyam accepts this offer but wants to pay the price of the house in five quarterly instalments. Ram does not agree to it. Thereafter Shyam agrees to pay the price of the house in the way as originally desired by Ram. But Ram does not reply to it. Can Shyam compel Ram to sell his house to him?

A. Shyam can compel Ram to sell his house because Shyam ultimately agrees to pay the price as originally desired by Ram

B. Shyam can compel Ram to sell his house because Shyam in the first instance substantially complied with the desire of Ram

C. Shyam can compel Ram to sell his house because Ram’s offer does not exclude the payment of price in instalments

D. Shyam can not compel Ram to sell his house because Shyam imposes a new condition about payment of price of the house while accepting the offer which is not ultimately accepted by Ram.

 

Q. 152 Principle: Generally an agreement without consideration is not valid. Therefore, in order to make a valid agreement, some consideration which may have some value in the eyes of law, is essentially required.

Facts: William has an old car of which he makes seldom use. He voluntarily enters into an agreement with Smith to sell this car for rupees ten thousand. Thereafter one Anson approaches William and offers to buy that car for rupees one lac as the car was one which Anson has been searching for long. Now William wants to cancel his agreement with Smith and refuses to deliver the car to him saying that consideration (price) for the car  romised by Smith is negligible and, therefore, agreement with him can not be said to be valid one. 

A. William can cancel his agreement with Smith as the consideration involved in that is really inadequate.

B. William can not cancel his agreement with Smith as the sale of car for rupees ten thousand was voluntary and this price has some value in the eyes of law.

C. William can cancel his agreement with Smith as he was ignorant about the value / price of the car for which it could be sold.

D. William can cancel his agreement with Smith as he is entitled to get full market value/price of his car.

 

Q. 153 Principle: In order to be illegible to appear in the semester examination, a student is required to attend, under all circumstances, at least 70% of the total classes held in that semester as per University rules.

Facts: Anand, an economically poor but a very brilliant student of LL.B. final semester, while going to his University by cycle received some leg injuries in road accident. Consequently Anand could not attend his classes for one week as he was advised rest by his doctor for that period. Due to this absence from the University, Anand failed to have 70% attendance essential to appear in the examination and, therefore, he was debarred from appearing in the examination by the University authorities. Anand challenges this decision in the court of law

A. Anand will succeed in the court of law as the accident was beyond his control 

B. Anand will definitely get favour of the court on humanitarian ground as he comes from a economically poor family and may not afford to take readmission

C. Anand will not succeed as he could very easily fulfill eligibility criteria for appearing in the examination by being reasonably regular in the class throughout the semester.

D. Anand will succeed as requirement of 70% attendance may be declared arbitrary and, therefore, unreasonable by the court of law.

 

Q. 154 Principle: A seller of goods cannot transfer better rights than he himself possesses in the goods sold to the buyer.

Facts: Komal leaves his watch by mistake on a seat in the park. Sonal finds that watch and immediately sells the same for good price to Monal who without inquiring whether Sonal is its owner or not. Komal later on claims that watch from Monal. Decide whether Komal can succeed?

A. Komal cannot succeed as Monal has paid good price of the watch.

B. Komal cannot succeed as Monal is unaware of the fact that Sonal is not its owner.

C. Komal can not succeed as it was his carelessness and nothing else which enabled Sonal to sell the watch to Monal.

D. Komal can not succeed as Sonal is merely finder of the watch and, therefore, cannot transfer ownership rights thereon to Monal.

 

Q. 155 Principle: All citizens shall have the Fundamental Right to carry on any occupation, trade or business. But reasonable restrictions on the exercise of such rights can be imposed by law in the interest of the general public.

Facts: A large number of persons had been carrying on the business of dyeing and printing in Rajkot area for the last 25 years providing employment to about 30000 families. From these business places untreated dirty water was being discharged on the roads thereby causing damage to the public health. A notice, therefore, was given to close this business till necessary measures to protect public health as provided under the environmental statutes were taken by those business men.

A. Notice can not be justified as it will cause loss of employment to 30000 families.

B. Notice can not be justified as it amounts to violation of the fundamental right of the persons who have been carrying on the business for the last 25 years.

C. The notice can not be justified on the ground of damage to public health as the persons in that area have been voluntarily residing for long and have become used to that environment.

D. The notice can be justified as the right to business is not absolute and reasonable restriction can be imposed by law in the interest of the public.

 

Q. 156 Principle: A Contract can not be enforced by or against a person who is not a  party to it. However, where some benefit is conferred on third party by the contract itself, there third party can be allowed to enforce that contract to get such benefit.

Facts: Dinesh is liable to pay Rs. 50000/- to Suresh. In order to discharge this liability Dinesh enters into a contract with Ramesh by which Dinesh sells his car to Ramesh for Rs. 1 lac. Ramesh takes the delivery of the car and promises/ assures to pay its price at the earliest. Dinesh separately informs Suresh about this contract for his satisfaction. Ramesh fails to pay the car’s price. Suresh wants to join Dinesh in filing suit against Ramesh for the recovery of price of the car. Whether Suresh is entitled to do so?

A. Suresh is entitled to do so because the contract was made for his benefit.

B. Suresh is entitled to do so because Dinesh is liable to him and discharge of this liability depends upon the payment of the price of the car by Ramesh.

C. Suresh is not entitled to do so because liability of Dinesh does not depand upon any assurance of Ramesh.

D. Suresh is not entitled to do because he is not a party to the contract between Dinesh and Ramesh.

 

Q. 157 Principle: If a contract is made by post between two persons living in two different cities, then the contract is said to be complete as soon as the letter of acceptance is properly posted, and the place of completion of the contract is that city where acceptance is posted. It is worth mentioning here that in every contract there is always an offer from one party and the acceptance of the offer from the other party.

Facts: Sani, a resident of Patna, gives an offer by post to sell his house for Rs. 25 lacs to Hani, a resident of Allahabad. This offer letter is posted on 1st January 2013 from Patna and reaches Allahabad on 7th January 2013. Hani accepts this offer and posts the letter of acceptance on 8th January 2013 from Allahabad which reaches Patna on 16th January 2013. But Sani presuming that Hani is not interested in accepting his offer, sells his house to Gani at same price on 15th of January, 2013. Hani files a suit against Sani for the breach of contract in the competent court of Allahabad. Whether Hani will succeed?

A. Hani can not succeed as Sani can not be compelled by law to wait for the answer from Hani for an indefinite period of time.

B. Hani can not succeed as he could use some other effective and speedy mode for communicating his acceptance in minimum possible time.

C. Hani can succeed as he properly posted the letter of acceptance and the delay was beyond his control

D. Hani can succeed as contract became complete in the eyes of law on the date of posting the letter of acceptance.

 

Q. 158 Principle: He, who goes to the court of law to seek justice, must come with clean hands. 

Facts: P enters into a contract with S under which S has to construct a house for P and has to complete the same within one year from the date of the contract. This contract includes two very important terms. According to first term if there is price hike of the materials to be used in the construction, then the escalation charges at a particular rate shall be payable by P to S. According to second term if the construction of the house is not completed within the period prescribed for it, then S will have to pay penalty at a particular rate to P. Before the completion of the construction work the workers of S go on strike and strike continues up to three months even after the expiry of one year. After that period workers return and the construction work again starts. During the last three months period of strike there was a considerable rise in the price of the building material. S claimed escalation cost from P. P did not agree to it. S filed a suit in the court of law either to order the payment of the price of the building material on the basis of escalated price or to allow him to stop the work without incurring any penal liability towards P. 

A. S will succeed as strike by his workers was unexpected and beyond his control.

B. S can succeed as there is an escalation clause in the contract

C. S can not succeed as he has failed to complete the construction work in time and strike can not be treated as a valid excuse for delay in work.

D. S can succeed if he pays penalty to P for delay.

 

Q. 159 Principle: If the object of an agreement is or becomes unlawful or immoral or opposed to public policy in the eyes of law, then the courts will not enforce such agreements. Law generally prohibits Child labour.

Facts: P enters into an agreement with T by which P has to let his house to T for two years and T has to pay Rs. 20000.00 per month to P as rent. T starts a child care centre in that house. But after some time in order to earn some money for the maintenance of the centre, T starts sending the children of the centre on the rotation basis to work for four hour a day in some nearby chemical and hazardous factories. When P comes to know about this new development, he asks T either to stop the children from working in factories or to leave his house immediately. T neither agrees to leave the house nor to stop the children from working in the factories. P files a suit in the court of law for appropriate relief/ action

A. P can not succeed as the agreement was for the two years and it can not be terminated before the expiry of that period

B. P can not succeed as the object at the time of making of the agreement was not clear C. P will succeed as the object of the agreement has become unlawful.

D. P will not succeed if T agrees to share the wages of the children with P

 

Q. 160 Principle: whosoever by his act or omission causes environmental pollution shall be held liable for any loss caused by such pollution. It shall be no defence in such cases that all due diligence or reasonable care was taken while carrying out the act or omission in question. 

Facts: Hari is carrying on a chemical and fertilizer industry near a bank of a river. In order to prevent and control any kind of harm to the environment, suitable waste treatment and disposal plants were installed in the factory. Due to some sudden mechanical/ technical problem, these plants ceased to work properly and, therefore, caused environmental pollution, which ultimately caused substantial harm to the environment and to the people living around the factory. Victims of such pollution file a suit for suitable remedy.

A. Victims can not succeed as necessary precautions to prevent any harm were taken by Hari.

B. Victims can not succeed as the mechanical/ technical problem was sudden and, therefore, beyond the control of Hari.

C. Victims can succeed as it is the duty of Hari to see that no harm is caused to the environment/ people due to his activity under any circumstances.

D. Victims could succeed if treatment/ disposal plant were not installed in the factory

 

Q. 161 Principle: If a person transfers movable or immovable property with its full ownership and without any consideration to some other person, then it is called a gift.

Facts: S, who has no child of his own, makes a gift of his house worth Rs. 25 lacs to his nephew R. After completing all the legal formalities required for a valid gift, S says to R that in case of need R will provide that house to S for use without any questions. R does not react to it. After one year of this gift, S really needs that house and request R to make the house available to him, but R refuses to do so.

A. R cannot refuse as he got the house without paying any consideration for that.

B. R cannot refuse as S is without children.

C. R can refuse as he has become full owner of the house.

D. R can refuse as he himself may be in need of that house.

 

Q. 162 Principle: An agreement to do an act impossible in itself cannot be enforced by a court of law.

Facts: Ramesh agrees with his girl friend Shilpa to pluck stars from the sky through his extraordinary will power, and bring them down on earth for her within a week. After the expiry of one week, Shilpa filed a suit for damages against Ramesh for the breach of contract as Ramesh failed to perform his promise.

A. Shilpa can succeed in getting damages as Ramesh has deceived her.

B. Ramesh cannot be held liable as he honestly believes that his love for Shilpa is true and, therefore, he will succeed in his endeavour.

C. The court cannot entertain such suits as the act promised under the agreement is impossible in itself.

D. Ramesh can be held liable for making an absurd promise.

 

Q. 163 Principle: An agreement may be oral or written. However, if a law specifically requires that an agreement must be in writing then the agreement must be in writing. A law specifically requires that the agreements relating to transfer of the copyright in novel between an author of a novel and the producer of a motion picture must be in writing.

Facts: The author of a novel, Love at Lost Sight, had several rounds of discussion with a producer of motion picture regarding making of a motion picture based on Love at Lost Sight. During the last discussion, they decided to make a motion picture on Love at Lost Sight. The producer made a motion picture on Love at Lost Sight after making a payment of Rs. 10, 00,000/- (Ten Lac Only) in cash to the author who happily accepted this amount as full and final payment. Later on, on the advice of his lawyer, the author brought a case in a court of law against the producer on the ground that there is no written agreement between the producer and him.

A. The author is likely to succeed in the case as the agreement is not in accordance with the law

B. The author cannot succeed in the case as he has given his consent to the agreement

C. The author is not likely to succeed in the case because he has already accepted the amount of Rs 10,00,000/- as full an final payment

D. The author can succeed in the case as the consideration is not adequate

 

Q. 164 Principle: A person is said to be of sound mind for the purpose of making a contract if, at the time when he makes it, he is capable of understanding it and of forming a rational judgment as to its effect upon his interest.

Facts: X who is usually of sound mind, but occasionally of unsound mind enters into a contract with Y when he (X) is of unsound mind. Y came to know about this fact afterwards and now wants to file a suit against X.

A. X cannot enter into contract because he is of unsound mind when he entered into contract.

B. X can enter into contract but the burden is on the other party to prove that he was of unsound mind at the time of contract.

C. X can enter into contract but the burden is on X to prove that he was of sound mind at the time of contract.

D. None of these.

 

Q. 165 Principle: Whosoever commits any act forbidden by the Indian Penal code with a view to obtain the consent of any person to enter into an agreement, he cannot get the agreement enforced by law but the person whose consent has been so obtained may get the agreement enforced by law. The Indian Penal code defines various offences and prescribes punishments therefor.

Facts: A obtains the consent of B to enter into an agreement by an act amounting to criminal intimidation under the Indian Penal Code. A brings a case against B for performance of agreement.

A. A will succeed in the case

B. A may succeed in the case

C. B will succeed in the case

D. B will not succeed in the case

 

Q. 166 Principle: A contract between the father and his son is a contract of utmost good faith. In such a type of contract law presumes that at the time of entering into the contract the father was in a position to dominate the will of his son. Where one of the parties was in position to dominate the will of the other party, the contract is enforceable only at the option of the party whose will was so dominated.

Facts: Ram had advanced a sum of Rs. 10,000/- to his minor son Shyam. When Shyam became major, his father Ram misused his parental position and entered into an agreement with Shyam and obtained a bond from him for a sum of Rs. 30,000 in respect of the advance. Whether this agreement is enforceable?

A. The agreement is enforceable against Shyam only for Rs. 10,000/-, the actual amount of money advanced to him

B. The agreement is enforceable against Shyam for Rs. 30,000/- because he has signed the bond 

C. The agreement is enforceable against Shyam because he was major at the time of agreement.

D. The agreement is not enforceable against Shyam because Ram has misused his position as father to obtain an unfair advantage.

 

Q. 167 Principle: When at the desire of one person, any other person has done or abstained from doing something, such act or abstinence or promise is called a consideration for the promise. Only a promise coupled with consideration is enforceable by law.

Facts: X, the uncle of Y, made a promise to pay him an amount of Rs. 10,000/- as reward if Y quits smoking and drinking within one year. X also deposited the above mentioned amount in a bank and informed Y that the said amount will be paid to him if he quits smoking and drinking within one year. Within a period of six months of making the promise X died. After the expiry of one year of making the promise by X, Y made a request to the legal heirs of X demanding the promised money. The legal heirs of X declined the request of Y.

A. The promise of X to Y is enforceable by law because Y has quitted smoking and drinking.

B. The promise of X to Y is not enforceable by law because Y has benefitted by quitting smoking and drinking.

C. The promise of X to Y is enforceable by law because the amount of Rs. 10, 000/- has been deposited in a bank.

D. The promise of X to Y is enforceable by law because X has died within a period of six months of making the promise.

 

Q. 168 Principle: ‘Work’ means literary work, artistic work, dramatic work, musical work, cinematographic film and sound recording but does not include acting in a cinematographic film. Only the works, as enumerated above, can be protected under copyright law.

Facts: A very famous actor acted in a cinematographic film. The actor was also the producer and director of cinematographic film.

A. The acting of the actor can be protected under copyright law

B. The acting of the actor can be protected under copyright law only as an artistic work

C. The acting of the actor cannot be protected under copyright law

D. The acting of the actor cannot be protected under copyright law as a cinematographic film

 

Q. 169 Principle: In case where the government is a party, the government shall be the first owner of the copyright in the work unless there is an agreement to the contrary.

Facts: The Government of the State of X entered into an agreement with a retired Professor of Botany. The Professor agreed to write a text book on Botany. The Government agreed to pay a sum of Rs. Ten Lacs to the Professor for this work.

A. The Government of the State of X shall be first owner of copyright in the text book.

B. The Professor shall be first owner of copyright in the text book.

C. Both the Government of the State of X and the Professor shall be the joint owners of copyright in the text book.

D. The Professor shall be first owner of copyright in the text book only if he refuses to accept the amount of Rs Ten Lacs from the Government.

 

Q. 170 Principle: Licence is an agreement whereby the owner of the copyright agrees to grant an interest in the copyright to the licencee. Assignment is an agreement whereby the owner of the copyright transfers all the property rights to the assignee. Property right is a bundle of rights consisting of right to possess, right to use, right to alienate, and the right to exclude others.

Facts: A, an owner of copyright in a cinematographic film enters into an agreement with B, a film distributer. B agrees to distribute the film only in Mumbai. A also enters into many such agreements with other distributers for distribution of his film in other cities.

A. The agreement between A and B is more in the nature of assignment than in the nature of licence.

B. The agreement between A and B is more in the nature of licence than in the nature of assignment.

C. The agreement between A and B is both in the nature of assignment and licence.

D. The agreement between A and B is neither in the nature of assignment nor in the nature of licence.

 

Q. 171 Principle: Qui facit per alium facit per se, i.e. he who does things through others does it himself.

Facts: Nisha, the owner of a car, asked her friend Saurabh to take her car and drive the same to her office. As the car was near her office, it hit a pedestrian Srikant on account of Saurabh’s negligent driving and injured him seriously. Now Srikant files a suit for damages against Nisha.

A. Nisha is not liable as it was the negligence of Saurabh.

B. Saurabh is solely liable as Nisha was not driving the car.

C. Nisha is liable as Saurabh was driving under her authority and for her purpose.

D. Saurabh will be exempted from liability under the principle of inevitable accident.

 

Q. 172 Principle: Words describing quality of things cannot be registered as trade mark. However, such words may be registered as trade mark if they acquire a secondary meaning. Words acquire secondary meaning when people start associating the descriptive words with a person specific.

Facts: A hatchery located in Raipur is owned by ‘X’. X has been using the slogan “new laid eggs sold here” since 1970 to describe the quality of eggs sold in his hatchery. Over a period of time because of continuous use of this slogan, people started associating this slogan with X. X filed an application for registration of the words “new laid eggs” as trade mark in the year 1970.

A. The words will be registered as trade mark

B. The words will not be registered as trade mark

C. The words may be registered as trade mark

D. The words may not be registered as trade mark

 

Q. 173 Principle: Whoever takes away anything from the land of any person without that person’s consent is said to commit theft. A thing so long as it is attached to the earth is not the subject of theft; but it becomes capable of being the subject of theft as soon as it is severed from the earth.

Facts: Y cuts down a tree standing on the land of X with the intention of dishonestly taking the tree out of X’s possession without the consent of X. But Y is yet to take away the tree out of X’s possession.

A. Y has committed theft as soon as he came to the land of X

B. Y has committed theft as soon as the tree has been completely cut down by him

C. Y has committed theft as soon as he has started cutting down the tree

D. Y has not committed theft because he is yet to take away the tree out of X possession

 

Q. 174 Principle: res ipsa loquitur i.e. the thing speaks for itself.

Facts: Seema got herself operated for the removal of her uterus in the defendant’s hospital, as there was diagnosed to be a cyst in one of her ovaries. Due the negligence of the surgeon, who performed the operation, abdominal pack was left in her abdomen. The same was removed by a second surgery.

A. Surgeon cannot be held responsible because it is merely a human error.

B. Surgeon can be held responsible but Seema will have to prove in the court of law that the surgeon was grossly negligent

C. Surgeon will be responsible and Seema need not to prove surgeon’s negligence because presence of abdominal pack in her abdomen is sufficient proof therefor.

D. None of the above

 

Q. 175 Principle: Whoever takes away with him any minor person less than sixteen years of age if a male, or less than eighteen years of age if a female out of the keeping of parents of such minor person without the consent of such parent, is said to kidnap such minor person.

Facts: A female born on January 01, 1995 got admitted to an undergraduate program of a reputed University on July 01, 2012. She became friendly with one of the boys, born on June 01, 1994, of her class. The boy and the girl decided to marry. The parents of the boy agreed but the family of the girl did not agree. On December 15, 2012 the girl made a call from her Blackberry to the boy. The girl told the boy to come in his car at a particular place and time. The boy reached the stipulated place before the stipulated time. He waited there for about half an hour. The girl reached the stipulated place. She opened the door of the car and sat beside the boy who was on the driving seat. Without exchanging any pleasantries, the boy drove the car to an unknown place. The father of the girl lodged an FIR in the nearest police station on January 20, 2013.

A. The boy has committed the offence of kidnapping

B. The boy has not committed the offence of kidnapping

C. The boy has not committed the offence of kidnapping for there is a delay in filing the FIR

D. The boy has not committed the offence of kidnapping because the girl was his classmate

 

Q. 176 Principle: Whoever voluntarily has carnal intercourse against the order of nature with any man, woman or animal shall be punished.

Facts: Two adult men were found engaged in carnal intercourse by the police. The police arrested the men and produced them before the Court.

A. Court will punish the police officer who arrested the men

B. Court will not punish the men for they were adults

C. Court will punish the men

D. Court may be requested to declare the law unconstitutional

 

Q. 177 Principle: Wherever the causing of a certain effect, or an attempt to cause that effect, by an act or by an omission, is an offence, it is to be understood that the causing of that effect partly by an act and partly by an omission is the same offence.

Facts: A intentionally omitted to give food to his father. He also used to beat his father. Consequently A’s father died

A. A did not commit any offence

B. A committed only the offence of omitting to give food to his father

C. A committed only the offence of beating of his father

D. A committed the offence of killing of his father

 

Q. 178 Principle: Nothing is an offence which is done by a child under seven years of age.

Facts: A, a child born on January 01, 2005 killed another child ‘B’ on December 30, 2011.

A. A has committed no offence.

B. A has committed the offence as it is heinous crime

C. Killing of one child by another child is not an offence

D. A has not committed the offence for on the date of killing of B, A was a minor

 

Q. 179 Principle: A pact, other than a pact to commit suicide, to suffer any harm is not an offence, provided the age of the person who has given his consent to suffer harm is above eighteen years.

Facts: A enters into a pact with B, a boy of less than 18 years of age, to fence with each other for amusement. They agreed to suffer any harm which, in the course of such fencing, may be caused without foul play.

A. A, while playing fairly, hurts B, A commits no offence

B. A, while playing only unfairly, hurts B, A commits an offence

C. A, while playing fairly, hurts B, A commits an offence

D. A, while playing unfairly, hurts B, A commits no offence

 

Q. 180 Principle: When an act, which would otherwise be an offence, is not that offence by reason of the youth, the want of maturity of understanding, the unsoundness of mind or the intoxication of the person doing that act, every person has the same right of private defence against that act which he would have if the act were that offence. Nothing is an offence which is done in the exercise of the right of private defence.

Facts: A, under the influence of madness, attempts to kill B. B in order to save his life causes grievous hurt to A.

A. A has committed an offence

B. A has not committed an offence

C. B has committed an offence

D. B has not committed any offence

 

Q. 181 Principle: Mere silence as to facts likely to affect the decision of a person to enter into a contract does not amount to fraud, unless his silence is in itself equivalent to speech. 

Facts: A sells to B a horse which A knows to be of unsound mind. B says to A that if A does not say anything about the state of mind of horse, then B shall presume that the horse is of sound mind. A says nothing to B about the mental condition of horse.

A. A has committed fraud

B. A has committed misrepresentation

C. There cannot be a fraud because A says nothing about the mental condition of the horse

D. There can not be a fraud because B did not ask A whether the horse is of sound mind

 

Q. 182 Principle: Whoever by words, either spoken or written brings or attempts to bring into hatred or contempt, or excites or attempts to excite disaffection towards the Government established by law in India shall be punished. However, comments expressing disapprobation of the administrative or other action of the Government without exciting or attempting to excite hatred, contempt or disaffection, do not constitute an offence. 

Facts: A renowned professor of economics wrote a critical comment on the economic policies of the Government of India in a National Daily. This piece of writing generated academic debate not only in the print media but also on television and internet. A student of law asked the fellow Indians on a social networking website to assemble at a particular place for peaceful and silent demonstration against the said economic policies on a stipulated date and time. The crowed assembled at that venue and started shouting antigovernment slogans. Police arrested the professor.

A. The professor has committed the offence

B. The professor has not committed any offence

C. The student of law has committed the offence

D. The crowed has committed an offence

 

Q. 183 Principle: Where two or more persons have made a complaint for the grant of compulsory licence to the Copyright Board, the licence shall be granted to that complainant only who, in the opinion of the Copyright Board, would best serve the interests of the general public. 

Facts: Four persons made a complaint for the grant of compulsory licence to the Copyright Board.

A. Licence shall be granted to only one complainant

B. Licence may be granted to two complainants

C. Licence may be granted to three complainants

D. Licence must be granted to all the four complainants

 

Q. 184 Principle: Any police officer, not below the rank of a sub-inspector, may, if he is satisfied that an offence in respect of the infringement of copyright in any work has been, is being, or is likely to be, committed, seize without warrant, all copies of the work, and all plates used for the purpose of making infringing copies of the work, wherever found, and all copies and plates so seized shall, as soon as practicable, be produced before a Magistrate.

Facts: A Superintendent of Police (SP) conducted a raid on a shop and found pirated copies of books. The SP formed an opinion that infringement of copyright is taking place. He arrested the shop owner without warrant in the light of above mentioned propositions. 

A. The arrest of the shop owner was within the power of the SP

B. The arrest of the shop owner was not within the power of the SP

C. The shop owner can never be arrested

D. SP was not competent to know whether infringement of copyright has taken place

 

Q. 185 Principle: Whoever attempts to commit an offence punishable by the Indian Penal Code and in such attempt does any act towards the commission of the offence, shall be punished. Stealing is an offence punishable by the Indian Penal Code.

Facts: A makes an attempt to steal some jewels by breaking open a box, and after so opening the box, finds that there is no jewel in it.

A. A has committed no offence

B. A has committed the offence of stealing

C. A has attempted to commit the offence of stealing

D. None of the above

 

Q. 186 Principle: Whoever by words either spoken or intended to be read, or by signs or by visible representations, makes or publishes any imputation concerning any person intending to harm, or knowing or having reason to believe that such imputation will harm, the reputation of such person, is said to defame that person.

Facts: In a community there is a custom of stealing shoes of bridegroom during the marriage ceremony. The shoes of the bridegroom were stolen by Y. ‘A’ announced that Z has stolen the shoes. Everyone present in the marriage party started staring at Z with great surprise. Z felt very ashamed.

A. A defamed Z

B. A did not defame Z

C. A defamed Z for Z felt very ashamed

D. A defamed the whole marriage party

 

Q. 187 Principle: An employer is liable for the negligence of his employee. But an employer is not liable for the negligence of his employee if the victim of such negligence is one of his other employees.

Facts: ‘A’ and ‘B’ were working in a factory as unskilled labourers. A was carrying a basket of stones on his head. B was sitting on the ground. When A crossed B, all of a sudden a stone fell down from the basket and hit B on his head. B died immediately.

A. The owner of the factory will be liable

B. A and the owner of the factory shall be jointly liable

C. The owner of the factory will not be liable

D. None of the above

 

Q. 188 Principle: Damages are the money recompense, as far as money can do, for the violation of a right.

Facts: A, an Indian citizen, having a right to vote, was not allowed to cast his vote on the polling booth, by the returning officer. Name of A was mentioned in the voter’s list. A has also reported at the polling booth in time. However, the candidate in whose favour A would have cast his vote won the election. A filed a suit claiming damages.

A. A will be entitled to damages

B. A will not be entitled to damages

C. A will be entitled to only nominal damages

D. A will be entitled to exemplary damages

 

Q. 189 Principle: When a party to a contract has refused to perform, or disabled himself from performing, his promise in its entirety, the other party may put an end to the contract. 

Facts: A engaged B on April 12 to enter his service on June 1, but on May 11, A wrote to B that his services would not be needed. On May 22, B joined C for employment.

A. B must wait till June 1

B. B must have joined C on May 11

C. B is not bound to wait till June 1

D. A must pay damages to B

 

Q. 190 Principle: When a person voluntarily agrees to suffer some harm, he is not allowed to complain for that.

Facts: ‘A’ was one of the spectators at a formula one car race, being held at Gurgaon, on a track owned by one ‘M’ company. During the race, there was a collision between two racing cars, one of which was thrown away amidst spectators, thereby causing an injury to ‘A’. ‘A’ claims damages for the injuries caused to him.

A. M company will be liable for damages because the injury was caused during the race organized by it

B. M company will not be liable for damages because A had come to see the race on his own will.

C. M company will not be liable for damages because the collision between the cars was beyond its control

D. M company will be liable because it has earned huge revenue by way of sale of tickets for the event

 

Q. 191 Principle: An interest which is created on a transfer of property and depends upon the fulfillment of a condition will fail if the fulfillment of the condition is impossible or is forbidden by law or is of such a nature that, if permitted, it would defeat the provisions of any law or is fraudulent or involves or implies injury to the person or property of another or the court regards it as immoral or opposed to public policy. 

Facts: A gives Rs. Ten Lacs to B on condition that B shall marry A’s daughter C. On the date on which A gave Rs. Ten Lacs to B, C was dead.

A. B’s interest in Rs Ten Lacs fails because of impossibility

B. B’s interest in Rs Ten Lacs fails because of immorality

C. B’s interest in Rs Ten Lacs fails because of prohibition by law

D. B’s interest in Rs Ten Lacs does not fail

 

Q. 192 Principle: A condition precedent must be complied with before the happening of the event to which such a condition is attached. Fulfillment of such a condition after the happening of the event is no fulfillment of condition.

Facts: A transfers Rs. 5000 to B on condition that he shall marry with the consent of C, D and E. As C, D and E had to go abroad for some business purposes and as the date of

marriage was already fixed, therefore, B marries without the consent of C, D and E, but obtains their consent after the marriage when C, D and E return to their country.

A. B has fulfilled the condition

B. B has not fulfilled the condition

C. B was free to marry any one without the consent of any body

D. B must divorce his wife as he married her without fulfilling the condition

 

Q. 193 Principle: In an agreement, a condition subsequent must be complied with, to claim the benefit of that agreement.

Facts: A agrees to transfer a farm to B, provided that, if B does not go to England within three years after the date of the agreement, his interest in the farm shall cease. B does not go to England within the term prescribed.

A. B’s interest in the farm continues

B. B’s interest in the farm does not continue

C. B has a fundamental right to go to England or not to go to England and hence the condition was illegal

D. The agreement between A and B was void

 

Q. 194 Principle: Existence of all the alleged facts is relevant whether they occurred at the same time and place or at different times and places.

Facts: A, a permanent resident in a foreign country who never visited India, is accused of waging war against the Government of India by taking part in an armed insurrection in which property is destroyed, troops are attacked and prisons are broken open.

A. The existence of all the above mentioned alleged facts is relevant

B. Only the alleged fact that A is accused of waging war against the Government of India is relevant

C. The fact that A was a permanent resident in a foreign country who never visited India is not relevant

D. Only the alleged fact of taking part by A in armed resurrection is relevant

 

Q. 195 Principle: Whoever desires any Court to give judgment about any legal right or liability which depends on the existence of those facts which he asserts, must prove that those facts exist.

Facts: A asserts that B, C and D have committed an offence of criminal conspiracy and therefore A desires a Court to give judgment that B, C and D shall be punished for that crime which A says B, C and D have committed.

A. A must prove that B, C, and D have committed the crime

B. B, C, and D must prove that they have not committed the crime

C. A must prove that B, C, and D were present at the place of crime

D. Police must prove that B, C, and D have committed the crime

 

Q. 196 Principle: The fact that any person was born during the continuance of a valid marriage between his mother and any man, or within two hundred and eighty days after its dissolution, the mother remaining unmarried, shall be conclusive proof that he is the legitimate son of that man, unless it can be shown that the parties to the marriage had no access to each other at any time when he could have been begotten.

Facts: X and Y married on January 15, 1995. Y, the wife of X, never left her parental home and never went to her husband’s home. A boy was born to Y on July 15, 1995. For the Court: 

A. There shall be a conclusive proof that the boy is the legitimate son of X

B. There shall be no conclusive proof that the boy is the legitimate son of X

C. There shall be a conclusive proof that the boy is the illegitimate son of X

D. There shall be no evidence at all.

 

Q. 197 Principle: An unlawful interference with a person’s use or enjoyment of land, or some right over, or in connection with it, is a nuisance in law of tort.

Facts: During the scarcity of onions, long queues were made outside the defendant’s shop who having a license to sell fruits and vegetables used to sell only 1 Kg. of onion per ration card. The queues extended on to the highway and also caused some obstruction to the neighbouring shops. The neighboring shopkeepers filed a suit for nuisance against the defendant. Which one of the following decisions will be correct in this suit?

A. The defendant is liable for nuisance

B. The defendant is not liable for nuisance

C. The defendant is liable under the principle of strict liability

D. The plaintiff’s suit should be decreed in favour of the neighbouring shopkeeper

 

Q. 198 Principle: Every agreement in restraint of the marriage of any person, other than a minor, is void

Facts: Qadir Khan died in a road accident. Two co-widows, Sultana and Marjina enter into an agreement that if any of them will remarry, would forfeit her right to her share in the deceased husband’s property.

A. The agreement is void because it was restraint of marriage

B. The agreement is not void because no restraint was imposed upon either of two widows for remarriage.

C. The restraint was partial so agreement is valid

D. None of the above

 

Q. 199 Principle: Nothing is an offence merely by reason of its being done with the knowledge that it is likely to cause harm, if it be done without any criminal intention to cause harm, and in good faith for the purpose of preventing or avoiding other harm to a person or property.

Facts: Mr. Sharman, the Italian captain of a steam vessel, suddenly and without any fault or negligence on his part, finds himself near the Kochi coast in such a position that before he can stop his vessel, he must inevitably run down a boat B with twenty or thirty passengers on board, unless he changes the course of his vessel, and that by changing his course, he must incur risk of running down a boat C with only two passengers on board, which he may possibly clear. Whether Sharman has committed an offence? 

A. Sharman has committed no offence because this was done out of necessity

B. Sharman can be held responsible for the act of criminal negligence

C. Sharman can be held responsible for culpable homicide

D. This is a clear case of accident so Sharman cannot be held responsible

 

Q. 200 Principle: Only the Parliament or the State Legislatures have the authority to enact laws on their own. No law made by State can take away a person’s fundamental right. 

Facts: Parliament enacted a law, which according to a group of lawyers is violating the fundamental rights of traders. The group of lawyers filed a writ petition against the Parliament for enacting such law and requests the court to quash the law and to direct the Parliament to make a new law.

A. No writ would lie against the Parliament, as the Court has no authority to direct the Parliament to enact or re-enact a law.

B. The Court can quash the existing law if it violates fundamental right and can direct to make a new law.

C. The Court can quash the existing law if it violates fundamental right but cannot direct the Parliament to make a new law.

D. None of these.

 

Answer Sheet 
Question 1 2 3 4 5 6 7 8 9 10
Answer D B D B C A A B B C
Question 11 12 13 14 15 16 17 18 19 20
Answer A B C C C C C B C D
Question 21 22 23 24 25 26 27 28 29 30
Answer C C C C C C B B D D
Question 31 32 33 34 35 36 37 38 39 40
Answer A B D C A C A A B D
Question 41 42 43 44 45 46 47 48 49 50
Answer B D B C A A D D C C
Question 51 52 53 54 55 56 57 58 59 60
Answer D D A A C B C B D B
Question 61 62 63 64 65 66 67 68 69 70
Answer D B A B B D A B A B
Question 71 72 73 74 75 76 77 78 79 80
Answer D C B B C B C D C B
Question 81 82 83 84 85 86 87 88 89 90
Answer C A B D B D C C D D
Question 91 92 93 94 95 96 97 98 99 100
Answer B B D A A A C B C A
Question  101 102 103 104 105 106 107 108 109 110
Answer B D C B D B A C A A
Question  111 112 113 114 115 116 117 118 119 120
Answer D D B B D B C B C C
Question 121 122 123 124 125 126 127 128 129 130
Answer B A B B A A D B D A
Question 131 132 133 134 135 136 137 138 139 140
Answer B B D A D A B D C B
Question 141 142 143 144 145 146 147 148 149 150
Answer A B B D A C B D C B
Question 151 152 153 154 155 156 157 158 159 160
Answer D B C B D D D C C C
Question 161 162 163 164 165 166 167 168 169 170
Answer C C A A C D A C A B
Question 171 172 173 174 175 176 177 178 179 180
Answer C B D C A C D A C D
Question 181 182 183 184 185 186 187 188 189 190
Answer A B A B C B C A C B
Question 191 192 193 194 195 196 197 198 199 200
Answer A B B A A A A B A C

 

×

Hello!

Click one of our representatives below to chat on WhatsApp or send us an email to info@vidhyarthidarpan.com

×